Вы находитесь на странице: 1из 261

Hematology and Oncology:Question 1

A 67-year-old man in previously good health is


hospitalized because of a 2-day history of fever
and diminished consciousness. The patient
responds inconsistently to verbal commands.
His temperature is 39.5 °C (103.1 °F); he has
tachycardia, and his blood pressure is 80/58
mm Hg. There is no bleeding. His hemoglobin
is 12.1 g/dL, leukocyte count is 29,000/μL with
80% neutrophils, and platelet count is
20,000/μL. Which of the following studies
should be obtained in this patient?
A. Bone marrow aspiration and biopsy
B. Factor VIII level
C. Measurement of platelet-associated IgG
D. Measurements of fibrin D-dimer and total
fibrinogen
E. Bleeding time
Hematology and Oncology:Question 2
A 35-year-old woman has completed 6 months
of anticoagulation therapy for pulmonary
emboli. She had been using oral contraceptives
for the past 10 years but stopped when the
pulmonary emboli were detected. There is no
family history of thrombosis. Two weeks after
the discontinuation of warfarin, she undergoes
coagulation studies. She is found to be
heterozygous for the prothrom bin G2021 0A
mutation. On the basis of her history and
laboratory evaluation, what is the best
recommendation for this patient?
A. Resume oral anticoagulation therapy at a
target INR of 2 to 3
B. Resume oral anticoagulation at a target INR
of 1 .5 to 2 and advise her that it is safe to
begin taking oral contraceptives again
C. Advise the patient to take 81 mg of aspirin
each day
D. Prescribe a combination of aspirin and
dipyridamole
E. Advise the patient not to resume oral
contraceptives and that additional therapy is
not required for the thrombotic event
Hematology and Oncology:Question 3
A 22-year-old man with sickle cell disease has
four to six painful crises per year that require
parenteral hydration and narcotics. Two weeks
ago the patient developed an acute chest
characterized by fever, dyspnea, and the
radiographic appearance of pulmonary
infiltrates. The patient’s condition rapidly
responded to aggressive exchange transfusions
and empiric broad-spectrum antibiotics; he has
been asymptomatic for 1 week.
Laboratory studies:
Hemoglobin 10.4 g/dL
Hematocrit 32%
Leukocyte count 12,000/μL
Platelet count 277,000/μL
Hemoglobin electrophoresis shows the
following results.
Hemoglobin A (α2 β2) 62% (normal > 97.5%)
Hemoglobin A2(α2 δ2) 2% (normal< 2.5%)
Hemoglobin F (α2γ2) 1% (normal 0%)
Hemoglobin S (α2 βS2) 35% (normal 0%)
What is the best next step in this patient’s
management?
A. Long-term transfusion therapy
B. Hydroxyurea therapy
C. Consultation for therapeutic gene therapy
D. Consultation for therapeutic bone marrow
transplantation
E. Long-term oxygen therapy
Hematology and Oncology:Question 4
A 24-year-old woman is evaluated in the
emergency department because of fever,
chills, sore throat, and difficulty talking. Two
months ago she was diagnosed with
schizophrenia, and clozapine therapy was
initiated, at which time her complete blood
count was normal.
On physical examination, her tonsils are
swollen and red bilaterally; she has
submandibular lymphadenopathy and high
fever. Her temperature is 40 °C (104 °F),
pulse rate 140/min, and blood pressure is
90/50 mm Hg.
Her hemoglobin is 12 g/dL, hematocrit is 36%,
leukocyte count is 2000/μL, and platelet count
is 180,000/μL.
Throat and blood cultures are obtained; broad-
spectrum intravenous antibiotic therapy is
initiated and clozapine is discontinued.
Which of the following is the most appropriate
next step for this patient?
A. Bone marrow aspiration and biopsy
B. Initiation of subcutaneous granulocyte
colony-stimulating factor daily
C. Granulocyte transfusion
D. Search for a suitable bone marrow donor
E. Test for HIV

Hematology and Oncology:Question 5


A 46-year-old woman with alcoholism is
evaluated because of a 4-day history of nausea
and vomiting after a recent binge. She has
diffuse epigastric pain. She has been
hospitalized intermittently for years for medical
complications related to alcohol abuse,
including cirrhosis with ascites and bleeding
esophageal varices. On physical examination,
her temperature is 37.3 °C (99.1 °F), pulse
rate is 68/min, and blood pressure is 135/70
mm Hg. She has jaundice and is icteric. Her
lungs are clear to auscultation. The remainder
of her physical examination is unremarkable.
Laboratory studies:
Hemoglobin 7.1 g/dL
Leukocyte count 4500/μL
Platelet count 115,000/μL
Mean corpuscular volume 107 fL
Reticulocyte count 5.4%
Serum alkaline phosphatase 592 U/L
Serum aspartate aminotransferase 198 U/L
Serum alanine aminotransferase 166 U/L
Serum total bilirubin 9.2 mg/dL
Serum indirect bilirubin6.4 mg/dL
Serum albumin 2.2 g/dL
Serum folate 12 ng/mL
Which of the following diagnoses best explains
the findings?
A. Spur cell anemia
B. Folic acid deficiency
C. Hypersplenism
D. Acanthocytosis

Hematology and Oncology:Question 6


A 62-year-old man is evaluated because of
increasing fatigability. He previously felt fine.
He has recently been found to have
osteomyelitis underlying a diabetic foot ulcer
and is being treated with ciprofloxacin and
amoxicillin/clavulanate.
Laboratory studies:
Hemoglobin 9.8 g/dL
Mean corpuscular volume 79 fL
Serum creatinine0.9 mg/dL
Serum iron 38 μg/dL
Serum total iron-binding capacity 190 μg/dL
Serum ferritin 298 ng/mL
Which of the following constitutes appropriate
additional therapy?
A. Cyanocobalamin
B. Ferrous sulfate
C. Parenteral iron
D. Recombinant human erythropoietin and
ferrous sulfate
Hematology and Oncology:Question 7
A previously healthy, 59-kg (130-Ib), 35-year-
old woman delivers her fifth child after an
uncomplicated pregnancy and labor. Her blood
type is group O, Rh-positive. Soon after the
birth, vaginal bleeding is noted that requires
emergency transfusion of four units of packed
red blood cells. Results of an antibody screen
and direct antiglobulin test were negative at 2
months gestation and again when the patient
was hospitalized for the delivery. Seven days
later, faint scleral icterus is noted. The
hemoglobin is 9.5 g/dL (approximately 1 g/dL
less than at discharge after delivery). Serum
total bilirubin is 2.1 mg/dL, with a direct
component of 0.3 mg/dL. Serum lactate
dehydrogenase level is 200 U/L, and
haptoglobin is 20 mg/dL. Direct antiglobulin
(Coombs) testing shows IgG coating the
erythrocytes. There is no hemoglobinemia, and
the urine is negative for blood.
What is the most likely cause of hemolysis in
this patient?
A. Improper storage of the units of blood prior
to transfusion
B. Autoimmune hemolytic anemia
C. Glucose-6-phosphate dehydrogenase
deficiency
D. Erroneous transfusion of group A blood
E. Hemolytic transfusion reaction
Hematology and Oncology:Question 8
A 60-year-old woman is evaluated because of
a 1-year history of fatigue. Her menses ended
when she was 50 years old, and she has not
had any blood loss since that time. She also
notes that she is short of breath with exertion.
She recalls that a bone marrow biopsy was
done some years ago. She is not sure about
her family history because her father died
when she was young and her mother died of
“liver complications at an early age. On
physical examination, she has a bronze hue to
her skin. The liver is enlarged 3 cm below the
right costal margin. She has swelling of the
first and second metacarpophalangeal joints of
both hands.
Laboratory studies:
Hemoglobin 13.0 g/dL
Hematocrit 40%
Serum ferritin 1900 ng/mL
Transferrin saturation 70%
Fasting plasma glucose 115 mg/dL
Serum alanine aminotransferase 80 U/L
Serum aspartate aminotransferase 35 U/L
A copy of the bone marrow study that she
mentioned reveals only trace macrophage iron
stores. A polymerase chain reaction test of
mutations in the HFE gene is performed on the
basis of the clinical picture, and the patient is
found to be a homozygote for the C282Y
mutation.
Which of the following is the next best step in
this patients management?
A. Liver biopsy
B. Phlebotomy
C. Follow-up of liver function and iron status
D. Liver biopsy and phlebotomy
Hematology and Oncology:Question 9
A 37-year-old man is hospitalized because of
fever and right-sided chest pain. He has been
having fatigue and recently developed dyspnea
on exertion and intermittent chills. At the age
of 29 years he was diagnosed with stage III
Hodgkin’s disease and treated with multiagent
chemotherapy and radiation therapy. A year
ago, he developed mild anemia with no
obvious cause. He takes thyroid hormone
replacement when he remembers and has used
fexofenadine as needed for allergic rhinitis for
the past 5 years.
On physical examination, his temperature is
38.7 °C (101.7 °F), pulse rate is 112/min, and
blood pressure is 110/70 mmHg. There is
dullness at the right lower lung and egophony.
The hemoglobin is 8.5 g/dL, hematocrit is 26
%, leukocyte count is 2200/μL, and platelet
count is 70,000/μL.
What is the most likely diagnosis?
A. Relapsed Hodgkin’s disease
B. Sepsis
C. Hypothyroidism
D. Secondary myelodysplastic syndrome
E. Drug-induced bone marrow suppression
Hematology and Oncology:Question 10
A 43-year-old man with severe acquired
aplastic anemia has not responded to
immunosuppressive agents. He remains
neutropenic and transfusion-dependent for
platelets and red cells. He has an HLA-identical
brother who has been cleared as a donor for
his planned allogeneic stem cell transplant.
They are both cytomegalovirus-seronegative.
Which of the following would be prevented by
using irradiated cellular blood products for this
patient?
A. Cytomegalovirus disease
B. Alloimmunization
C. Transfusion-related graft-versus-host
disease
D. Febrile nonhemolytic transfusion reaction
E. Hemolytic transfusion reaction
Hematology and Oncology:Question 11
A 40-year-old Chinese woman is evaluated
because of pulmonary emboli that developed 1
week after she arrived in the United States
after a 16-hour airplane trip. She has no other
medical problems. She had taken oral
contraceptives for several years and had one
uncomplicated pregnancy but is not taking any
medications at present. Her older brother and
father both have a history of thrombosis.
Results of a complete blood count, prothrombin
time, and activated partial thromboplastin time
are normal.
Which of the following studies should be done
at this time?
A. Factor V Leiden and prothrombin G20210A
mutation analyses
B. Protein C, protein 5, and antithrombin III
measurements
C. Testing for a lupus anticoagulant
D. Screening tests for cancer including CT
scans of the chest, abdomen, and pelvis
E. No testing is required
Hematology and Oncology:Question 12
A 34-year-old woman is evaluated because of
progressive fatigue and recurrent attacks of
abdominal pain. She weighs 61.2 kg (135 Ib).
Her pulse rate is 110/min and her blood
pressure is 110/70 mm Hg. She is pale, and
her spleen is palpable 4 cm below the costal
margin.
Laboratory studies:
Hemoglobin 6 g/dL
Hematocrit 20%
Leukocyte count 2500/μL
Platelet count 80,000/μL
Reticulocyte count 10%
Blood smear Anisocytosis and
polychromatophilia
Haptoglobin 0 mg/dL
Serum lactate dehydrogenase 645 U/L
There is high level of hemosiderin in the urine.
What is the most likely cause of this patients
pancytopenia?
A. Autoimmune hemolytic anemia
B. Aplastic anemia
C. Myelodysplastic syndrome
D. Paroxysmal nocturnal hemoglobinuria
E. Acute myeloid leukemia

Hematology and Oncology:Question 13


A 45-year-old man with alcoholic cirrhosis is
evaluated because of hematemesis from active
bleeding due to esophageal varices. He
receives eight units of uncrossmatched group
O packed cells over 3 hours, but his blood
pressure remains unstable and his bleeding
continues.
When he was first evaluated in the emergency
department, his prothrombin time and partial
thromboplastin time were 34 s and 57 s,
respectively. The blood bank never received a
specimen for type and screen, and no
pretransfusion blood is available for testing.
What is the most appropriate transfusion
product to treat this patients bleeding?
A. Group O whole blood
B. Group AB whole blood
C. Group O fresh frozen plasma
D. Group AB fresh frozen plasma

Hematology and Oncology:Question 14


A 24-year-old man who moved to this country
from Sweden 15 years ago is evaluated
because of severe weakness and shortness of
breath. He has a 4-year history of HIV
infection and has had intermittent infections,
predominantly sinusitis, over the past year.
Two months ago, his hemoglobin was 10.8
g/dL, leukocyte count was 3900/μL, and
platelet count was 220,000/μL. He has been
noncompliant with his antiretoviral medications
because of the side effects, but admits to
taking trimethoprim-sulfamethoxazole twice
daily for the past week. His appetite is poor.
Cn physical examination, he exhibits extreme
pallor. His temperature is 37.5 °C (99.5 °F),
his pulse rate is 110/min, and his blood
pressure is 130/89 mm Hg. His respiratory rate
is 22/min at rest, increasing to 33/min when
walking. He has no skin lesions. His
conjunctivae are pale. He has generalized
shotty lymphadenopathy. Chest radiography is
normal. The remainder of his physical
examination is unremarkable.
Laboratory studies:
Hemoglobin 4 g/dL
Leukocyte count 2400/μL
Platelet count 129,000/μL
Mean corpuscular volume 101 fL
Reticulocyte count 0%
Peripheral blood smear No abnormalities
Serum total bilirubin 3.2 mg/dL
Serum indirect bilirubin2.8 mgldL
Serum vitamin B12 222 pg/mL
Serum folate 14 ng/mL
What is the most likely diagnosis?
A. Drug-induced marrow suppression
B. Parvoviral infection
C. HIV-induced marrow dysplasia
D. Autoimmune hemolytic anemia
Hematology and Oncology:Question 15
A 24-year-old black woman is evaluated at an
urgent care clinic because of sore throat,
temperature of 39.4 °C (103 °F), chills, and
severe fatigue of several weeks duration. She
does not take any prescription or over-the-
counter medications and she has never been
ill. She lives with her two sisters.
On physical examination she has pallor and
swollen and red tonsils bilaterally. Her pulse
rate is 120/min, and her blood pressure is
90/50 mm Hg. Her hemoglobin is 6 g/dL,
leukocyte count 100/μL, platelet count
17,000/μL, and reticulocyte count 0%.
After throat and blood cultures are obtained,
broad-spectrum intravenous antibiotic therapy
is administered, and the patient receives a
transfusion of cytomegalovirus-negative
irradiated red blood cells. Bone marrow
aspirate and biopsy show scattered plasma
cells and almost complete absence of myeloid
elements.
What is the most appropriate therapy for long-
term survival of this patient?
A. Intravenous immunoglobulin, 500 mg/kg
daily for 4 days
B. Allogeneic stem cell transplantation from an
HLA-identical sister
C. Oral melphalan and prednisone
D. Antithymocyte globulin and cyclosporine
E. Long-term transfusion and iron chelation
therapy
Hematology and Oncology:Question 16
A 63.5-kg (140-Ib) 29-year-old woman
underwent repeat thoracic spinal fusion
surgery. She had extensive bleeding and
received a total of 20 units of packed red blood
cells. The surgery lasted 3 hours and required
the use of 7 L of Ringers lactated solution.
As they started to close the incision, the
surgeons noticed diffuse bleeding from the
surgical site that was not easily controlled by
cauterization. An emergency complete blood
count showed a hemoglobin of 11.7 g/dL,
hematocrit of 35%, a leukocyte count of
6300/μL, and a platelet count of 65,000/μL.
What is the most appropriate choice of blood
product to stop this patient’s bleeding?
A. Platelet concentrates
B. Whole blood
C. Fresh frozen plasma
D. Cryoprecipitate

Hematology and Oncology:Question 17


A 54-year-old woman is evaluated because of
progressively worsening fatigue and inability to
sustain her work hours. Metastatic breast
cancer was diagnosed 6 months ago, and she
is currently receiving multiagent systemic
chemotherapy.
On physical examination, she is afebrile. Her
blood pressure is 120/80 mm Hg. She has total
alopecia and a left mastectomy scar.
Laboratory findings include a leukocyte count
of 3200/μL, hemoglobin of 8.5 g/dL,
hematocrit of 26%, platelet count of
142,000/μL, and serum ferritin level of 375
mg/dL.
Which of the following is the most appropriate
management of this patient?
A. Daily administration of granulocyte colony-
stimulating factor subcutaneously
B. Cessation of chemotherapy
C. Weekly adminstration of erythopoietin
subcutaneously
D. Administration of androgens
E. Administration of vitamin E
Hematology and Oncology:Question 18
A 46-year-old woman is referred for
preoperative evaluation before undergoing
resection of a newly diagnosed glioblastoma.
Her hemoglobin is 12.5 g/dL, leukocyte count
is 6700/μL, and platelet count is 193,000/μL.
The surgeon is particularly interested in an
opinion concerning her risk of bleeding.
Which of the following will provide the best
estimate of her surgical risk of bleeding?
A. Bleeding time
B. Platelet Function Analyzer-i 00 analysis
C. Platelet aggregation studies
D. Medical history, including outcomes of
previous surgical procedures
E. Prothrombin time and partial thromboplastin
time
Hematology and Oncology:Question 19
A 42-year-old woman with rheumatoid arthritis
is found be anemic, with a hemoglobin of 11.0
g/dL. Her serum iron level is 21μg/dL, and iron
replacement with ferrous sulfate, 325 mg three
times daily, is started.
On a return visit 3 months later, the patient’s
hemoglobin level remains unchanged. The
patient claims to be taking ferrous sulfate as
directed and reports mild epigastric discomfort
and constipation since starting. On rectal
examination, stool is black but negative for
occult blood. There has been no change in the
patient’s menstrual blood loss, which has
typically been light.
Which of the following is the next appropriate
step in this patient’s management?
A. Change to an iron polysaccharide complex
B. Order a serum ferritin test
C. Add ascorbic acid
D. Administer parenteral iron
Hematology and Oncology:Question 20
A 67-year-old woman is evaluated because of
a 3-month history of dry cough and
progressive fevers. She denies having chills
but notes intermittent drenching night sweats.
She was treated 6 months ago with
corticosteroids for polymyalgia rheumatica.
She currently takes prednisone, 5 mg every
other day, with reasonable symptom control.
She smoked two packs of cigarettes daily for
20 years. Her family history is unremarkable,
except that her husband was treated for
tuberculosis 20 years ago. On physical
examination, her temperature is 38.6 °C (101 .
5 °F), pulse rate is 86/min, and blood pressure
is 132/80 mm Hg. She is anicteric. Abdominal
examination reveals a palpable spleen tip. The
remainder of her physical examination is
unremarkable.
Radiograph of the chest shows a diffuse
micronodular pattern confirmed on CT scan of
the chest. Abdominal CT scan confirms mild
splenomegaly and shows a 1.5-cm periportal
lymph node. Blood cultures (including fungal
culture), urine cultures, and sputum cultures
(including stains for acid-fast bacteria) show
no growth. Bronchoscopy with bronchoalveolar
lavage is nondiagnostic for infectious or
neoplastic disorders. Laboratory studies:
Hemoglobin 10.1 g/dL
Leukocyte count 12,400/μL
Platelet count 625,000/μL
Mean corpuscular volume 89 fL
Reticulocyte count 1.3%
Differential 62% neutrophils, 10% bands, 3%
metamyelocytes, 15% lymphocytes, 6%
eosinophils, 2% nucleated erythrocytes, 2%
monocytes
Serum alkaline phosphatase 360 U/L
Serum aspartate aminotransferase 112 U/L
Serum alanine aminotransferase 178 U/L
Serum total bilirubin 1.1 mg/dL
Serum indirect bilirubin0.7 mg/dL
Serum albumin 3.2 g/dL
Skin tests for tuberculosis and Trichophyton
are negative; she is anergic.
What would be the most appropriate next step
in the management of this patient?
A. Liver biopsy
B. Bone marrow biopsy
C. Splenectomy
D. Abdominal (periportal) lymph node
aspiration
Hematology and Oncology:Question 21
A 32-year-old woman with sickle cell disease is
evaluated in the emergency department
because of diffuse bone and joint pains typical
of sickle cell crisis. In the past week, she has
noted generalized malaise, a low-grade fever,
and mild exertional dyspnea without cough.
The patient states that her last painful crisis
occurred more than 6 months ago and was
treated with rehydration, intravenous narcotic
analgesics, and two units of packed red blood
cells.
On physical examination, the patient appears
uncomfortable. Her temperature is 38.4 °C
(101.2 °F), pulse rate 108/min, respiratory
rate 20/min, blood pressure 126/92 mm Hg.
She is mildly icteric. Bibasilar crackles are
audible on pulmonary examination. Her
abdomen is soft and nontender; there is no
palpable organomegaly. Transcutaneous pulse
oximetry with the patient breathing room air
indicates 95% arterial oxygen saturation.
Results of chest radiography are
unremarkable. Laboratory studies: Hemoglobin
4.2 g/dL
Hematocrit 11 .8%
Mean corpuscular volume 88 fL
Mean corpuscular hemoglobin 22 pg
Mean corpuscular hemoglobin concentration 34
g/dL
Leukocyte count 1 2,000/μL
Platelet count 200,000/μL
Reticulocyte count 0.2%
What is the most likely cause of the patients
current status?
A. Parvovirus B19 infection
B. Alloantibodies against transfused
erythrocytes
C. Folate deficiency
D. Splenic sequestration
Hematology and Oncology:Question 22
A 67-year-old man who had peripheral
vascular surgery 2 months ago is hospitalized
because of substernal chest pain and
electrocardiographic changes consistent with
myocardial ischemia. His hemoglobin is 12.2
g/dL, leukocyte count is 7300/μL, and platelet
count is 235,000/μL. An infusion of
unfractionated heparin is initiated, but pain
persists intermittently. Two days after
admission, his repeat platelet count is
90,000/μL.
What is the most appropriate management for
this patient?
A. Continue the heparin infusion, monitor the
platelet count daily
B. Continue the heparin infusion and order an
ELISA test for heparin-platelet factor IV
antibodies.
C. Discontinue the heparin infusion
D. Add an antiplatelet drug
E. Discontinue the heparin infusion and
institute thrombin-inhibitor therapy
Hematology and Oncology:Question 23
Four days ago, a 30-year-old woman with
ulcerative colitis required transfusion with
packed red cells for recurrent gastrointestinal
bleeding during a flare of her disease. The
blood bank distributed group O, Rh-negative
units for her because they were the only
antigen-negative units available. The antibody
screen performed in the emergency
department did not show any additional
antibodies.
This patient has received many red cell
transfusions since her diagnosis 10 years ago.
Her blood type is group A, and she is Rh-
positive. She has a history of positive response
to a red cell antibody screen, with anti-Kell,
anti-Duffy(a), and anti-E identified.
On the day after her present transfusion, the
patients hemoglobin level increases by 3 g/dL.
The bleeding subsides after high-dose
corticosteroid therapy but starts again as the
dose is lowered. The patients blood pressure is
stable, but her physician requests that
additional units be prepared for her in case the
bleeding becomes more active. The blood bank
refuses the request until a new compatibility
sample is sent.
The new compatibility sample is needed to
detect which of the following?
A. New alloantibodies
B. A hemolytic transfusion reaction
C. Residual group O red cells
D. New autoantibodies
E. Antibodies missed in the first sample
Hematology and Oncology:Question 24
A 55-year-old man who is HIV-positive is
evaluated because of weakness, shortness of
breath, and a dry, nonproductive cough. He
also reports dizziness, cold intolerance, and an
inability to continue his regular daily jogging
routine. Physical examination reveals pallor, a
flow murmur, and bilateral crackles in the
lungs.
Laboratory studies:
Hemoglobin 7 g/dL
Mean corpuscular volume 75 fL
Red cell distribution width 17
Reticulocyte count 0.8 %
Leukocyte count 8700/μL
Serum ferritin 180 ng/mL
Serum transferrin 180 mg/dL
Serum iron 10 μg/dL
A chest radiograph shows an interstitial
infiltrate consistent withPneumocystis carinii
pneumonia.
Which of the following is the most appropriate
approach to the anemia?
A. Assume the patient has the anemia of
chronic disease; direct diagnosis and treatment
to the underlying inflammatory process
B. Initiate treatment with ferrous sulfate and
observe the patients response over the next
several weeks
C. Treat the patient with erythropoietin
D. Perform a bone marrow aspirate and stain
with iron stain; if negative, evaluate the
gastrointestinal tract for evidence of bleeding
Hematology and Oncology:Question 25
In an office visit for an annual checkup, a 46-
year-old man reports that he has had malaise
and intermittent sweats for the past few
months but has been able to continue his job
as a high school teacher. Two years ago he
was treated for stage III diffuse large-cell non-
Hodgkin’s lymphoma with six cycles of
cyclophosphamide, doxorubicin, vincristine,
and prednisone (CHOP) and attained complete
remission. He takes simvastatin for
hypercholesterolemia and hydrochlorothiazide
for hypertension.
On physical examination, he has
lymphadenopathy: a 3-cm right axillary node
and a 2-cm right supraclavicular node. His
spleen tip is palpable. Laboratory evaluation
shows mild normochromic, normocytic anemia
and an elevated serum lactate dehydrogenase
level. CT scans of his chest and abdomen
reveal additional mediastinal and
retroperitoneal lymphadenopathy.
What is the best next step in this patient’s
management?
A. A 2-week course of oral antibiotics
B. Referral for salvage chemotherapy and
autologous stem cell transplantation
C. A repeat physical examination and CT scans
in 3 months
D. Referral for treatment with investigational
agents
E. A repeat course of CHOP
Hematology and Oncology:Question 26
A 50-year-old woman requests hormone
replacement therapy (HRT) for severe hot
flashes. Family studies done after her daughter
developed deep venous thrombosis in
association with use of oral contraceptives
showed the patient to be heterozygous for the
factor V Leiden mutation. Subsequently,
several other members of her large extended
family were found to have the factor V Leiden
mutation, but none has yet had a thrombotic
event, including several older than 65 years.
The patient is healthy, and results of a physical
examination are within normal limits.
Which of the following regimens is most
appropriate for management of this patient?
A. Initiate HRT along with warfarin, with a
target INR of 1.5 to 2
B. Initiate HRT along with enoxaparin and
warfarin, with a target INR of 2 to 3
C. Initiate HRT along with warfarin, with a
target INR of 2 to 3
D. Recommend against the use of HRT; initiate
anticoagulant prophylaxis at times of increased
thrombotic risk
E. Recommend against the use of HRT and
prescribe aspirin therapy
Hematology and Oncology:Question 27
A 37-year-old woman is referred for evaluation
of the new onset of thrombocytopenia. She
underwent orthotopic cardiac transplantation 4
months ago following the development of
postpartum cardiomyopathy. Function of the
allograft has been good with only one episode
of mild rejection several weeks after transplant
that was successfully managed by changing
her immunosuppressive regimen. Her current
medications include cyclosporine, tacrolimus,
trimethoprim-sulfamethoxazole, and
furosemide. Physical examination shows no
petechiae, purpura, or splenomegaly.
Laboratory studies reveal a hemoglobin of 11.9
g/dL, leukocyte count of 5000/μL, and platelet
count of 30,000/μL. Peripheral blood smear
shows reduced platelets but is otherwise
normal.
What is the optimal management of this
patient?
A. Discontinuation of trimethoprim-
sulfamethoxazole
B. Bone marrow aspiration and biopsy
C. Platelet transfusion
D. Administration of epsilon-aminocaproic acid
E. Discontinuation of immunosuppressive drugs

Hematology and Oncology:Question 28


A 25-year-old pregnant woman is evaluated at
the end of her first trimester. Three years ago,
she sustained deep venous thrombosis while
taking an oral contraceptive preparation. Her
medical history is otherwise unremarkable and
she is not currently taking anticoagulants or
other medications. She recently underwent
screening for thrombophilia, and no
abnormalities were identified. There is no
family history of venous thrombosis.
Which of the following is the best management
recommendation?
A. Monitor her pregnancy and initiate low-
molecular-weight heparin at prophylactic doses
after delivery and continue for 6 weeks
B. Immediately initiate prophylactic doses of
low-molecular-weight heparin and continue
prophylaxis for 6 weeks after delivery
C. Immediately initiate therapeutic doses of
low-molecular-weight heparin and continue
therapy for 6 weeks after delivery
D. Terminate the pregnancy because of the
high risk of thrombosis and obstetric
complications E. Do not administer
anticoagulation before or after delivery unless
she develops symptomatic venous
thromboembolism
Hematology and Oncology:Question 29
A 27-year-old woman is found to have a
platelet count of 50,000/μL. Her medical
history is unremarkable; she has not had
excessive bruising, epistaxis, or menorrhagia.
The hemoglobin level is 13.2 g/dL and the
leukocyte count is 50004iL. The physical
examination is within normal limits. A
peripheral blood smear shows decreased
platelets, but no other abnormalities.
What is the most appropriate management for
this patient?
A. Bone marrow aspiration and biopsy
B. Platelet aggregation studies
C. Measurement of the platelet count of her
parents and siblings
D. Prednisone, 1 mg/kg per day
E. No intervention, follow-up with monthly
platelet counts
Hematology and Oncology:Question 30
A 61-year-old man is evaluated because of
progressive fatigue, dyspnea on exertion, and
headaches over the course of the past 6
months. He also notices intense itching after
he takes a shower. He had successful right hip
replacement surgery 3 years ago for severe
degenerative arthritis. He takes a cholesterol-
lowering agent for hypercholesterolemia. He
does not smoke but drinks one or two shots of
whiskey several days a week. On physical
examination, his pulse rate is 84/min, and his
blood pressure is 145/85 mm Hg. His face
appears swollen and flushed. Examination of
the chest reveals no abnormalities. The spleen
is palpable 3 cm below the costal margin. He
has no edema. Hematocrit is 62%, leukocyte
count 17,200/μL, and platelet count
544,000/μL. Arterial blood gas results are
normal, with an oxygen saturation of 97%. His
bone marrow is hypercellular, but the
karyotype does not reveal the presence of the
Philadelphia chromosome.
What is the best next step in this patient’s
management?
A. Splenectomy
B. Phlebotomy
C. Cytarabine and idarubicin induction
chemotherapy
D. Discontinuation of alcohol intake
Hematology and Oncology:Question 31
A 29-year-old woman is evaluated because of
thrombocytopenia. She is currently in her 31st
week of an otherwise uncomplicated
pregnancy. Her platelet count was normal until
approximately week 26, after which it declined
gradually. She has had no epistaxis or other
bleeding complications and no personal or
family history of hematologic disease. One
previous pregnancy resulted in term delivery of
a healthy child. Current medications include
prenatal vitamins.
On physical examination, her blood pressure is
120/80mm Hg. Her uterus is appropriately
enlarged for gestational age.
Laboratory studies:
Hemoglobin 12.1 g/dL
Leukocyte count 8200/μL
Platelet count 93,000 /μL
A peripheral blood smear shows only
decreased platelets.
Which of the following is the most likely cause
of this patients thrombocytopenia?
A. Disseminated intravascular coagulation
B. Idiopathic (immune) thrombocytopenic
purpura
C. Incidental (gestational) thrombocytopenia
D. Preeclampsia
E. Drug-induced thrombocytopenia
Hematology and Oncology:Question 32
A 29-year-old woman with systemic lupus
erythematosus and end-stage renal disease
from glomerulonephritis is dialysis-dependent.
She has significant arthritis and polyserositis,
which are only partially helped by anti-
inflammatory therapy. She has anemia, and
has been receiving weekly erythropoietin and
intravenous iron for 2 months, without much
response.
Her physician wishes to order a transfusion to
treat her increasing fatigue. She has never
received any kind of blood product. She is
cytomegalovirus-seropositive and has never
been pregnant. There are no family members
who are HLA matched as potential donors, and
she is on the waiting list for a cadaveric kidney
transplant.
Which of the following would be prevented by
using leukocyte-reduced blood for this patient?

A. Cytomegalovirus disease
B. HLA alloimmunization
C. Transfusion graft-versus-host disease
D. Febrile nonhemolytic reaction
E. Hemolytic transfusion reaction
Hematology and Oncology:Question 33
A 55-year-old black American woman prepares
to tour South Africa and visit relatives. She
begins malaria prophylaxis with primaquine
shortly before she is scheduled to leave. Over
the next week she develops weakness and
notes dark urine. Two days later, she feels
short of breath and goes to the emergency
department.
Four years ago, she had a cardiac valve
replaced for severe aortic stenosis and takes
warfarin. Her medical history is otherwise
unremarkable. Her parents were both raised in
South Africa but have no known history of
anemia or other blood dyscrasias.
On physical examination, her temperature is
37.3 °C (99.1 °F), pulse rate is 11 5/min,
blood pressure is 155/100 mm Hg. Results of
head, eyes, ears, nose, and throat examination
are unremarkable. She has slight icterus.
Cardiac examination is unremarkable except
for mechanical valve sounds. The remainder of
her physical examination is normal.
Laboratory studies:
Hemoglobin 9.2 g/dL
Leukocyte count 1 0,400/μL
Platelet count 325,000/μL
Mean corpuscular volume 101 fL
Reticulocyte count 7.3%
Serum total bilirubin 2.0 mg/dL
Serum indirect bilirubin 1.6 mg/dL
Serum lactate dehydrogenase 667 U/L
Differential count 60% neutrophils, 15%
bands, 3% metamyelocytes, 15%
lymphocytes, 3% eosinophils, 2% nucleated
erythrocytes, 2% monocytes
Urine is negative for hemoglobin and positive
for hemosiderin. Oxygen saturation is 97%
with the patient breathing room air. Peripheral
blood smear shows Heinz bodies and bite cells.

Which of the following statements about this


patients condition is true?
A. Long-term continuation of primaquine would
lead to severe, uncompensated hemolysis.
B. The leukoerythroblastic blood smear
suggests a marrow infiltrative process.
C. The absence of free hemoglobin in the urine
excludes recent intravascular hemolysis.
D. Weekly dosing of primaquine is preferable
to low-dose daily administration.
E. The patients ethnic background indicates
she has a severe enzyme deficiency.
Hematology and Oncology:Question 34
A previously healthy 65-year-old man is
evaluated because of a 3-day history of
swelling, warmth, and erythema in the left
calf; compression ultrasound testing reveals
deep venous thrombosis extending into the
popliteal vein.
The patient is a heavy smoker. There is no
history of recent surgery, travel, or
immobilization. His mother had phlebitis in her
80s.
Rectal examination is unremarkable and a stool
specimen is negative for occult blood. Results
of a complete blood count, prothrombin time,
partial thromboplastin time, routine serum
chemistry studies, and prostate-specific
antigen are normal. No lupus anticoagulant is
detected and chest radiograph is within normal
limits.
Which of the following is the next best step in
the evaluation of this patient?
A. No further evaluation is required
B. CT scans of the chest, abdomen, and pelvis
C. CT scan of the chest; upper and lower
gastrointestinal endoscopy
D. Fasting plasma homocysteine level
E. Evaluation for a hereditary thrombotic
disorder (tests for factor V Leiden and
prothrombin G2021 0A mutations;
measurements of levels of antithrombin III,
protein C, and protein S)
Hematology and Oncology:Question 35
A 50-year-old, previous healthy man presents
with a 3-day history of low-grade fever and
confusion.
Physical examination demonstrates pallor and
ecchymoses. Laboratory studies reveal a
hemoglobin of 8.3 g/dL, leukocyte count of
9000/μL, and platelet count of 12,000/μL. The
peripheral blood smear shows schistocytes and
decreased platelets.
Which additional studies are of most use in
diagnosing this disorder?
A. Functional assessment of plasma ADAMTS
13 (von Willebrand factor-cleaving protease)
activity
B. Measurement of antibodies against ADAMTS
13
C. Measurement of serum lactate
dehydrogenase
D. Liver function studies
E. CT scan of the brain
Hematology and Oncology:Question 36
A 54-year-old man presents to the office for an
annual checkup. He feels well and exercises
regularly. He has gained 2.3 kg (5 Ib) since
last year. His only complaint is bloating and
fullness after meals. He takes
hydrochlorothiazide for hypertension and
aspirin, 81 mg/d.
On physical examination, his temperature is
36.5 °C (97.7 °F), pulse rate is 84/min, and
his blood pressure is 140/85mm Hg. The
spleen tip is palpable below the costal margin,
but there is no abdominal tenderness. He has
no lymphadenopathy, abdominal mass, or
carotid or abdominal bruit. Peripheral pulses
are palpable and normal. There is no occult
blood in the stool.
Laboratory studies:
Hemoglobin 13 g/dL
Hematocrit 40%
Mean corpuscular volume 90 fL
Leukocyte count 37,000/μL
Platelet count 470,000/μL
Differential Mostly segmented and band
neutrophils and circulating metamyelocytes
and myelocytes
What is the diagnostic test of choice?
A. CT scan of the abdomen
B. Cytogenetic analysis of peripheral blood
mononuclear cells
C. Transesophageal echocardiography
D. Leukocyte alkaline phosphatase score
E. Blood culture
Hematology and Oncology:Question 37
A 67-year-old woman has an uncomplicated
myocardial infarction. She returns to the
hospital 2 weeks later for cardiac
catheterization, which shows three-vessel
coronary artery disease. Coronary artery
bypass grafting is recommended, and the
patient undergoes an uncomplicated procedure
2 weeks later. Her platelet count immediately
before her surgery is 325,000/μL, and the day
after is 210,000/μL. She is discharged 3 days
after surgery with the platelet count not having
been repeated.
Four days later she is evaluated in the
emergency department because of shortness
of breath. She is afebrile, and radiography of
the chest shows no infiltrates. Platelet count at
this time is 85,000/μL, prothrombin time and
activated partial thromboplastin time are both
normal, and peripheral blood smear shows only
decreased numbers of platelets. A ventilation-
perfusion lung scan is indeterminate, and low-
molecular-weight heparin therapy is empirically
initiated to treat a presumed pulmonary
embolism. The following morning, the patient’s
platelet count is 25,000/μL, and the patient
has developed acute ischemia of her right foot.
What is the most likely cause of her
thrombocytopenia?
A. Idiopathic thrombocytopenic purpura
B. Seq uela of bypass surgery
C. Disseminated intravascular coagulation
D. Heparin-induced thrombocytopenia
E. Thrombotic thrombocytopenic purpura
Hematology and Oncology:Question 38
An 18-year-old black man is discovered to
have a mild microcytic hypochromic anemia
during a routine physical examination. The
patient is unaware of any significant relevant
family history. Physical examination reveals a
normal-appearing youth who is anicteric; the
spleen is not palpable.
Laboratory studies:
Hemoglobin 13 g/dL
Hematocrit 32.4%
Mean corpuscular volume 64 fL
Mean corpuscular hemoglobin 19 pg
Mean corpuscular hemoglobin concentration 30
g/dL
Reticulocyte count 4.0%
Hemoglobin electrophoresis shows the
following results.
Hemoglobin A (α2β2) 95% (normal > 97.5%)
Hemoglobin A2(α2δ2) 5% (normal <2.5%)
Hemoglobin F (α2γ2 )0% (normal 0%)
Hemoglobin S (α21βS2) 0% (normal 0%)
Iron studies are pending.
What is the most likely cause of this patient’s
anemia?
A. Iron deficiency
B. β-Thalassemia trait
C. α-Thalassemia trait
D. Glucose-6-phosphate dehydrogenase
deficiency
E. Sickle cell disease
Hematology and Oncology:Question 39
A 78-year-old woman underwent partial colonic
resection for repair of a diverticular abscess.
During her 2-week hospitalization, she
received a twice-daily injection of
unfractionated heparin. The platelet count was
21 5,000/μL on admission and remained stable
at discharge.
Two weeks after discharge she is readmitted to
the hospital with a platelet count of 27,000/μL
and a painful, dusky left lower extremity.
Hemoglobin is 12.1 g/dL and leukocyte count is
6500/μL. Arteriogram of the left lower
extremity shows a thrombus in the left
common femoral artery. Review of the
peripheral blood smear shows only decreased
numbers of platelets and no schistocytes. The
prothrombin time and partial thromboplastin
time are normal.
Which study is most likely to yield the cause of
this patient’s thrombosis?
A. Genetic assay for the factor V Leiden
mutation
B. Genetic assay for the prothrombin G2021 0A
mutation
C. 1 4C-serotonin release assay for heparin-
platelet factor 4 antibodies
D. Platelet aggregation studies
E. Platelet-associated IgG study

Hematology and Oncology:Question 40


What virus is associated with the highest risk
of transmission through blood transfusion?
A. Human T-cell lymphotropic virus
B. Hepatitis C virus
C. Human immunodeficiency virus
D. Hepatitis B virus
Hematology and Oncology:Question 41
A 52-year-old woman is evaluated because of
transient numbness on the right side of her
face and blurred vision lasting up to 24 hours.
The patient has a history of migraine
headache, but her current symptoms are
distinct from those associated with her usual
migraines. Physical examination is
unremarkable except for livedo reticularis,
which she has had for the past decade.
Neurologic examination is within normal limits.

Results of MRI of the head, transesophageal


echocardiography, and carotid ultrasound
examination are all within normal limits. Her
complete blood count and serum chemistry
studies are also normal.
The prothrombin time is within normal limits,
but the partial thromboplastin time is 55 s
(normal limit as high as 34 s). Results of a
lupus anticoagulant panel are positive. The
cardiolipin IgG is elevated at 50 GPL U/mL
(reference range <16 GPL U/mL); 1gM
antibody level is normal.
Which of the following is the best therapy for
this patient?
A. Aspirin, 325 mg/d
B. Warfarin, with a target INR of 2 to 3, and
aspirin, 81 mg/d
C. Warfarin, with a target INR of 3 to 4
D. Aspirin, 325 mg/d, and clopidogrel
E. Continue medications that she takes for her
migraine headaches
Hematology and Oncology:Question 42
A previously healthy 38-year-old woman is
evaluated because of a 2-week history of
fatigue and malaise. She has not had any
unexplained weight loss and says she does not
have fevers or night sweats. She states that
she does have occasional bilateral hand
stiffness that responds to ibuprofen.
Physical examination shows slight scleral
icterus and a mildly enlarged spleen. There is
no lymphadenopathy.
Laboratory studies:
Hemoglobin 9.2 g/dL
Leukocyte count 8200/μL
Platelet count 335,000/μL
Reticulocyte count 9.8%
A peripheral blood smear shows
polychromatophilia, occasional spherocytes,
and a rare nucleated erythrocyte.
What is the best initial therapy for this patient?

A. Prednisone
B. Intravenous immunoglobulin
C. Danazol
D. Splenectomy
E. Erythropoietin
Hematology and Oncology:Question 43
A 62-year-old black woman is evaluated after a
recent emergency department visit for a
transient ischemic attack. She has intermittent
dizziness and headaches but not vision
changes, weakness, or numbness in her
extremities. She has not seen a physician for
more than 5 years and describes herself as
generally healthy. She has not had any recent
change in weight. On physical examination, her
temperature is 36.4 °C (97.5 °F), her pulse
rate is 84/min, and her blood pressure is
135/82 mm Hg. She has no carotid bruits or
cardiac murmur, no splenomegaly, and no
edema. Stool is negative for occult blood.
Hemoglobin is 12.5 g/dL, leukocyte count
7200/1iL, platelet count 1,100,000/μL, and
fasting blood glucose 165 mg/dL. Results of
iron studies are normal. Results of
electrocardiogram and chest radiography are
normal. Her bone marrow is hypercellular;
fibrosis is not evident and karyotype is normal.
What is the most appropriate treatment of this
patients disorder?
A. Hydroxyurea and warfarin
B. Low-dose aspirin
C. Hydroxyurea and low-dose aspirin
D. Busulfan and aspirin
Hematology and Oncology:Question 44
A 55-year-old retired longshoreman is
evaluated because he has 3+ edema in both
legs. Eight months ago, he had a deep venous
thrombosis in his left iliac vein associated with
a central venous catheter. He was treated with
low-molecular-weight heparin followed by
warfarin. The warfarin was stopped after 3
months because of difficulty regulating his INR
and intermittent melena. He refused
colonoscopy at that time. Two months ago, he
had a recurrent thrombosis in the iliac vein,
and low-molecular-weight heparin has been
administered twice daily since then. Two days
ago, recurrence of gastrointestinal bleeding
was detected but has now stopped. He
required a transfusion of four units of blood.
His systolic blood pressure is now stable at 100
mm Hg. His prothrombin time is 17 s and
partial thromboplastin time is 33 s. Serum
albumin level is 1.9 g/dL. Colonoscopy reveals
an ulcerating polyp, and surgical resection is
planned.
What is the most appropriate reason to order
transfusion of fresh frozen plasma before this
patient undergoes surgery?
A. Liver disease
B. Hypovolemia
C. Hypoproteinemia
D. Prolonged prothrombin time
E. Low-molecular-weight heparin therapy
Hematology and Oncology:Question 45
A 34-year-old woman is evaluated because of
easy bruisability. She has previously been in
good health with no personal or family history
of hematologic disease. On physical
examination, occasional petechiae are evident
over the distal lower extremities. She has no
hepatosplenomegaly. Her hemoglobin is 12.8
g/dL, leukocyte count is 6200/μL, and platelet
count is 18,000/μL. Examination of the
peripheral blood smear shows decreased
platelets, with some platelets slightly increased
in size. What is the most appropriate
management for this patient?
A. Observation alone, with monthly platelet
counts
B. Intravenous immunoglobulin
C. Intravenous anti-Rh(D)
D. Prednisone followed by immediate
splenectomy
E. Prednisone
Hematology and Oncology:Question 46
A 45-year-old man develops symptomatic deep
venous thrombosis of the left leg 1 week after
arthroscopic surgery. Doppler ultrasound
examination shows a thrombus in the left
posterior tibial vein extending to within 2 mm
of the popliteal vein. The patient has mild
hypertension, for which he takes an
angiotensin-converting enzyme inhibitor. He
has no other medical problems and is a
nonsmoker. There is no family history of
thromboembolic disease.
Which of the following treatments is the most
appropriate?
A. Ibuprofen, 600 mg every 8 h, and obtain a
repeat ultrasound in 1 week
B. Warfarin, 10 mg for 2 nights; check the
patients INR the following day and adjust the
dosage if necessary
C. Enoxaparin, 1 mg/kg body weight
subcutaneously every 12 hours, and warfarin,
5 mg/d, adjusted to an lNRof 2 to 3
D. Placement of an inferior vena cava filter
E. Enoxaparin, 30mg subcutaneously every 12
hours, and warfarin, 5 mg/d, adjusted to an
INR of 2 to 3.
Hematology and Oncology:Question 47
A 43-year-old previously healthy woman is
evaluated because she develops a severe
headache and becomes confused. She has had
fatigue, lethargy, and dyspnea on exertion for
2 days.
Her temperature is 37.7 °C (99.8 °F), pulse
rate 93/min, and blood pressure 110/80 mm
Hg. Her conjunctivae and mucous membranes
are pale. Neurologic examination reveals a
confused and disoriented patient without focal
findings.
Laboratory studies:
Hemoglobin 7.3 g/dL
Hematocrit 23%
Leukocyte count 6500/μL
Platelet count 37,000/μL
Reticulocyte count 3.5% (elevated; absolute
count 107,000 cells/μL)
Serum haptoglobin < 6 mg/dL
Serum lactate dehydrogenase 2130 U/L
Serum electrolytes Normal
Serum creatinine 2.3 mg/dL
Blood urea nitrogen 43 mg/dL
Urinalysis 3+ blood, 1-3 erythrocytes, 0
leukocytes/hpf; no casts
Results of a direct antiglobulin test are
negative. The laboratory identifies
polychromasia, numerous schistocytes
(fragmented erythrocytes), and nucleated
erythrocytes on a peripheral blood smear.
Prothrombin time and partial thromboplastin
time are normal.
What is the best initial therapy for this patient?

A. Plasmapheresis with saline and albumin as


replacement fluid
B. Plasmapheresis with fresh frozen plasma as
replacement fluid
C. Platelet transfusion
D. High-dose corticosteroids
Hematology and Oncology:Question 48
A 57-year-old man is referred by his
orthopedist for recommendations regarding
anticoagulation therapy before knee
replacement surgery next month. He had an
unprovoked deep venous thrombosis 6 months
ago. He takes no medications except warfarin.
In addition to stopping the warfarin, which of
the following anticoagulation regimens is most
appropriate for this patient?
A. Administer antithrombotic prophylaxis for 1
week after surgery
B. Provide adequate antithrombotic prophylaxis
after surgery and continue anticoagulation for
a total of 6 weeks
C. Hospitalize the patient for intravenous
administration of heparin when his INR is less
than 2.0; stop the heparin before surgery and
administer anticoagulant prophylaxis for 1
week after surgery
D. Begin therapeutic doses of low-molecular-
weight heparin, and give a prophylactic dose
the morning of surgery; resume therapeutic
doses of low-molecular-weight heparin 12
hours after surgery
E. Use pneumatic compression boots during
and after surgery and start aspirin therapy,
325 mg/d postoperatively for 10 days
Hematology and Oncology:Question 49
A 69-year-old homeless man is hospitalized
because of chest pain and electrocardiographic
changes, and a diagnosis of acute myocardial
infarction is considered. He also has fever and
dyspnea, and chest radiograph shows lung
infiltrates consistent with aspiration
pneumonia.
Broad-spectrum antibiotics are administered
intravenously, but his respiratory status
worsens and he requires intubation and
ventilatory support. His hemoglobin is 9.7 g/dL
and his hematocrit is 32%. His serum albumin
is 2.2 mg/dL on admission. His left ventricular
ejection fraction is 35% by echocardiogram,
and his systolic blood pressure has ranged
from 70 to 110 mm Hg. Arterial blood gas
measurement shows a Po2 of 75 mm Hg, Pco2
of 35 mm Hg, and pH of 7.35. Oxygen
saturation is 94% while breathing 50%
inspired oxygen of 50%. His pulmonary
capillary wedge pressure is below normal.
What is the most appropriate reason to order
packed red cell transfusion for this patient?
A. Hem atocrit below 36%
B. Ventilatory support
C. Hypoxia
D. Hypotension
E. Hypovolemia
Hematology and Oncology:Question 50
A 44-year-old man is evaluated because he
has had foamy, nonbloody diarrhea for 3
weeks and has lost 4.5 kg (10 Ib). He reports
five to six bowel movements a day, including
at night. His diarrhea is associated with
crampy abdominal pain. He has a history of
chronic myeloid leukemia, for which he
underwent allogeneic stem cell transplantation
9 months ago. He is taking prednisone, 10
mg/d; loperamide, two to three times daily,
and trimethoprim-sulfamethoxazole twice a
week.
On physical examination, he has generalized
hyperpigmentation, and there are lichenoid
changes over the skin of his arms, chest, and
back. He has generalized muscle atrophy.
Laboratory results show a hematocrit of 33%,
leukocyte count of 45004iL, platelet count of
110,000/μL, and total serum bilirubin of 1.1
mg/dL. Stool culture shows no growth of
enteric pathogens.
What is the most likely cause of his diarrhea?
A. Pseudomembranous colitis
B. Laxative abuse
C. Chronic graft-versus-host disease
D. Recurrence of chronic myeloid leukemia
E. Shigella enteritis
Hematology and Oncology:Question 51
A 40-year-old woman developed deep venous
thrombosis and pulmonary embolism 1 month
post partum. She was hospitalized for 2 weeks
before the delivery because of preeclampsia;
she required a caesarean section. Warfarin
therapy is begun after delivery.
Her brother had an episode of venous
thrombosis at the age of 33 years and was
found to have protein S deficiency. Her father
was also found to have a protein S deficiency
but is in his 60s and has not had venous
thromboembolism.
After 6 months of anticoagulation, her warfarin
therapy is discontinued and coagulation studies
are done. Two weeks later, she too is found to
have protein S deficiency.
Which of the following is appropriate therapy
for this patient?
A. Resume oral anticoagulation therapy for an
indefinite period at a target INR of 2 to 3
B. Administer low-molecular-weight heparin
and resume anticoagulation with warfarin at a
fixed dose of 1 mg/d for 2 years
C. Discontinue oral anticoagulation and counsel
her about prophylactic anticoagulation in high-
risk situations
D. Initiate treatment with aspirin and
clopidogrel
Hematology and Oncology:Question 52
A 74-year-old woman with a history of
diverticulosis and hypertension presents for an
urgent office visit. She has had progressive
fatigue and poor appetite, and generally does
not feel well. She has been taking ramipril, 5
mg/d, for the past year and aspirin, 81 mg/d,
for more than 10 years.
On physical examination, she is pale. There is
no abdominal tenderness, splenomegaly, or
lymphadenopathy. She is afebrile, and her
pulse rate is 100/min.
Laboratory studies:
Hemoglobin 7.5 g/dL
Leukocyte count 2200/μL
Mean corpuscular volume 105 fL
Reticulocyte count 0.8%
Platelet count 87,000/μL
Peripheral blood smear shows ovalomacrocytes
and a few nucleated erythrocytes with nuclear
budding. Neutrophils have reduced
segmentation and granulation, and there is
pseudo-Pelger-Huet hypolobulation.
What is the most likely diagnosis?
A. Iron deficiency anemia
B. Drug-induced aplastic anemia
C. Myelodysplastic syndrome
D. Vitamin Bl2deficiency
E. Myelofibrosis
Hematology and Oncology:Question 53
A 65-year-old man is hospitalized because of
an episode of epistaxis that lasted longer than
1 hour and some rectal bleeding. He has
myelodysplastic syndrome and low blood
counts, including a platelet count of 1
5,0004/μL and an absolute neutrophil count of
600/μL (normal is greater than 2500/μL).
The patient receives a transfusion of a pool of
five platelet concentrates over 30 minutes
through a leukocyte-reduction filter. The
patient’s blood type is known to be group A,
and the blood bank releases group AB platelets
for his transfusion.
A half-hour after the transfusion, the patient
develops severe hypotension; his temperature
is 38.9 °C (102 °F), and he has rigors and
slight dyspnea. A pulse oximeter evaluation
shows 93% saturation, and no wheezing or
lymphedema is noted. Vascular collapse and
acute renal failure follow within a few hours,
despite intravenous administration of fluids
and the use of pressors.
What is the most likely cause of this adverse
transfusion reaction?
A. Bacterial contamination
B. Anaphylaxis
C. ABC incompatibility
D. Patient leukoagglutinins
E. Donor leukoagglutinins
Hematology and Oncology:Question 54
A 68-year-old woman is hospitalized for right
total hip replacement. She receives daily
injections of unfractionated heparin for
prophylaxis of venous thrombosis. On
admission her hemoglobin is 11.9 g/dL,
leukocyte count is 6500/μL, and platelet count
is 286,000/μL. On day 6 of hospitalization, the
platelet count has fallen to 105,000/μL and a
proximal right lower extremity deep venous
thrombosis is documented by venous
ultrasound. A hematology consultant
recommends initiating systemic anticoagulation
therapy with argatroban after the result of a
heparin-platelet factor 4 enzyme-linked
immunosorbent assay is positive. Argatroban
therapy is initiated at a dose of 2 μg/kg per
minute, which is titrated to a final dose of 3
μg/kg per minute to maintain a stable partial
thromboplastin time of 2 X control.
What would be the best schedule for
discontinuing this patient’s agatroban therapy
and initiating warfarin therapy?
A. Initiate warfarin immediately after
discontinuing argatroban
B. Initiate warfarin 24 h after discontinuing
argatroban
C. Initiate warfarin at 5 mg/d, and discontinue
argatroban when the INR is greater than 2.0
D. Initiate warfarin at 5 mg/d, and discontinue
argatroban when the INR is greater than 4.0
E. Initiate warfarin at a loading dose of 10
mg/d, and discontinue argatroban when the
partial thromboplastin time is greater than 4 X
control
Hematology and Oncology:Question 55
A 60-year-old woman is evaluated because her
left leg has become warm and swollen 3 weeks
after total right hip replacement. Compression
ultrasound shows proximal deep venous
thrombosis of the symptomatic leg. She was
treated postoperatively with prophylactic doses
of low-molecular-weight heparin for 1 week.
How long should the therapy be continued?
A. 6 Weeks
B. 3 Months
C. 1 Year
D. For the rest of her life
Hematology and Oncology:Question 56
After delivery of her sixth child, a 37-year-old
woman has severe vaginal bleeding from
uterine atony. She weighs 70 kg (154 Ib). She
receives 12 units of packed red cells and four
units of fresh frozen plasma over 3 hours,
along with 4 L of normal saline. Her
prothrombin time and partial thromboplastin
time are normal, and the plasma fibrinogen
level is 330 mg/dL. Her platelet count was
160,000/μL before delivery and is now
55,000/μL. The bleeding is unresponsive to
uterine massage, adminstration of
prostaglandins, and uterine curettage. An
emergency hysterectomy is planned, and she
receives a five-pack of pooled platelet
concentrates. Platelet count 10 min after
transfusion is 60,000/μL.
What is the most likely cause of the 5000/μL
platelet-count increment?
A. Active bleeding
B. Gestational thrombocytopenia
C. Disseminated intravascular coagulopathy
D. HELLP syndrome
E. Alloimmunization
Hematology and Oncology:Question 57
A 55-year-old man of Greek ancestry is
evaluated because of fatigue. He has had
anemia since childhood but has never received
a blood transfusion. He drinks wine regularly in
the evening with his meal but denies excessive
use of alcohol.
On physical examination his liver is palpable 3
cm below the right costal margin.
Laboratory studies:
Hemoglobin 10.2 g/dL
Mean corpuscular volume 70 fL
Reticulocyte count 1.5%
Serum ferritin 1800 ng/mL
Transferrin saturation 40%
Serum alanine aminotransferase 60 U/L
Serum aspartate aminotransferase 40 U/L
Serum bilirubin (total) 2.1 mg/dL with an
elevated indirect fraction
Serum haptoglobin Decreased
Serologic studies for hepatitis B and C Negative

Results of a test for the C282Y mutation in the


HFE gene are negative.
What would be the most appropriate next step
in managing this patients presumptive iron
overload?
A. Phlebotomy to remove one unit of blood
weekly until the serum ferritin level is less than
20 ng/mL
B. Diagnostic liver biopsy to include staining
for iron and determination of quantitative iron
concentration
C. Bone marrow study to include staining the
aspirate with Perls’ reagent
D. Desferrioxamine therapy, 50 mg/kg body
weight daily by subcutaneous infusion over 10
hours
Hematology and Oncology:Question 58
A 74-year-old woman has an annual checkup.
She has a history of congestive heart failure,
atrial fibrillation, hypothyroidism, diet-
controlled diabetes mellitus, and degenerative
arthritis. She reports dyspnea on exertion and
bilateral knee pain. She has lost 2.3 kg (5 Ib)
since last year. She takes ramipril, furosemide,
digoxin, thyroid hormone replacement, aspirin
(81 mg/d), and a nonsteroidal anti-
inflammatory drug.
On physical examination, her temperature is
36.5 °C (97.7 °F), her pulse rate is 72/min,
and her blood pressure is 105/70mm Hg. The
spleen tip is palpable below the costal margin.
There is no occult blood in her stool.
Laboratory studies:
Hemoglobin 11 .5 g/dL
Hematocrit 36%
Leukocyte count 55,000/μL
Platelet count 370,000/μL
Differential Mostly segmented and band
neutrophils and circulating metamyelocytes
Bone marrow aspirate smear shows a
hypercellular specimen with a very high
myeloid:erythroid ratio, a significant left shift,
and 2% blasts. Karyotypic analysis shows a
translocation between chromosomes 9 and 22
in all 20 metaphases.
What is the best next step in the treatment of
this patient?
A. Hospitalize her to start induction
chemotherapy
B. Refer her for hospice care
C. Administer interferon-alfa therapy
D. Refer her to a bone marrow transplant
center
E. Initiate therapy with imatinib mesylate
(Gleevec®)
Hematology and Oncology:Question 59
A 35-year-old woman is evaluated in her 36th
week of pregnancy because of the gradual
onset of severe right upper quadrant
abdominal pain.
On physical examination, her blood pressure is
110/70 mm Hg.
She has decreased bowel sounds and mild
rebound tenderness in the right upper
quadrant.
Laboratory studies:
Hemoglobin 9.6 g/dL
Leukocyte count 4600/μL with 80% neutrophils

Platelet count 35,000/μL


Serum aspartate aminotransferase 120 U/L
Serum total bilirubin 2.1 g/dL
Urinalysis No protein
The peripheral blood smear shows abundant
schistocytes and decreased platelets.
What is the most appropriate therapy for this
patient?
A. Administration of intravenous
immunoglobulin
B. Urgent delivery of the fetus
C. Plasma exchange
D. Aggressive diuresis
E. Administration of antiplatelet agents
Hematology and Oncology:Question 60
A 74-year-old woman presents for an urgent
office visit. She has been experiencing
epigastric discomfort over the last 3 months.
Discomfort is worse during meals and prevents
her from eating full portions. She has lost 5.4
kg (12 Ib). She also reports dyspnea on
exertion. She has been taking ramipril, aspirin,
and simvastatin.
On physical examination, her pulse rate is
88/min, and her blood pressure is 155/85 mm
Hg. There is mild epigastric tenderness with
palpation. Her spleen is massively enlarged
and palpable at the umbilicus. The edge of her
liver is also palpable 6 cm below the costal
margin.
Her hemoglobin is 7.5 g/dL, mean corpuscular
volume 89 fL, leukocyte count 11,200/μL, and
platelet count 114,000/μL.
Peripheral blood smear shows numerous
erythroblasts, myeloid precursors, and
teardrop cells. Bone marrow cannot be
aspirated, and biopsy specimen shows marked
fibrosis. Analysis of blood is negative for
t(9;22) by fluorescent in situ hybridization
(FISH).
What is the most likely diagnosis?
A. Chronic myeloid leukemia
B. Hairy cell leukemia
C. Myelodysplastic syndrome
D. Disseminated tuberculosis
E. Myelofibrosis
Hematology and Oncology:Question 61
A 50-year-old woman has factor XI deficiency
(2% activity) and a history of
menometrorrhagia and of excessive bleeding
during previous foot surgery. She is group A,
Rh-positive. Her only medication is ramipril for
hypertension.
The patient has a tooth abscess and requires
oral surgery. Her hematologist orders four
units of fresh frozen plasma to be given just
before surgery. The units are group AB, Rh-
positive.
Thirty minutes after starting infusion of the
first unit, she develops an erythematous,
pruritic rash over her arms, trunk, and face
with generalized urticaria and some
lymphedema of the oral mucosa. Wheezing
and stridor develop. Her systolic blood
pressure is 90 mm Hg, and she complains of
crampy abdominal pain and nausea. She is
afebrile and has no back pain. Results of
urinalysis are normal.
What is the most likely cause of this
transfusion reaction?
A. Bacterial contamination
B. Anaphylaxis
C. ABC incompatibility
D. Transfusion-related acute lung injury
E. Use of an angiotensin-converting enzyme
inhibitor
Hematology and Oncology:Question 62
A 40-year-old woman has an unprovoked first
episode of deep venous thrombosis. She has
no family history of thrombosis. Testing shows
that she is heterozygous for the prothrombin
G2021 0A gene mutation, but she has no other
biologic defects predisposing her to
thrombosis.
Which of the following is the most important
consideration to relay to this patient?
A. Her risk of recurrence is less than 5% per
year
B. The prothrombin gene mutation is a risk
factor for an initial episode of deep venous
thrombosis, and her first-degree relatives
should be tested for the mutation
C. The presence of the prothrombin gene
mutation clearly identifies her as being at
higher risk for recurrent deep venous
thrombosis than patients without a
prothrombotic defect
D. She should take warfarin at a target INR of
2 to 3 for the next 2 years
Hematology and Oncology:Question 63
A 44-year-old woman with idiopathic
thrombocytopenic purpura is evaluated
because of worsening thrombocytopenia. She
presented 3 years ago with menorrhagia and a
platelet count of 10,000/μL. She responded for
several months to oral corticosteroid therapy
but ultimately required more than 60 mg/d of
prednisone to maintain her platelet count
above 10,000/μL. She underwent splenectomy
18 months ago, and her platelet count initially
improved. Approximately 6 months ago,
however, her platelet count began to fall and
currently is 12,000/μL.
Her hemoglobin is 11.8 mg/dL, and leukocyte
count is 8200/μL. Prednisone, 30 mg/d, had
been reinitiated without significant
improvement in her platelet count. Her blood
type is group B and she is Rh-positive.
A trial of which of the following therapies would
NOT be appropriate for this patient?
A. Intravenous immunoglobulin
B. Cyclophosphamide
C. Rituximab
D. Intravenous anti-Rh(D)
E. Danazol

Hematology and Oncology:Question 64


A 68-year-old woman is evaluated because of
a lump on her chest wall. Seventeen years ago
she developed stage I ductal adenocarcinoma
(estrogen receptor-positive) of the left breast;
her disease was managed with lumpectomy,
breast radiation therapy, and 5 years of
tamoxifen therapy.
The lump she now has is separate from the
breast, subcutaneous in location, fixed to the
underlying 4th rib, and nontender. The mass is
excised and found to be a fibrosarcoma.
What is the relationship of this fibroscarcoma
to her original cancer and its treatment?
A. It is not related to the original cancer or its
treatment
B. It is related to the radiation therapy given to
control the breast cancer
C. It is related to the tamoxifen given to
control the breast cancer
D. It is related to the primary estrogen
receptor-positive breast cancer
Hematology and Oncology:Question 65
A 70-year-old man with an 80-pack-year
smoking history is evaluated because of a
chronic cough of 6 months duration. Chest
radiograph shows a 3-cm mass in the left
perihilar region, and bronchoscopic biopsy
confirms poorly differentiated adenocarcinoma.
There are no significant findings on physical
examination, and all blood studies are normal.
Bone scan and CT scan of the head are normal,
but CT scans of the chest and abdomen show
the mass and two 1-cm nodules in the right
lobe of the liver. Intravenous contrast
perfusion of the two nodules during the CT
scan is not suspicious for hemangioma.
Positron emission tomography scan shows
distinct uptake in the left perihilar mass but
only faint focal activity in the right lobe of the
liver. Pulmonary function tests show mild
obstructive disease.
What is the best next step in this patient’s
management?
A. Radiation therapy
B. Chemotherapy
C. CT-directed liver biopsy
D. Serum carcinoembryonic antigen test
E. Surgery

Hematology and OnCology:Question 66


A 25-year-old woman has her first routine
check-up. Her mother died of lung cancer at
the age of 60 years, and her father has had a
head and neck cancer. She is very worried
about getting cancer and wants to know what
she can do to reduce her risk, as much as
possible, of getting either of these cancers.
In addition to avoidance of tobacco, which of
the following approaches has been
demonstrated to decrease risk for one or both
of these cancers?
A. Avoidance of alcohol abuse
B. Daily intake of antioxidant vitamins
C. Avoidance of exposure to benzene
D. Daily intake of β-carotene
Hematology and Oncology:Question 67
A 66-year-old man has newly diagnosed
adenocarcinoma of the prostate gland. His
clinical stage is Tic (that is, clinically organ-
confined and detected by screening serum
prostate-specific antigen [PSA] blood test
only). His PSA is 7 ng/mL and his Gleason
score is 6.
He has decided to undergo local therapy for his
prostate cancer. He has met with a radiation
oncologist and a urologist and has decided to
undergo external-beam radiation therapy.
Which of the following side effects is he most
likely to have with radiation therapy?
A. Urinary incontinence
B. Impotence
C. Thrombosis
D. Difficulties with bowel function

Hematology and Oncology:Question 68


A 69-year old black man is evaluated because
of a history of steadily increasing upper
abdominal pain, loss of appetite, and a 4.6-kg
(1 0-Ib) weight loss over the past several
months. He recently noticed that the color of
his urine was darker than usual. The patient
had a 40-pack-year history of cigarette
smoking, but quit smoking 4 years ago. He has
hypertension that is controlled by medical
management.
Laboratory studies:
Hemoglobin 11.5 g/dL
Serum total bilirubin 3.3 g/dL
Serum albumin 3.2 g/dL
Serum aspartate aminotransferase 105 U/L
Serum alanine aminotransferase 95 U/L
Urinalysis shows elevated bilirubin. CT scan of
the abdomen shows dilation of the common
bile and pancreatic ducts, a 5.2-cm mass in
the head of the pancreas, and compression of
the superior mesenteric vein.
A fine-needle aspirate of the mass reveals
atypical cells that are suspicious for
malignancy.
What is the most appropriate course of action?
A. Refer the patient for exploratory
laparotomy, biopsy, and biliary bypass
B. Request a CT-guided biopsy of the mass in
the head of the pancreas to establish a
diagnosis
C. Obtain a consultation for endoscopic
retrograde cholangiopancreatography and
possible biopsy and biliary stent placement
D. Refer the patient to a tertiary-care center
with surgical expertise in the management of
patients with pancreatic and hepatobiliary
disease

Hematology and OnCology:Question 69


Which of the following cancer screening tests
has been shown in randomized trials to
decrease the risk of death from the target
cancer?
A. Fecal occult blood testing every 2 years for
colorectal cancer
B. Human papillomavirus DNA test for cervical
cancer every year
C. Pap smear of the cervix every year in
sexually active women for uterine cancer
D. Chest radiography every year in cigarette
smokers for lung cancer
Hematology and OnCology:Question 70
A 45-year-old woman is evaluated because of
a palpable 2-cm right axillary lymph node. The
lymph node is completely resected, and
histologic study shows adenocarcinoma;
hormone receptors are negative.
Bilateral mammography is negative, as is MRI
of the right breast. CT scans of the chest and
abdomen are negative for tumor, and no
further lymphadenopathy is detected.
Results of bone scan and MRI of the head are
also unremarkable. The patient had been
previously healthy and has never smoked.

Which of the following statements about this


patients condition is correct?
A. Breast cancer is the most likely diagnosis,
and optimal therapy for breast cancer
paradigm should be initiated.
B. Lung cancer is the most likely diagnosis,
and optimal therapy for lung cancer paradigm
should be initiated.
C. All known disease has been resected, and
the patient requires careful monitoring for
possible future recurrence.
D. Radiation therapy to the right axilla is
required with fields encompassing the right
breast.

Hematology and Oncology:Question 71


A 42-year-old woman is evaluated because she
has noticed a thickening in her left breast over
the past few weeks. She noticed it a few
months ago, but because it seems to come and
go, she put off seeking medical attention.
She is premenopausal. She had menarche at
age 13 years. When she was 35 years, she had
one child (whom she breast-fed) after a normal
first full-term pregnancy. She took oral
contraceptives for 10 years before her
pregnancy. She has no known radiation
exposure and no family history of breast
cancer.
On physical examination, there appears to be
some asymmetry in breast tissue density in the
upper outer quadrant of the left breast
compared with the right one, but no discrete
mass. There are no palpable lymph nodes.
Results of a mammogram are negative.
What would be the most appropriate next step
in this patients management?
A. Breast ultrasound; consultation with a
surgeon experienced in breast diagnosis
B. Reassurance that because her family history
is negative and the mammogram is negative,
no additional studies or treatment is required
C. MRI of the breast
D. Measurement of circulating tumor markers,
including CA15-3 and carcinoembryonic
antigen
Hematology and Oncology:Question 72
A 26-year-old man with testicular cancer who
is receiving chemotherapy is evaluated in the
emergency department. His temperature is
38.9 °C (102 °F), and he complains of feeling
flushed and tired. He has an indwelling central
venous port, but there are no localizing
symptoms to suggest a source of infection.
Chest radiograph shows no abnormalities.
The patients hemoglobin is 8.9 gIdL; the
absolute neutrophil count is 165/μL and the
platelet count is 56,000/μL. Results of other
laboratory studies and urinalysis are within
normal limits. Blood samples are obtained from
a peripheral vein and through the port and
sent for culture; a urine culture is also ordered.
The patient is hospitalized and intravenous
ceftazidime is initiated.
Three days later, the patient’s temperature is
37.8 °C (100 °F) and absolute neutrophil count
is 4504/μL; his clinical condition is otherwise
stable. Results of blood and urine cultures are
negative. A repeat chest radiograph is normal,
and blood and urine specimens are again sent
for culture.
What is the most appropriate treatment
strategy for this patient?
A. Continue the current antibiotic regimen
B. Add vancomycin
C. Switch to oral ciprofloxacin and amoxicillin-
clavulanic acid
D. Add granulocyte colony-stimulating factor
E. Remove the central venous port
Hematology and Oncology:Question 73
A 43-year-old man is evaluated because of
crushing substernal chest pain that developed
during a pickup basketball game with
colleagues at work. He was treated 15 years
ago for stage IIB massive mediastinal
Hodgkin’s disease. Treatment included
doxorubicin, bleomycin, vinbiastine, and
dacarbazine (ABVD) followed by mantle-field
radiation therapy to a total dose of 4400 cGy.
He has had no recurrence of his Hodgkin’s
disease. He takes thyroid medication because
he developed hypothyroidism 2 years after
completing his therapy for Hodgkins disease.
What is the most likely diagnosis?
A. Recurrent Hodgkins disease
B. Myocardial infarction
C. Pulmonary fibrosis
D. Anemia associated with secondary
myelodysplasia evolving to acute leukemia
E. Constrictive pericarditis

Hematology and Oncology:Question 74


Four years ago, a 67-year-old man had a
serum prostate-specific antigen (PSA) level of
16 ng/mL. Biopsy specimen showed
adenocarcinoma of the prostate gland. His
Gleason score was 7. He was treated with
external-beam radiation therapy.
One month ago, the patient noted fatigue and
rib pain. His PSA was found to be 87 ng/mL,
and bone scan revealed diffuse metastatic
disease.
What is the best treatment for this patient?
A. Chemotherapy with docetaxel
B. Chemotherapy with mitoxantrone and
prednisone
C. Androgen ablation (medical or surgical)
D. Radiation therapy with strontium-89

Hematology and Oncology:Question 75


A 68-year-old woman is evaluated because of
rectal bleeding that began recently and a sense
of fullness in the rectum. Flexible
sigmoidoscopy shows a mass 11 cm from the
anus, and biopsy reveals adenocarcinoma.
The patient is referred to a colorectal surgeon,
and a low anterior resection is performed. The
primary tumor, a 4-cm moderately
differentiated adenocarcinoma, penetrates the
bowel wall. No lymph nodes are involved.
What is the most appropriate next step in this
patient’s management?
A. No chemotherapy or radiation therapy;
annual flexible sigmoidoscopy
B. Postoperative adjuvant chemotherapy and
pelvic radiation therapy
C. Postoperative pelvic radiation therapy
D. Complete colonoscopy within the first year,
repeated every 3 to 5 years
Hematology and Oncology:Question 76
A 58-year-old postmenopausal woman has
been taking hormone replacement therapy
with combined estrogen and progestin for the
past 4 years because she was told it would
decrease her risk for heart disease. Her father
died at age 65 years of heart disease, and she
is concerned that she is also at risk. Recently,
her sister has been diagnosed with breast
cancer. Her mother died of breast cancer.
Three years ago, the patient had a breast
biopsy that showed atypical hyperplasia. She
has not had a hysterectomy.
The patient is considering chemoprevention for
breast cancer, but is nonetheless concerned
about heart disease. Her calculated risk of
breast cancer is 10.4% over the next 5 years.
Which of the following options is reasonable to
consider for this patient?
A. Continue hormone replacement therapy
alone for primary prevention of heart disease
B. Continue hormone replacement therapy and
add tamoxifen
C. Discontinue hormone replacement therapy
and start tamoxifen
D. Change her hormone replacement therapy
to estrogen alone and add tamoxifen
Hematology and Oncology:Question 77
A 68-year-old man with locally advanced non-
small-cell lung cancer is evaluated because of
the new onset of low back pain over the past 2
weeks. It is relieved with ibuprofen, and his
only other symptom is mild fatigue. The
patient completed combined chemotherapy
and radiation therapy 6 months ago, and
restaging scans afterward showed marked
shrinkage of the right perihilar mass. He has
no muscle weakness. Plain radiograph of the
thoracic and lumbar spine shows no
abnormalities other than signs of mild
osteoarthritis. Neurologic examination is
unremarkable.
What is the most appropriate next step in the
management of this patient?
A. Gallium scan
B. Re-evaluation if the symptoms get worse
C. CT scan of the chest and abdomen with
bone windows of the spine
D. MRI of the spine
E. Intravenous dexamethasone and MRI of the
spine
Hematology and Oncology:Question 78
A 72-year-old man is evaluated because of
constipation, abdominal pain, and distention
that have worsened over the past week. Two
years ago, he was diagnosed with stage III
rectal cancer (primary tumor and renal
involvement) and underwent low anterior
resection. Chemotherapy with 5-fluorouracil
and leucovorin followed, and pelvic radiation
was given with concurrent infusion of 5-
fluorouracil. He has been having regular bowel
movements, and results of his most recent
colonoscopy (1 year ago) were unremarkable.
On physical examination, his pulse rate is
100/min. He has orthostatic hypotension, a
slightly distended abdomen with hyperactive
bowel sounds, and some guarding to deep
palpation in the left lower quadrant. Plain
radiograph of the abdomen shows distended
loops of small bowel, with no stool in the distal
colon or rectum. The patient is hospitalized for
bowel rest and intravenous hydration.
What is the next step in this patient’s
management?
A. Complete colonoscopy
B. Upper gastrointestinal series with small-
bowel follow-through
C. Positron emission tomography scan of the
abdomen
D. Measurement of serum carcinoembryonic
antigen
E. CT scan of the abdomen with oral and
intravenous contrast
Hematology and Oncology:Question 79
A 60-year-old postmenopausal woman at
elevated risk for breast cancer is taking
tamoxifen to reduce her risk. She has not had
a hysterectomy.
Which of the following surveillance strategies
for the detection of endometrial cancer is most
important to incorporate into this patient’s
care?
A. Annual transvaginal ultrasound
B. Annual transabdominal pelvic ultrasound
C. Annual endometrial aspiration sampling
D. Biennial dilatation and curettage
E. Annual routine bimanual pelvic examination
Hematology and Oncology:Question 80
An 82-year-old woman who has never smoked
is evaluated because of a persistent cough.
Chest radiograph shows several lung nodules
and infiltrates. There are no significant findings
on physical examination. The patient reports
no weight loss. Blood studies show no
abnormalities. CT scans confirm pulmonary
involvement only, and results of bone scan and
CT scan of the head are normal.
The patient most likely has which of the
following histologic types of lung cancer?
A. Bronchoalveolar cell carcinoma
B. Small-cell lung cancer
C. Large-cell carcinoma
D. Squamous cell carcinoma
E. Carcinoid tumor

Hematology and OnCology:Question 81


A 45-year-old woman has recently undergone
lumpectomy and radiation therapy for a stage
II breast cancer. She received chemotherapy
for 6 months. Her periods ceased while she
was receiving chemotherapy, and she remains
amenorrheic. Her tumor was rich in estrogen
receptor, and she is taking tamoxifen. Her hair
is growing back, her energy is returning, and
she has no specific complaints, but she is
worried about recurrence.
In addition to routine follow-up, what is the
most appropriate management of this patient?
A. Positron emission tomography now and
annually
B. No further management
C. Routine tumor marker evaluation every 3 to
4 months (serum CAl 5-3, carcinoembryonic
antigen)
D. Bone scan and annual CT of the chest,
abdomen, and pelvis
E. Estrogen replacement therapy

Hematology and Oncology:Question 82


A 66-year-old black woman diagnosed with
stage III colon cancer underwent definitive
resection of the primary tumor 3 years ago.
After surgery, she received adjuvant
chemotherapy with 5-fluorouracil and
leucovorin.
Approximately 20 months later, she developed
metastatic disease in the liver (five lesions in
right and left lobe) and lungs (one lesion in the
right and left lobe). She was treated with
irinotecan, 5-fluorouracil, and leucovorin, and
had a partial response to therapy.
One month ago, disease progression was
documented on restaging CT scans. The
patient’s laboratory studies show relatively
normal organ function. She has fatigue,
decreased appetite, and has noticed a 2.3-kg
(5-Ib) weight loss over the past 3 months. She
has stopped participating in weekend bike trips
with a cycling club, but remains involved in
church and family activities and states that she
wants to be as aggressive as possible in
fighting the cancer.
What is the most reasonable recommendation
for this patient?
A. Metastatectomy
B. Hepatic arterial infusion
C. Second-line chemotherapy regimen with
leucovorin and oxaliplatin
D. High-dose chemotherapy with autologous
peripheral stem cell transplantation

Hematology and Oncology:Question 83


A 68-year-old man is evaluated because of
worsening chronic epigastric pain. He now has
fatigue and early satiety. He has iron
deficiency anemia.
Results of upper gastrointestinal endoscopy
reveal diffuse gastritis, along with mucosal
thickening in the gastric antrum associated
with a mass lesion. Abundant Helicobacter
pylon organisms are noted on biopsy, and
histologic evaluation of the mass lesion shows
it to be a gastric lymphoma of mucosa-
associated lymphoid tissue (MALT) type.
What is the most appropriate next step in the
management of this patients illness?
A. Combination chemotherapy with 5-
fluorouracil, doxorubicin, and mitomycin C
(FAM)
B. Combination chemotherapy with
cyclophosphamide, doxorubicin, vincristine,
and prednisone (CHOP)
C. Four cycles of CHOP followed by involved-
field radiation therapy
D. Eradication of Helicobacter pylon
E. Total gastrectomy followed by radiation
therapy
Hematology and Oncology:Question 84
A 65-year-old woman has a modified radical
mastectomy for a 1.0-cm, well-differentiated
breast cancer. The tumor is positive for
estrogen and progesterone receptors and
negative for HER2. Sentinel node mapping and
excision show that none of the three lymph
nodes removed is positive for metastasis. She
is otherwise healthy.
What is the best treatment for this patient at
this time?
A. Chest wall radiation therapy and tamoxifen
for 5 years
B. Chest wall radiation therapy and anastrozole
for 5 years
C. Tamoxifen for 5 years
D. Tamoxifen and anastrozole for 5 years
Hematology and Oncology:Question 85
A 59-year-old woman with an 80-pack-year
smoking history is evaluated because of weight
loss and severe pain in the upper part of her
left leg. She has lost 11.3 kg (25 Ib) from
baseline weight of 59 kg (130 Ib). A large lytic
lesion is noted on the left femur, with erosion
into the cortex. Bone scan shows multiple
lesions, and CT scan of the chest shows a large
left hilar mass and med iastinal
lymphadenopathy. Bronchoscopic biopsy
specimen shows poorly differentiated
adenocarcinoma. Radiation therapy to the left
femur is initiated.
Which of the following findings would preclude
use of palliative chemotherapy for this patient?

A. A solitary liver metastasis with normal


serum bilirubin level
B. Multiple liver metastases with a serum
bilirubin level of 3.0 mg/dL
C. A cytologically positive pleural effusion
D. Hypercalcemia
E. Poor performance status
Hematology and Oncology:Question 86
A 57-year-old man has tried to stop smoking
unsuccessfully for the past year. He has tried
smoking cessation counseling and use of
nicotine gum. Although he has no new
symptoms, the ‘smokers cough” that he has
had for years is a constant reminder that he is
at risk of dying of lung cancer. He wants to
know what measures he can take that have
been shown to reduce that risk.
In addition to a smoking cessation program,
what is the best recommendation for this
patient?
A. Daily dietary supplementation with the
antioxidantl3-carotene
B. Daily isotretinoin, titrated to limit skin
toxicity
C. Daily bupropion
D. An annual low-dose spiral CT scan of the
chest
E. Sputum cytology every4 months
Hematology and Oncology:Question 87
A 71-year-old man with mild chronic
obstructive pulmonary disease and mild
hypertension is evaluated during an annual
routine visit. Review of systems is notable for
intermittent cough, increasing dyspnea on
exertion, a 2.3-kg (5 Ib) weight loss, and
fatigue. The patient takes aspirin, 81 mg/d,
and hydrochlorothiazide, 50 mg/d orally. He
smoked one pack of cigarettes per day for 49
years but quit smoking 2 years ago. On
physical examination, distant breath sounds
are audible in both lungs and there are
scattered rhonchi. Chest radiograph shows a
perihilar mass. Abnormal laboratory results
include hemoglobin of 12.5 g/dL and a serum
sodium of 127 meq/L. Endobronchial biopsy
reveals small-cell lung cancer. Further staging
studies suggest that the disease is limited-
stage.
What is the most appropriate treatment for this
patient’s hyponatremia?
A. Fluid restriction to 1 L/d
B. Fluid restriction to 1 L/d and demeclocycline
therapy
C. Discontinuation of hydrochiorothiazide
D. Combination chemotherapy for the small-
cell lung cancer

Hematology and Oncology:Question 88


A 23-year-old man is evaluated because of a
painless right-sided scrotal mass.
Ciprofloxacin, 500 mg every 12 hours, is
administered for 10 days, but he notes little
improvement in the swelling. His serum α-
fetoprotein level is elevated at 100 ng/mL, and
his 13-human chorionic gonadotropin level is
64 m/UImL. Testicular ultrasound examination
reveals a hypoechoic mass.
Which of the following would be the most
appropriate next step in his treatment?
A. Retroperitoneal lymph node dissection
B. Combination chemotherapy with bleomycin,
etoposide, and cisplatin
C. Radiation to the pelvis
D. Inguinal orchiectomy
Hematology and Oncology:Question 89
A 40-year-old woman has a routine
gynecologic examination. At the age of 32
years, she gave birth to twins after receiving
fertility drugs. She has just seen a television
program on ovarian cancer, and is concerned
because previous use of fertility drugs and a
family history of ovarian cancer were
mentioned as possible risk factors. Her mother
developed endometrial cancer at 56 years, and
a paternal uncle was diagnosed with a type of
lymphoma at 60 years. She has one older
sister with hypertension and one younger
brother who is healthy. Results of her pelvic
examination are unremarkable, but she asks
what additional tests she should have to look
for ovarian cancer in the early stages that
cannot be detected by physical examination.
What is the most appropriate management for
this patient?
A. Blood test for CA-125
B. Rectovaginal pelvic examination in 1 year
C. Transvaginal ultrasound
D. Screening test for BRCA-1 and BRCA-2
Hematology and Oncology:Question 90
A 44-year-old woman has a lumpectomy and
radiation therapy for stage II breast cancer.
She receives 6 months of chemotherapy. Her
periods cease while she is receiving
chemotherapy, and she remains
amenorrheic. Her tumor was found to be rich
in estrogen receptor, and she is taking
tamoxifen.
Her hair is growing back and her energy is
returning, but she is having severe hot flushes
that keep her awake at night. She is so tired
during the day that she is unable to perform
her job in a satisfactory manner. She also
relates that she and her husband have been
unable to have satisfying sexual relations
because intercourse is painful for her.
What would be the most appropriate treatment
for this patient?
A. A selective serotonin reuptake inhibitor and
use of nonhormonal vaginal lubricating
preparations
B. Reassurance that these normal physiologic
responses to menopause will resolve in time
C. Estrogen replacement therapy with a
progesterone supplement
D. Work-up for endometrial cancer because
she is taking tamoxifen
Hematology and Oncology:Question 91
A 64-year-old white man presents for serum
prostate-specific antigen (PSA) screening at his
wife’s urging. He is in generally good health,
except for mild hypertension. His digital rectal
examination reveals a mildly enlarged prostate
gland with no discrete nodules. He has no
family history of prostate cancer.
What is the best advice to give him?
A. There is no evidence that PSA screening
leads to the earlier detection of prostate cancer

B. There is insufficient evidence to establish


whether PSA screening affects overall mortality
rates
C. On the basis of his lack of symptoms it is
unlikely that he has an elevated PSA
D. Because a digital rectal examination is more
sensitive than a serum PSA test, his normal
examination makes it unlikely that he has
cancer
Hematology and Oncology:Question 92
A 58-year-old woman with a 60-pack-year
smoking history is evaluated because of
hemoptysis and weight loss. Chest radiograph
and CT scan show a right perihilar mass with
mediastinal adenopathy. The results of CT
scans of the abdomen, bone scan, and MRI of
the head are otherwise negative. Examination
of a specimen by bronchoscopic biopsy
confirms small-cell lung cancer, and the
findings suggest limited-stage disease.
After the patient completes mediastinal
radiation therapy and four cycles of cisplatin
and etoposide, repeat CT scans indicate that
her disease is in complete remission.
What is the best recommendation concerning
further therapy?
A. Prophylactic cranial irradiation
B. Four more cycles of chemotherapy,
including a taxane
C. Resection of the localized disease
D. No further therapy; follow-up only
Hematology and Oncology:Question 93
A 63-year-old man is evaluated because of a
several-month history of increasing fatigue and
some vague upper abdominal discomfort. His
medical history includes gastroesophageal
reflux disease, coronary artery disease, and
clinical depression, all of which are well
controlled with medications. The patient has
been working full time.
On physical examination, he has mild
hepatomegaly. His hemoglobin is 12.2 g/dL,
serum alkaline phosphatase level 280 U/L, and
serum aspartate aminotransferase level 65
U/L. CT scan of the abdomen and pelvis shows
multiple hepatic lesions ranging in size from 1
cm to 4 cm; mesenteric, para-aortic, and
paracaval lymphadenopathy, and a colonic
mass at the splenic flexure. Colonoscopy
reveals a nonobstructing, non bleeding lesion;
biopsy shows it to be poorly differentiated
adenocarcinoma. Fine-needle aspiration of one
of the liver lesions confirms the presence of
malignant cells consistent with a primary colon
cancer.
What is the most appropriate next step in the
management of this patient?
A. Resection of the primary tumor followed by
systemic chemotherapy
B. Combined regional chemotherapy to the
liver and systemic chemotherapy
C. Exploratory laparotomy with resection of the
primary tumor and placement of a hepatic
arterial infusion pump
D. Systemic chemotherapy
E. Best supportive care
Hematology and Oncology:Question 94
A 69-year-old man underwent a radical
prostatectomy ii years ago. His serum
prostate-specific antigen (PSA) level was 7.1
ng/mL, clinical stage was Tic (clinically organ-
confined and detected by screening PSA blood
test only), and his Gleason score was 5.
His PSA level became detectable 4 years ago.
Over the past 3 years, his PSA values have
been 1.4 ng/mL, 1 .8 ng/mL, and 2.2 ng/mL.
He remains asymptomatic, and a recent bone
scan revealed only degenerative disease.
The patients risk for rapid metastasis is low for
which of the following reasons?
A. He was initially treated by radical
prostatectomy.
B. His PSA level became undetectable.
C. He has had a slow, prolonged rise in his
PSA.
D. He is almost 70 years old.
Hematology and Oncology:Question 95
A 61-year-old man with a 120-pack-year
smoking history is evaluated for a persistent
and worsening cough and found to have a 3-
cm mass in the right upper lobe on chest
radiography. CT-directed needle biopsy is
positive for squamous cell carcinoma. CT scans
of the abdomen and pelvis, bone scan, and
MRI of the head are negative for metastatic
disease. Positron emission tomography scan
shows uptake only in the right upper lobe
mass, and pulmonary function tests indicate
that the patient has adequate pulmonary
reserve to undergo resection.
The patient undergoes a right upper
lobectomy. All margins are clear, and all
peribronchial lymph nodes are negative for
tumor within the resected specimen.
What is the most reasonable adjuvant therapy
for this patient?
A. Four cycles of combination chemotherapy
including a taxane
B. Four cycles of combination chemotherapy
including a taxane, followed by radiation
therapy to the draining lymph node sites in the
mediastinum
C. Radiation therapy to the draining lymph
node sites in the mediastinum
D. Six cycles of combination chemotherapy
including a platinum agent
E. No radiation therapy or chemotherapy is
required at this time
Hematology and Oncology:Question 96
A 47-year-old woman is recovering from
surgery for stage III ovarian cancer (spread to
the peritoneal cavity but without parenchymal
liver involvement). Not all of the visible cancer
could be resected; the largest residual tumor
was 3 cm in diameter. Her performance status
is excellent. She is interested in pursuing
aggressive therapy and wishes to be treated as
soon as possible in her local community. The
proposed systemic therapy will involve cisplatin
or carboplatin and paclitaxel.
What further treatment is appropriate for this
patient?
A. No further treatment beyond the proposed
standard therapy
B. Another drug that has a different
mechanism of action
C. Regional peritoneal therapy
D. High-dose therapy with peripheral or bone
marrow stem cell rescue
E. A second-look laparotomy

Hematology and Oncology:Question 97


A 62-year-old postmenopausal woman with a
family history of breast cancer in two first-
degree relatives wishes to consider taking
tamoxifen to reduce her risk of breast cancer.
Her baseline risk of breast cancer qualifies her
for consideration of tamoxifen (i.e., an
absolute risk of at least 1.66% over the next 5
years). She has had a prior hysterectomy, but
her ovaries are intact.
Which of the following effects of tamoxifen are
relevant to the decision?
A. Decreased risk of myocardial infarction or
ischemic heart disease
B. Increased risk of deep venous thrombosis
C. Increased risk of ovarian cancer
D. Increased risk of major depression
E. Increased risk of colorectal cancer

Hematology and Oncology:Question 98


A 50-year-old perimenopausal woman is
evaluated because of abdominal swelling. CT
scan of the chest, abdomen, and pelvis shows
mesenteric lymphadenopathy and a small
amount of ascites but no ovarian masses.
Needle biopsy specimen shows
adenocarcinoma; the tumor is found to be
negative for hormone receptors. The serum
carcinoembryonic antigen level is normal at 2.0
ng/mL and the serum CA-125 is 1200 ng/mL
(elevated).
What is the best next step in her treatment?
A. Combination chemotherapy
B. Debulking surgery
C. Radiation therapy to a wide port
D. Positron emission tomography scan to
assess other sites of disease
E. Bone scan to rule out skeletal metastases

Hematology and Oncology:Question 99


A 50-year-old menstruating woman has a 1 .5-
cm moderately differentiated breast cancer.
The lesion is completely excised, and the
margins of the excision are negative. Axillary
node sampling shows that she has three
positive nodes. The tumor is negative for
estrogen and progesterone receptors and is
highly positive for HER2. She is otherwise
healthy.
Her mother had breast cancer at 62 years and
was treated by mastectomy; she is alive and
healthy at the age of 80 years. The patients
sister had breast cancer at 54 years and was
treated by breast conservation therapy; 4
years later she died of a recurrence of breast
cancer.
The patient has seen two different surgeons
with opposing viewpoints regarding the best
treatment, and she has been reading
extensively on the Internet and has become
confused about her options.
Which of the following represents the best
treatment for this patient?
A. Modified radical mastectomy, followed by
tamoxifen and chemotherapy
B. Chemotherapy with no further treatment to
the breast
C. Chemotherapy and tamoxifen, with no
further treatment to the breast
D. Chemotherapy and radiation therapy to the
breast and axillary regions
E. Chemotherapy, tamoxifen, and radiation
therapy to the breast and axillary regions

Hematology and Oncology:Question 100


A 32-year-old woman is evaluated because of
a painless swelling in her lower neck. She
noted the swelling about 3 weeks ago and now
thinks it is getting larger. She has no history of
foreign travel. She smokes but does not drink
alcohol. She owns a cat. She has not had fever
or unexplained weight loss, but has had night
sweats twice in the past week.
She has been feeling fatigued lately and has
noted dyspnea on exertion. On physical
examination, a 3 X 5-cm left supraclavicular
lymph node is palpable. It is firm but not rock-
hard and is nontender. The remainder of her
physical examination and results of a complete
blood count are normal.
What is the best next step in this patient’s
management?
A. Observe for 2 weeks
B. Refer her to an otolaryngologist for a
thorough search for a primary lesion of the
oropharyngeal mucosa
C. Administer oral antibiotics and observe the
course of the node
D. Perform a needle aspiration of the node
E. Perform a chest radiograph
Hematology and Oncology:Question 101
A 64-year-old man is evaluated because of
fatigue and shortness of breath on exertion. He
has had three episodes of urinary tract
infection in the past 7 months. On physical
examination, he has pallor but is otherwise
normal. He denies blood loss, and his stool is
negative for occult blood on three
measurements.
He is anemic (hemoglobin 8.4 g/dL) with
normochromic, normocytic indices, and his
serum creatinine level is 2.9 mg/dL. The total
serum protein level is elevated, and the serum
albumin level is low normal.
His serum calcium level is 11.8 mg/dL.
What is the most likely cause of his symptoms?

A. Multiple myeloma
B. Chronic pyelonephritis
C. Iron deficiency anemia
D. Colon cancer
E. Small-cell lung cancer

Hematology and Oncology:Question 102


A 63-year-old man is evaluated because of
new-onset hemoptysis and a 100-pack-year
smoking history. Chest radiograph shows a 4-
cm right perihilar mass, and bronchoscopic
biopsy and cytologic evaluation of sputum
confirm squamous cell carcinoma.
There are no significant findings on physical
examination, and the patient reports no weight
loss.
Complete blood count is normal except for a
serum alkaline phosphatase level at 110 U/L.
CT scans of the chest and abdomen show no
mediastinal adenopathy. Results of a positron
emission tomography scan are negative,
except for the right perihilar mass.

In which of the following situations would


surgical therapy be offered?
A. A solitary bone metastasis is noted on bone
scan, and a lytic lesion is confirmed
radiographically.
B. A solitary liver metastasis is noted on CT
scan and confirmed by needle biopsy and
positron emission tomography scan.
C. A solitary brain metastasis is noted on CT
scan of the head.
D. A small effusion is noted in the right
hemithorax, and cytopathologic examination of
the pleural fluid is positive for tumor.

Hematology and Oncology:Question 103


A 40-year-old woman was treated for breast
cancer 5 years ago by lumpectomy, breast
irradiation, and 3 months of chemotherapy.
Her original cancer was negative for estrogen
and progesterone receptors
and unequivocally positive for HER2. Her
periods stopped during chemotherapy but
resumed 4 months later, and she remains
premenopausal now.
She is evaluated now because of pain in her
back, a nagging cough, and fatigue. Physical
examination shows palpable skin nodules over
the affected breast, axillary adenopathy, and
dullness to percussion, and decreased breath
sounds in the base of the left lung; her liver
edge is palpable below the right costochondral
border.
Liver function values are approximately twice
normal; serum bilirubin is normal. Radiograph
of the chest shows multiple pulmonary nodules
and a left pleural effusion confirmed by CT,
which also shows the liver nodules. Bone scan
and MRI are consistent with bone metastases.
Biopsy of one of the skin lesions is consistent
with metastatic breast cancer; it is negative for
estrogen and progesterone receptors, and
positive for HER2. What is the most
appropriate treatment for this patient?
A. Hospice care and comfort measures
B. Combination endocrine therapy with
tamoxifen and an aromatase inhibitor
C. High-dose chemotherapy with bone marrow
stem cell support
D. Trastuzumab and taxane-based
chemotherapy
E. Combination endocrine therapy with ovarian
ablation and an aromatase inhibitor
Hematology and Oncology:Question 104
A 65-year-old man has a routine screening
examination, and his serum prostate-specific
antigen (PSA) level is found to be 6.7 ng/mL.
Digital rectal examination reveals a mildly
enlarged prostate gland, but no discrete
nodules. One year ago, his PSA was 2.1
ng/mL. The patient is referred to a urologist
who performs a biopsy of the gland. Biopsy
specimen shows adenocarcinoma of the
prostate with a Gleason score of 9.
Which of the following best describes his
prognosis on the basis of his Gleason score?
A. The Gleason score is not reproducible and
should not be used as a prognostic variable.
B. Because the patient is asymptomatic, the
Gleason score will not be predictive of
outcome.
C. The Gleason score indicates that the patient
would have a high risk of recurrence after a
radical prostatectomy.
D. The Gleason score indicates that the patient
would have a very low risk of recurrence after
radiation therapy.

Hematology and OnCology:Question 105


A 78-year-old man is evaluated because of
shortness of breath of acute onset. The patient
reports a 13.6-kg (30-Ib) weight loss, vague
upper abdominal discomfort, nausea, loss of
appetite, and fatigue. He has hypertension and
symptomatic atherosclerotic coronary artery
disease despite optimal medical management.
Even before the recent episode of dyspnea, his
physical activity has been limited; he spends
most of the day resting either in bed or in a
chair.
Laboratory studies:
Hemoglobin 10.5 g/dL
Serum albumin 2.6 g/dL
Serum alanine aminotransferase 65 U/L
Serum aspartate aminotransferase 78 U/L
CA 19-94500 U/L
Arterial oxygen saturation 85% by pulse
oximetry
Spiral CT scan of the chest shows a pulmonary
embolism.
The patient is hospitalized for anticoagulation
and oxygen therapy and gradually improves.
Subsequent CT of the abdomen shows a 4-cm
mass in the tail of the pancreas and numerous
low-attenuation lesions in the liver. CT-guided
biopsy of the hepatic lesions and the
pancreatic mass is deferred during this
hospitalization.
What is the most appropriate next step in the
management of this patient?
A. Best supportive care and referral to a
hospice
B. Diagnostic CT-guided biopsy under a
heparin window” when the patients clinical
status improves
C. Neurolytic celiac axis block
D. Palliative chemotherapy
E. Palliative radiation therapy
Hematology and Oncology:Question 106
A 76-year-old man was diagnosed with
prostate cancer 8 years ago. At that time he
had a serum prostate-specific antigen (PSA)
level of 12 ng/mL, a Gleason score of 6, and a
clinical stage of T2b (organ-confined on digital
rectal examination but involving both lobes of
the prostate gland). He was treated with
external-beam radiation therapy, and his PSA
level dropped to a low of 1.4 ng/mL.
Four years later, his PSA level began to rise
and now is 14.1 ng/mL. A recent bone scan
and CT scans revealed no metastatic disease.
His urologist has suggested initiating a course
of leuprolide. Which of the following is a
potential side effect of leuprolide that the
patient should be informed about before
commencing therapy?
A. Impaired urinary flow
B. Bone thinning
C. Weight loss
D. Increased hair growth
Hematology and Oncology:Question 107
A 68-year-old man with a 60-pack-year
smoking history is evaluated because of
hemoptysis. Radiograph of the chest shows a
right hilar mass and mediastinal widening. CT
scans of the chest and abdomen confirm the
hilar mass and bulky lymphadenopathy in the
mediastinum with no other overt metastases.
Bronchoscopic biopsy specimen shows small-
cell lung cancer. Bone scan and CT scan of the
head are negative for tumor. A low serum
sodium level and inappropriately high urine
osmolality suggest that he has the syndrome
of inappropriate antidiuretic hormone
secretion. Which of the following is the most
appropriate treatment recommendation?
A. Surgery followed by chemotherapy
B. Radiation therapy followed by chemotherapy
C. Chemotherapy alone
D. Radiation therapy and concomitant
chemotherapy
E. Chemotherapy followed by radiation therapy

Hematology and Oncology:Question 108


A 78-year-old woman with metastatic breast
cancer involving the bones and soft tissues
who has been taking hormone replacement
therapy is evaluated in the emergency
department because of lethargy and weakness,
nausea, thirst, and dizziness. She is orthostatic
and clinically dehydrated.
She has a history of congestive heart failure
that has been controlled with medications.
Laboratory studies:
Blood urea nitrogen 42 mg/dL
Total serum calcium 11 .4 mg/dL
Serum creatinine 1.6 mg/dL
Serum albumin 3.0 g/dL
What is the most appropriate initial treatment?

A. Slow rehydration with half-normal saline


B. Intravenous administration of a
bisphosphonate
C. Vigorous rehydration with normal saline
D. Intravenous administration of furosemide
along with saline rehydration
E. Intravenous administration of corticosteroids

Hematology and Oncology:Question 109


A 52-year-old woman who is a nonsmoker has
a 3-cm right inguinal lymph node. Biopsy
specimen shows a poorly differentiated
malignancy that is difficult to characterize by
light microscopy.
The biopsy specimen is negative for leukocyte
common antigen, cytokeratin, and estrogen
receptors.
Which of the following additional tests would
best establish the source of this tumor?
A. Bone scan
B. Measurement of serum carcinoembryonic
antigen
C. Stain for S-100
D. Measurement of serum CA 19-9

Hematology and Oncology:Question 110


A 59-year-old man who is a heavy smoker
presents with cough, shortness of breath, and
a sensation of head fullness. Physical
examination is notable for cervical venous
distention and facial edema. A chest
radiograph shows a widened superior
mediastinum.
What is the most appropriate initial
intervention for this patient?
A. Initiation of radiation therapy while a
diagnostic work-up is pursued
B. CT of the chest and consultation with a
pulmonologist and a thoracic surgeon
C. CT of the chest and a venous dye study to
rule out thrombosis of the superior vena cava
D. Mediastinoscopy with biopsy
E. Ventilation/perfusion scan
Hematology and Oncology:Question 111
A 29-year-old man was diagnosed with stage II
(that is, spread from the primary tumor to
retroperitoneal lymph nodes)
nonseminomatous germ cell tumor 1 year ago.
Treatment included orchiectomy and
retroperitoneal lymph node dissection.
He now presents with an elevated level of β-
human chorionic gonadotropin hormone and
multiple pulmonary lesions ranging in size from
0.61 to 3.22 cm.
Needle biopsy specimen of the largest lesion
reveals a germ cell tumor consistent with the
initial diagnosis.
He is asymptomatic.
What is the best treatment for this patient?
A. Surgical resection of the lung lesions
B. Combination chemotherapy with bleomycin,
etoposide, and cisplatin
C. Radiation therapy to the chest
D. Watchful waiting
Hematology and Oncology:Question 112
A 64-year-old woman has the following family
history: One sister, who was a smoker, had
lung cancer at the age of 63 years; another
sister had breast cancer at the age of 70 years.
Her father had prostate cancer at 82 years of
age, and her mother had breast cancer at age
71 years. She has three other sisters, now
aged 58, 60, and 65 years who have no history
of cancer. She has two daughters and two sons
between the ages of 25 and 39 years, who
have no history of cancer. Her ancestry is
European in general, and she is not of known
Ashkenazi Jewish descent. She is concerned
that “cancer runs in the family” and would like
to have one of those “gene tests” she has read
about.
What is the best advice to give her and her
family?
A. Have blood testing for abnormalities in
known germ-line tumor suppressor genes
(such as BRCA-1 and BRCA-2).
B. Encourage her children to have genetic
counseling and testing for abnormalities in
known germ-line tumor suppressor genes.
C. Advise her children to start routine cancer
screening tests immediately, including
mammography, colonoscopy or sigmoidoscopy,
and prostate-specific antigen testing.
D. Advise her that genetic testing is
unnecessary, and recommend that she and her
family continue screening and risk-reduction
strategies as recommended for the general
population.
E. Recommend bilateral prophylactic
mastectomies and oophorectomies for her and
her daughters.
Hematology and Oncology:Question 113
A 72-year-old man is evaluated because of
back pain and fatigue. Laboratory values
indicate he is anemic, with normal leukocyte
count and platelet count. He has mild
hypercalcemia and a normal serum creatinine
level. His gamma globulins are elevated, and
serum protein electrophoresis shows the
presence of a monoclonal protein, an IgG-K
light chain containing immunoglobulin at 4.4
g/dL. His bone marrow contains 20% plasma
cells. A skeletal survey reveals multiple
osteolytic lesions in the spine, ribs, and skull.
Therapy with melphalan and prednisone is
initiated.
Which of the following agents would NOT be
routinely used in his management?
A. Erythropoietin
B. Radiation therapy
C. Intravenous gamma globulin
D. Adequate analgesia (including narcotics, if
necessary)
E. A bisphosphonate

ANSWERS
Hematology and Oncology:Question 1
The correct answer is D
Educational Objectives
Diagnose thrombocytopenia in a critically ill
patient.
Critique
The acute onset of a febrile illness
accompanied by leukocytosis, hypotension,
and thrombocytopenia is consistent with
sepsis, although the cause may not be
immediately evident. Routine studies not
mentioned should involve blood and urine
cultures, and chest radiography with sputum
examination and culture if indicated. As
always, the peripheral blood smear should be
examined closely in any patient with
hematologic abnormalities.
The levels of fibrin D-dimer and total fibrinogen
should be assessed in this patient, in
conjunction with routine coagulation studies.
Increased levels of D-dimer and low-normal or
decreased levels of fibrinogen would be
consistent with disseminated intravascular
coagulation.
Since the patient was known to be in
previously good health without underlying
hematologic disease, it is likely that the
thrombocytopenia is acute and results from
accelerated platelet clearance. Therefore, a
bone marrow aspiration and biopsy are not
likely to be helpful. Measuring factor VIII will
not provide useful information in this case—the
level may be normal, decreased due to
consumption, or increased due to acute
inflammation. Platelet-associated IgG levels
are elevated in many disorders, including
sepsis, and therefore will not provide useful
information concerning the likelihood of a
primary immune-mediated thrombocytopenia.
The bleeding time has poor predictive value for
bleeding and is usually prolonged in
thrombocytopenic patients.
Diagnosis of disseminated intravascular
coagulation would provide a likely explanation
for the patients thrombocytopenia, although
sepsis alone may lead to decreased platelet
counts. Moreover, the coagulation status in a
thrombocytopenic patient should be assessed,
and factor replacement initiated if indicated.
Hematology and Oncology:Question 2
The correct answer is E
Educational Objectives
Treat a first unprovoked episode of pulmonary
emboli provoked by a transient risk factor.
Critique
The duration of the anticoagulation therapy
this patient has received is appropriate (3 to 6
months) for a first episode of venous
thromboembolism. An episode of pulmonary
embolism that is associated with a transient
risk factor (such as oral contraceptive use)
carries a substantially lower annual risk of
recurrence than one that is “unprovoked.’
Although heterozygosity for the prothrombin
G2021 0A mutation is a risk factor for a first
episode of unprovoked embolism (and
embolism in association with oral contraceptive
use), its presence does not confer an increased
risk of recurrence. She should not resume
taking oral contraceptives and be made aware
of the symptoms of recurrent
thromboembolism.
The recurrence-prevention benefits of
anticoagulation at an INR of 2 to 3 for more
than 3 to 6 months have not been
demonstrated to outweigh the bleeding risk
associated with extended treatment. Although
low-intensity warfarin (target INR 1 .5 to 2)
was recently shown to protect against
recurrent venous thrombosis without a
significant increase in bleeding risk, this
patient’s relatively low annual recurrence risk
in the absence of provocative risk factors
would argue against the continued use of even
low-intensity warfarin. The extent to which oral
anticoagulation at a target INR of 1.5 to 2
provides antithrombotic protection to women
with a history of pulmonary embolism and
continued use of oral contraceptives is
unknown; it is not a preferred recommendation
for this patient.
Aspirin or a combination of aspirin and
dipyridamole has not been shown to decrease
the risk of recurrent venous thromboembolism,
although aspirin is effective in the primary and
secondary prevention of myocardial infarction.
Hematology and Oncology:Question 3
The correct answer is B
Educational Objectives
Recognize therapeutic options in patients with
sickle cell disease.
Critique
Hydroxyurea therapy reduces the incidence of
sickle-cell-related events through its capacity
to re-activate expression of the
developmentally silenced fetal gamma globin.
The potential toxicities of hydroxyurea are well
described. The patients levels can be easily
monitored through periodic complete blood
counts. The patient’s current transfusion-
related decrease in hemoglobin S should not
affect the decision to initiate therapy, because
the benefits of hydroxyurea are generally
delayed.
Although bone marrow transplantation might
be a consideration for this patient, it is
attended by substantially higher toxicity than
hydroxyurea therapy. Gene therapy
approaches for the treatment of sickle cell
disease remain investigational. There is no role
for either long-term transfusion therapy or
oxygen therapy for this patient.
Hematology and Oncology:Question 4
The correct answer is B
Educational Objectives
Diagnose and treat drug-induced neutropenia.
Critique
The patient should be treated with granulocyte
colony-stimulating factor. Two types of
neutropenia are reported with clozapine. One is
mild-to-moderate neutropenia (absolute
neutrophil count, 500 to 1500 μ/L) that occurs
in 1.5% to 2.0% of patients receiving
clozapine. When clozapine is discontinued,
recovery occurs within 3 to 7 days, and
patients remain asymptomatic. The second
type of neutropenia is more severe. It consists
of agranulocytosis (absolute neutrophil count
less than 500/μL) and has an incidence of
0.8% in patients treated for 1 year.
Most cases of agranulocytosis occur within 3
months after the start of treatment. All
affected patients are at risk for neutropenic
sepsis. There is usually a complete, selective
cessation in the bone marrow of neutrophil-
precursor production beyond the myeloblast
stage. Recovery requires 14 to 22 days; G-CSF
therapy has been shown to shorten this
duration. Given the patient’s history of
clozapine use and the absence of anemia and
thrombocytopenia, it is not necessary to
perform bone marrow aspiration and biopsy
unless her neutropenia fails to improve over
the next 2 weeks.
Granulocyte transfusions are reserved for
patients with uncontrolled infection despite
antibiotic therapy and persistent severe
neutropenia. Drug-induced neutropenia
commonly resolves after discontinuation of the
drug, and bone marrow transplantation is not
warranted. This patient reported no HIV risk
factors, and her presentation does not suggest
HIV infection.

Hematology and Oncology:Question 5


The correct answer is A
Educational Objectives
Understand the differential diagnosis of anemia
in severe hepatic failure.
Critique
The patient has spur cell anemia. In a small
percentage of patients with alcoholic cirrhosis,
erythrocytes accumulate large amounts of free
cholesterol with significant erythrocyte
membrane distortion. A progressive severe
hemolytic anemia ensues, with cells that partly
resemble acanthocytes.
Unlike true acanthocytes, the cells in spur cell
anemia are small and spheroidal with short,
spiny projections (hence the term spur cell).
Increased amounts of membrane cholesterol
lead to increased membrane surface area.
Membrane distortion forms the basis of the
morphologic presentation secondary to
abetalipoproteinemia. The spur cells are
trapped and destroyed in the splenic cords
leading to progressive splenic enlargement.
The prognosis of patients with spur cell anemia
is extremely poor. Splenectomy is occasionally
effective in moderating the hemolysis, but
patients usually have underlying portal
hypertension and coagulopathies, which
complicate the procedure.
More often, patients with alcoholic liver disease
may have chronic, mild to moderate anemia
that is multifactorial (liver disease,
hypersplenism, alcohol as a direct toxin to
erythroid progenitor cells). Liver disease,
particularly extrahepatic biliary obstruction,
may be associated with membrane changes
such as target cell formation. With an increase
in cholesterol, the cholesterol : phospholipid
ratio changes, and the cell membrane surface
area increases. Thus, the cells become broad
and flat. As the cells dry, hemoglobin collects
in the thicker areas of the cells (the center and
edges) producing a “target’ appearance.
Folic acid deficiency should be considered in
any patient with alcoholism who has a
macrocytic anemia. In this patient, however,
the high reticulocyte count makes this
diagnosis unlikely. The patient probably has
preexisting hypersplenism, based on her
diagnosis of cirrhosis, but it would not account
for her severe anemia and elevated
reticulocyte count. Patients who have chronic
alcoholism should be maintained
prophylactically on folic acid supplementation.
Hematology and Oncology:Question 6
The correct answer is D
Educational Objectives
Treat symptomatic anemia of chronic disease.
Critique
The patient has anemia of chronic disease, for
which erythropoietin (usually given
concurrently with iron) is an effective therapy.
His serum iron and total iron-binding capacity
are low, but his reticuloendothelial iron stores
are adequate, as demonstrated by his elevated
serum ferritin level. The clinical picture is
characteristic of anemia of chronic disease,
which often includes mild microcytosis.
If the patient were asymptomatic, it would be
appropriate to focus treatment on the
osteomyelitis alone. Because the anemia is
associated with symptoms, specific therapy for
anemia is indicated. Iron therapy by itself is
not effective in anemia of chronic disease.
Cyanocobalamin is a treatment for vitamin B12
deficiency but not for anemia of chronic
disease.

Hematology and Oncology:Question 7


The correct answer is E
Educational Objectives
Recognize the presentation of delayed
hemolytic transfusion reaction.
Critique
The patient is having a delayed hemolytic
transfusion reaction. Multitransfused and
multiparous recipients are at risk for prior
erythrocyte alloimmunization. These antibodies
are usually detected in the pretransfusion
antibody screen by means of reagent cells that
carry most clinically significant antigens. The
time course and presentation are typical for a
delayed hemolytic transfusion reaction arising
from an anamnestic response to rechallenge by
erythrocyte antigens after previous
sensitization.
If the antibody titer decays to a subdetectable
level, as can happen, the antibody screen or
even full crossmatching will be negative using
a pretransfusion sample. Conversely, if the
alloantibody is specific for a low-frequency
antigen not represented on the reagent cells in
the antibody screen, pretransfusion testing will
not detect the incompatibility unless a full
crossmatch is performed. There is usually no
problem because most donors are negative for
low-frequency antigens. Occasionally,
however, a patient might receive a low-
frequency antigen-positive unit. This situation
causes a more acute presentation of the
hemolysis, because preformed, but not
detected, antibody is present.
Failure to keep a unit fully refrigerated can
cause erythrocytes to become nonviable;
hemolysis occurs during storage from
excessive red blood cell metabolic activity. The
effects of transfusion with such a unit are seen
within 24 hours as the cell stroma and
supernatant hemoglobin are cleared and the
nonviable cells are destroyed.
Transfusion can trigger autoimmune
hemolysis, but the fact that this patients direct
antiglobulin test was negative makes it unlikely
that her symptoms are attributable to
autoimmune hemolysis. Glucose-6-phosphate
dehydrogenase deficiency is X-linked and rare
in females. It causes immediate rather than
delayed hemolysis in response to oxidant
stress, and does not cause a positive result on
a direct antiglobulin test. Errors in ABC
compatibility show up immediately rather than
after 7 days.
Hematology and Oncology:Question 8
The correct answer is D
Educational Objectives
Diagnose hereditary hemochromatosis.
Critique
Diagnostic liver biopsy is the procedure of
choice, along with a therapeutic phlebotomy
program. Although the diagnosis of hereditary
hemochromatosis can be made on the basis of
homozygosity for the C282Y mutation in the
HFE gene, important prognostic information
can be obtained from a diagnostic liver biopsy.
If the diagnosis of HFE hemochromatosis is
made before the development of cirrhosis,
then therapeutic phlebotomy to remove excess
iron stores and to prevent reaccumulation
should enable the patient to have a normal life
expectancy. If the diagnosis of
hemochromatosis is made after the
development of cirrhosis, there is an increased
risk for the development of hepatocellular
carcinoma even after removal of excess body
iron.
In general, in a C282Y homozygote, a
diagnostic liver biopsy should be recommended
if the serum ferritin concentration is greater
than 1000 ng/mL or if liver function test results
are elevated at the time of diagnosis. Swelling
of the first and second metacarpophalangeal
joints is a characteristic but not necessary
finding in the diagnosis of HFE
hemochromatosis. A finding of low amounts of
iron in the usual storage sites of bone marrow
macrophages is a characteristic of HFE
hemochromatosis. It is important to proceed
with appropriate diagnostic and therapeutic
measures; treatment of hemochromatosis with
iron removal can arrest organ damage and
improve survival.
Hematology and Oncology:Question 9
The correct answer is D
Educational Objectives
Diagnose myelodysplastic syndrome secondary
to chemotherapy and radiation therapy.
Critique
This patient presents with pancytopenia and
infection, most likely pneumonia. Given his
history of chemotherapy, radiation therapy,
and anemia, the current finding of
pancytopenia strongly suggests that he has
secondary myelodysplastic syndrome or
leukemia. A bone marrow aspiration and
biopsy should be performed to confirm the
diagnosis.
This lethal late complication is seen in 5% of
Hodgkins disease survivors treated with
multimodality therapy. Most of the
myelodysplastic syndromes and acute
leukemias occur 3 to 11 years after therapy.
Deletions or loss of chromosomes 5 and 7 is
commonly associated with alkylating agent
therapy.
Relapse of Hodgkins disease with bone marrow
infiltration and pancytopenia is uncommon and
very unlikely 8 years after successful therapy.
This patients clinical presentation is not
consistent with severe sepsis and associated
pancytopenia.
Hypothyroidism can result in anemia but rarely
pancytopenia. There is no history of new drug
intake to suggest drug-associated bone
marrow suppression.
Hematology and Oncology:Question 10
The correct answer is C
Educational Objectives
Recall the purpose of blood irradiation prior to
transfusion.
Critique
The sole reason for blood irradiation is to
prevent transfusion-related graft-versus-host
disease (GVHD) in patients at risk for it.
Transfusion-related GVHD has a high mortality
rate. Not only are the liver, skin, and
gastrointestinal system under immunologic
attack, but the recipients bone marrow is also
destroyed by the donor T cells.
Gamma irradiation of cellular blood products
prevents transfusion-related GVHD by
rendering donor T cells nonviable and
incapable of mounting a GVHD response. The
dose used is between 25 and 50 Gy. Higher
doses may interfere with the function or
survival of red cells and platelets. Leukocyte
reduction is not sufficient for this purpose
because it leaves behind too many cells.
Irradiation at this dose has no effect on the
transmission of cytomegalovirus (or any
pathogen), alloimmune sensitization, or
leukocyte antigenicity. Although the cells are
nonviable and will not cause a sustained attack
against the host, they can still elaborate
cytokine. Therefore, irradiation will not prevent
febrile reactions. Irradiation has no effect on
any of the mechanisms of transfusion-induced
hemolysis.
Hematology and Oncology:Question 11
The correct answer is E
Educational Objectives
Select the appropriate timing for a work-up for
a hereditary hypercoagulable state.
Critique
No testing is necessary; the patient should be
started on anticoagulation therapy. Although
her family history suggests that she has an
underlying hereditary defect predisposing to
venous thromboembolism, it is unlikely that
she has the factor V Leiden or prothrombin
G2021 0A mutation because these
abnormalities are not found in native Asian
populations.
It is possible that she has a deficiency of
protein C, protein 5, or antithrombin III, but it
is not advisable to test for these abnormalities
upon presentation with an acute thrombotic
event or after the initiation of anticoagulant
therapy. It is best to wait at least 2 weeks
after anticoagulants are discontinued (upon
completion of 3 to 6 months of therapy) to test
for these deficiency states. Testing for a lupus
anticoagulant is reasonable at presentation,
but it should not be done in the presence of
heparin; furthermore, her clinical history is
more suggestive of a hereditary than an
acquired thrombotic disorder (that is,
antiphospholipid antibody syndrome). The yield
of a search for an occult malignancy in a young
woman without any suggestive symptoms or
signs would be extremely low.
Hematology and Oncology:Question 12
The correct answer is D
Educational Objectives
Recognize the clinical presentation and
laboratory findings of paroxysmal nocturnal
hemoglobinuria
Critique
Paroxysmal nocturnal hemoglobinuria (PNH) is
an uncommon acquired chronic disorder of
hematopoietic stem cells. It is characterized by
attacks of intravascular hemolysis and
hemoglobinuria, pancytopenia, and recurrent
venous thromboses. An acquired somatic
mutation in the PIGA gene results in failure to
express phosphatidylinositol-glycan anchor.
This defect leads to absence of certain
membrane proteins in affected hematopoietic
stem cells, including proteins that regulate
complement activation, and targets
erythrocytes for complement-mediated
destruction. A history of dark-colored urine
indicative of hemoglobinuria is noted in only
25% of patients. Recurrent abdominal pain is
common in PNH and is probably related to
portal or mesenteric vein thrombosis. Moderate
splenomegaly is common, and the finding
helps differentiate this disorder from aplastic
anemia.
In PNH, laboratory evaluation typically shows
severe anemia, leukopenia, and moderate to
severe thrombocytopenia. There is evidence of
intravascular hemolysis with low or absent
haptoglobin, and high serum lactate
dehydrogenase levels. There is marked
reticulocytosis, although the absolute
reticulocyte count is often relatively low for the
given degree of anemia. Hemoglobin in the
urine is reabsorbed by the proximal convoluted
tubules and excreted as hemosiderin.
Diagnosis can be made by showing erythrocyte
sensitivity to acid-induced hemolysis (Hams
test) or lack of CD55 and CD59 proteins on
erythrocytes demonstrated by flow cytometry.
A diagnosis of autoimmune hemolytic anemia
would not explain the leukopenia and
thrombocytopenia. Aplastic anemia is not
associated with hemolysis or splenomegaly.
Myelodysplastic syndrome and acute myeloid
leukemia are not associated with severe
hemolysis as seen in this case and are
associated with characteristic peripheral blood
smear findings.
Hematology and Oncology:Question 13
The correct answer is D
Educational Objectives
Recall the principles of ABC compatibility
Critique
This patients liver disease makes it very likely
that his ability to synthesize clotting factor is
decreased; therefore, replacement therapy is
warranted. Even if clotting factor levels were
adequate for hemostasis before transfusion,
the levels would have been diluted through
replacement of his blood volume. The best
choice for clotting factor replacement in this
circumstance is group AB fresh frozen plasma.
Use of fresh frozen plasma avoids the risk of
causing polycythemia.
ABC hemagglutinins arise naturally and are
therfore expected to be present in plasma. In
an emergency when a patient’s ABC type is
unknown, Group O packed cells have neither A
nor B antigens and can be used since they are
the “universal donor” and are compatible with
all patients’ plasma. Even if a sample drawn for
another purpose was found, the blood bank
would be unable to use it to determine ABC
type unless it were proven to be properly
labled and the recipient’s identity had been
positively confirmed at the time of draw (U.S.
Food and Drug Administration requirements).
It is considered safer to use unmatched Group
O packed cells rather than risk making a
mistake leading to ABC incompatibility.
Because of the large amount of donor plasma
in whole blood as opposed to packed red cells,
it must be group specific, and so group O
whole blood can only be given safely to group
O recipients. Individuals who are group AB
have both A and B antigens on their red cells,
and so are incapable of making naturally
occurring anti-A or anti-B. Just as group O is
the universal donor for red cells, group AB is
the universal donor for plasma. Whole blood is
a poor choice to treat coagulopathies because
it loses labile factors during storage.
Hematology and Oncology:Question 14
The correct answer is B
Educational Objectives
Diagnose progressive anemia in a patient with
HIV infection.
Critique
Human parvovirus B19 can cause severe
chronic anemia in patients with HIV infection.
Against a background of underlying immune
deficiency, human parvovirus is able to infect
patients with a particular tropism for erythroid
precursors. The infection produces large,
multinucleated dysplastic proerythroblasts.
The erythrocyte P-antigen is the receptor for
the parvovirus. Infection produces intracellular
products that are directly toxic to erythroid
precursors, leading to a complete cessation of
erythropoiesis with associated
reticulocytopenia. The diagnosis is confirmed
by the demonstration of parvovirus in the
serum, blood, or marrow with the use of
polymerase chain reaction amplification of
DNA. Intravenous immunoglobulin contains
antiparvoviral IgG and neutralizes the infection
with resolution of the anemia.
HIV in general suppresses erythropoiesis and
produces a mild anemia that sometimes
responds to erythropoietin therapy. HIV
infection can produce nonspecific changes
sometimes associated with dysplasia, but not
distinct from other viral infections. Direct
antiglobulin tests are positive in 20% of
patients with HIV, but this condition of
autoimmunity rarely produces serious anemia,
and most patients do not require treatment.
Drug-induced marrow suppression is common
in patients with HIV and is often related to
antiretroviral agents as well as drugs such as
trimethoprim-sulfamethoxazole. This patient’s
rapidly progressive anemia and
reticulocytopenia would be inconsistent with
marrow suppression from use of trimethoprim-
sulfamethoxazole, which is usually mild. Drug-
induced glucose-6-phosphate dehydrogenase
deficiency and hemolysis represent a possible
but unlikely diagnosis in a person of northern
European extraction. The relatively
unremarkable peripheral blood smear and the
“classic” marrow findings make glucose-6-
phosphate dehydrogenase hemolysis very
improbable.
Hematology and Oncology:Question 15
The correct answer is B
Educational Objectives
Diagnose and treat severe aplastic anemia.
Critique
This patient has aplastic anemia, a rapidly fatal
disorder in which myeloid progenitor and stem
cells in the bone marrow are severely
diminished or absent. Either an intrinsic defect
of the stem cells or immune-mediated stem
cell destruction leads to transfusion-dependent
anemia, thrombocytopenia, and severe
neutropenia. The bone marrow is typically
aplastic or hypoplastic, without evidence of
infiltrating malignant cells. The differential
diagnosis includes acute leukemia (particularly
hypoplastic leukemia), myelodysplastic
syndromes, and drug-induced or nutritional
cytopenias.
Allogeneic stem cell transplantation is the
treatment of choice, particularly for patients
younger than 50 years with HLA-identical
sibling donors. Transplantation can result in
long-term survival of 75% to 90%.
Intravenous immunoglobulin is useful in pure
red cell aplasia due to parvovirus B19
infection. Parvovirus infection does not cause
severe anemia in healthy individuals who can
mount an antibody response, and it does not
affect leukocyte and platelet counts. Melphalan
and prednisone are effective in multiple
myeloma. This patient does not have a plasma
cell disorder. It is common to see relative
abundance of plasma cells in severely
hypoplastic bone marrow of patients with
aplastic anemia.
Immunosuppression with antithymocyte
globulin and cyclosporine can induce responses
but is usually reserved for patients who do not
have a suitable donor for allogeneic stem cell
transplantation or who are too old for this
procedure. Long-term follow-up of patients
treated with immunosuppressive therapy
shows an increased incidence of
myelodysplastic syndrome and leukemia,
making this option less attractive for young
patients.
Long-term transfusion therapy will not correct
the severe neutropenia and risk of life-
threatening infections.
Hematology and Oncology:Question 16
The correct answer is C
Educational Objectives
Recognize dilutional coagulopathy as a
consequence of massive transfusion.
Critique
This patient most likely has dilutional
coagulopathy as a result of the massive
transfusion with packed red blood cells and
crystalloid. Clinical bleeding during massive
transfusion is almost always the consequence
of depletion of clotting factors through blood
loss. It can also be a function of dilution by
intravenous fluid and banked blood units that
lack clotting factors. The best therapy is
administration of fresh frozen plasma to aid in
restoration of normal coagulation ability.
Platelet concentrates would not be appropriate
for this patient; low platelet counts are rarely a
cause of bleeding, especially when a dilutional
problem is involved, as in this case. After more
than three blood volumes have been replaced,
however, severe thrombocytopenia can result.
When severe thrombocytopenia occurs earlier,
it is often a sign of disseminated intravascular
coagulation or tissue ischemia causing a
superimposed consumptive coagulopathy.
Whole blood (which is almost never available
because it is needed for component
manufacture) would not help with hemostasis.
The plasma in whole blood loses labile clotting
factors during storage, and the platelets
become nonviable rapidly during refrigerated
storage.
Cryoprecipitate is devoid of most coagulation
factors and serves as a concentrated source of
fibrinogen, factor VIII, and a few other cold-
insoluble proteins. The development of more
purified products to treat hemophilia and von
Willebrand’s disease has obviated the use of
cryoprecipitate as a stand-alone agent to treat
bleeding. Its major use is as an adjunct to
fresh frozen plasma when additional fibrinogen
is required. Cryoprecipitate can also be used as
a fibrinogen source in the making of fibrin glue
used as a biologic sealant, but in this
application it is used chiefly as a patch for
serosal or other surface tears rather than to
stop diffuse bleeding.
Hematology and Oncology:Question 17
The correct answer is C
Educational Objectives
Diagnose and treat chemotherapy-induced
anemia and fatigue.
Critique
Weekly administration of erythropoietin would
be appropriate for this patient. Approximately
75% of patients being treated for cancer
experience fatigue. In most of these patients,
the fatigue is not treated, probably because it
is attributed to the underlying malignancy or
its treatment. Compared with placebo, therapy
with erythopoietin and longer-acting
darbepoetin has been shown to decrease
transfusion requirements, increase hemoglobin
levels, and improve quality of life. Response to
erythropoietin therapy usually occurs within 3
to 5 weeks, and if there is no response by 6
weeks, it is unlikely that the patient will benefit
from further therapy.
Granulocyte colony-stimulating factor
stimulates granulocytic precursors in the bone
marrow and does not affect hemoglobin levels.
Although cessation of chemotherapy would
eventually allow recovery from anemia, it is
not recommended because of the risk that the
cancer would progress. Administration of
androgens plays no proven role in the
treatment of fatigue associated with
chemotherapy-induced anemia. Vitamin E
supplements have not been shown to improve
anemia and fatigue related to cancer
chemotherapy.
Hematology and Oncology:Question 18
The correct answer is D
Educational Objectives
Understand the use of platelet function tests as
screening tests for surgical bleeding.
Critique
It is never possible to guarantee that any given
patient will not experience a bleeding
complication. In most cases, however, these
episodes reflect surgical lesions rather than an
underlying predisposition to bleeding. A
thorough patient history, focusing particularly
on previous surgery, dental procedures,
menstrual bleeding, and bleeding with prior
trauma, remains the best method of detecting
a patient who may be at risk for increased
bleeding during surgery.
The bleeding time might be a useful test when
screening for inherited or acquired deficiencies
in platelet function in a patient with an
appropriate history. It cannot, however,
predict surgical bleeding when used to screen
unselected surgical patients. The Platelet
Function Analyzer-100 (PFA-100) may be more
sensitive and perhaps more specific in the
diagnosis of platelet disorders and von
Willebrand’s disease than bleeding time; it has
not, however, been validated as a screening
test for surgical bleeding in unselected
patients. Therefore, in this regard it should not
be considered superior to the bleeding time.
Platelet aggregation studies should be done
only after a screening test such as the bleeding
time or the PFA-i 00 analysis suggests an
underlying platelet disorder. These studies are
expensive and have not been established as
preoperative screening tests.
The prothrombin time and partial
thromboplastin time may be prolonged in
hemophilia A or B or in deficiencies of other
intrinsic pathway factors such as factor XI or
factor XII. They can, however, be normal in
von Willebrand’s disease or in moderate
deficiencies of factors VIII or IX that are
sufficient for normal hemostasis but may
nonetheless predispose a patient to surgical
bleeding. Therefore, the partial thromboplastin
time is not a useful presurgical screening test
(although it has been suggested that the
partial thromboplastin time be used to screen
patients of Ashkenazi Jewish descent because
of the high incidence of factor Xl deficiency in
this population).
Hematology and Oncology:Question 19
The correct answer is B
Educational Objectives
Diagnose anemia with low serum iron.
Critique
There are two main reasons patients do not
respond to oral iron therapy: Either they are
noncompliant, or they are not iron-deficient. A
low serum iron level by itself does not
distinguish iron deficiency from anemia of
chronic disease. At the very least, the serum
ferritin level must also be determined.
The history and examination findings largely
rule out noncompliance with treatment.
Therefore, the physician should order a serum
ferritin test to correctly characterize the
anemia syndrome. Administration of an oral
iron polysaccharide or possibly parenteral iron
or addition of ascorbic acid would be correct if
the patient were found to be iron-deficient.
Hematology and Oncology:Question 20
The correct answer is B
Educational Objectives
Diagnose leukoerythroblastosis due to
tuberculosis.
Critique
The patient has fever, anemia, and a
leukoerythroblastic blood smear characterized
by immature myeloid cells and the presence of
nucleated erythrocytes and teardrop forms.
The differential diagnosis includes metastatic
cancer, myelofibrosis with myeloid metaplasia,
severe stress (blood loss, hemolysis, infection),
and miliary tuberculosis. Distortion of the
marrow sinuses causes deformity of the
erythrocytes and also allows transit of
immature myeloid and erythroid marrow cells
into the peripheral blood. In this patient, the
history of exposure to tuberculosis, the recent
use of corticosteroids as an
immunosuppressive agent, and the anergy
noted by skin testing should raise a very high
degree of suspicion of disseminated
tuberculosis.
Nearly 90% of patients with miliary
tuberculosis have anemia. Although infiltrative
myelopathies usually are caused by metastatic
cancer, tuberculosis may be implicated in as
many as 5% of cases. Tuberculosis is
increasingly recognized in elderly patients who
lack pulmonary symptoms, who have been
exposed to corticosteroids or other
immunosuppressive agents, and whose
findings slowly evolve toward a picture
consistent with an infiltrative myelopathy.
In this patient the easiest, safest approach to
confirm the diagnosis is bone marrow biopsy
with culture, which yields a diagnosis in 80%
of patients with miliary tuberculosis. The
demonstration of caseating granulomas is
strong evidence for tuberculosis or
histoplasmosis. Cultures of the marrow would
likely grow acid-fast bacilli. In the presence of
abnormal liver function studies, liver biopsy
probably would yield a diagnosis but has a
higher procedure risk profile in an elderly
patient compared with bone marrow biopsy.
Similarly, although open-lung biopsy would
also yield a diagnosis, it is very invasive.
Splenomegaly is consistent with tuberculosis,
and splenectomy would not be indicated
because preservation of splenic function is
desirable. Aspiration of an abdominal lymph
node likewise could yield a diagnosis of
tuberculosis, but the node is small and could
be difficult to access. Furthermore, the risk of
complicating infection or bleeding would be
greater than in bone marrow biopsy.
Since bronchoscopy with bronchoalveolar
lavage yields a diagnosis of tuberculosis in only
30% of patients with miliary disease, an open
lung biopsy or a transbronchial biopsy would
be appropriate next steps if the marrow biopsy
did not reveal a diagnosis.
Hematology and Oncology:Question 21
The correct answer is A
Educational Objectives
Recognize the need for prompt diagnosis of red
cell aplasia in sickle cell disease.
Critique
Infection with parvovirus B19 is a common
cause of transient reticulocytopenia in humans.
Patients with sickle cell anemia or thalassemia
compensate for their chronic hemolytic state
through elevated erythropoietic activity and
can rapidly develop life-threatening anemias in
the face of
parvovirus-mediated erythrocyte
hypoproliferation.
Although the formation of alloantibodies can
accelerate hemolysis of transfused blood, they
do not adversely affect marrow erythropoiesis.
From a practical perspective, donor
erythrocytes will have been cleared from the
circulation long before the 6-month
anniversary of the patients last transfusion.
Rarely, profound nutritional disorders can
cause marrow aplasia, but the presenting
symptoms of severe folate deficiency would be
megaloblastic, macrocytic anemia. Although
splenic sequestration crisis can cause a rapid
fall in hemoglobin levels, compensatory
reticulocytosis is typically maintained.
Hematology and Oncology:Question 22
The correct answer is E
Educational Objectives
Diagnose and treat heparin-associated
thrombocytopenia.
Critique
The best approach to management of this
patient would involve discontinuation of
heparin therapy followed immediately by the
institution of alternative anticoagulation
therapy such as lepirudin or argatroban.
This patient with presumptive exposure to
heparin during prior peripheral vascular
surgery develops acute thrombocytopenia 2
days after being exposed to heparin. There are
no other obvious causes for thrombocytopenia,
and a clinical diagnosis of heparin-induced
thrombocytopenia should be made.
After the diagnosis is made, it is essential to
discontinue all heparin exposure immediately.
Even when the platelet count has not reached
levels below 150,000/μL (or the lower limit of
normal for the specific laboratory), an acute
decrease in the platelet count in the setting of
ongoing heparin exposure should signal the
need to discontinue heparin therapy.
Continuation of exposure to heparin can cause
life-threatening cardiovascular thrombotic
events. Therefore, continuing the heparin
infusion regardless of close monitoring or in
anticipation of diagnosing the disease on the
basis of laboratory studies is not an acceptable
alternative.
Although antiplatelet therapy may have value
in the treatment of heparin-induced
thrombocytopenia, it is not acceptable first-line
therapy and initiating it does not make it safe
to continue the heparin therapy. Moreover,
recent studies suggest that patients with
heparin-induced thrombocytopenia have a
hypercoagulable state associated with a
significant risk of thrombosis that may persist
for days to weeks after heparin is discontinued.
To address this risk, particularly in this patient
in need of ongoing anticoagulation for cardiac
ischemia, discontinuing heparin as the only
intervention is not sufficient.
Hematology and Oncology:Question 23
The correct answer is A
Educational Objectives
Recognize the need for compatibility blood
samples in patients requiring multiple
transufsions..
Critique
The blood bank wants a new sample because
this patient’s recent transfusion may have
stimulated new alloantibody specificities to
emerge that would not be present in older
samples. The patient’s history marks her as an
antibody responder, and there is a higher
likelihood that she will make further antibodies
upon continued challenge. Although 4 days is
not sufficient time for primary
alloimmunization to proceed to detectable
levels of antibody, studies have shown that
anamnestic reactions can produce clinically
detectable antibodies in that short a time;
hence the ‘3-day rule’ for compatibility
samples.
There is no reason to suspect hemolysis,
because of the appropriate rise in hemoglobin.
Antibody screening does not include a direct
antiglobulin test, which is the preferred test to
identify an ongoing hemolytic reaction. It is
common to use group O cells when antigen-
negative units are required, in view of the
universal compatibility of group O blood. In the
absence of a hemolytic reaction, transfused
group O cells are expected to be detectable in
circulation after only 3 days, but there is no
reason to look for them.
Autoantibodies may be present, but
compatibility-testing methods were not
designed to screen for them. Rather, they are
used to identify homologous units that have a
high likelihood of surviving as long as the
autologous cells in circulation. Even though
autoantibodies can be detected in the antibody
screen, the purpose of this testing is to detect
alloantibodies. If an alloantibody had been
missed during compatibility testing, it would be
important to look for it only if hemolysis were
suspected.
Hematology and Oncology:Question 24
The correct answer is D
Educational Objectives
Differentiate between iron deficiency anemia
and anemia of chronic disease.
Critique
In this context, staining a bone marrow
aspirate for iron is the gold standard for
diagnosing iron deficiency anemia.
Anemia with a low serum iron concentration is
characteristic of the anemia of chronic disease
and of iron deficiency anemia. The anemia of
chronic disease is also typically associated with
low serum transferrin concentration and
normal to elevated serum ferritin
concentration, whereas iron deficiency anemia
is also typically associated with elevated
transferrin concentration and low serum ferritin
concentration. When iron deficiency occurs in
the setting of inflammation, the serum ferritin
concentration may be normal because ferritin
is an acute- and chronic-phase reactant.
Findings consistent with but not diagnostic of
iron deficiency in this patient are low mean
corpuscular volume, increased red cell
distribution width, and high normal to elevated
serum transferrin concentration. The finding of
iron deficiency should prompt a search for
blood loss, and endoscopy of the
gastrointestinal tract should be considered in
this regard.
Treatment with iron without identifying the
source of blood loss could delay the diagnosis
of an otherwise curable gastrointestinal
malignancy. Furthermore, iron therapy should
not be given to a patient with HIV disease in
the absence of iron deficiency, because iron
therapy can promote certain infections and
toxicities in HIV disease.
Erythropoietin can be used to treat anemia in
patients with HIV disease but should generally
be considered only after other causes of
anemia have been ruled out and the serum
erythropoietin level is found to be less than
500 IU/mL.
Hematology and Oncology:Question 25
The correct answer is B
Educational Objectives
Diagnose and treat relapsed lymphoma.
Critique
Relapse should be documented by a lymph
node biopsy, and staging should be completed
with bone marrow aspiration and biopsy.
Relapse occurs in approximately 50% to 60%
of patients with advanced-stage (II-IV)
aggressive non-Hodgkin’s lymphoma treated
with cyclophosphamide, hydroxydaunomycin,
vincristine, and prednisone. Systemic
symptoms, findings on physical examination
and CT scan, and the elevated serum lactate
dehydrogenase level are all consistent with
relapsed lymphoma in this patient.
Most patients with relapsed lymphoma respond
to second-line salvage chemotherapy. In these
chemotherapy-sensitive patients, autologous
stem cell transplantation has been shown to be
superior to salvage chemotherapy alone; 40%
to 50% of patients who underwent
transplantation had long-term disease-free
survival.
By contrast with the indolent lymphomas,
relapsed aggressive lymphomas have a
progressive clinical course, and treatment
should not be delayed. Because there are
numerous effective second-line chemotherapy
regimens for this patient, a trial of
investigational agents is not appropriate at this
time. Repeating a course of CHOP is not
recommended because of the increased risk of
doxorubicin-induced cardiotoxicity and the
availability of more active salvage regimens.
Hematology and Oncology:Question 26
The correct answer is D
Educational Objectives
Recognize the increased risk of venous
thromboembolism associated with hormone
replacement
therapy, particularly in an asymptomatic
carrier with an inherited thrombotic defect.
Critique
Not only should this patient be advised against
the use of hormone replacement therapy
(HRT), because of her history, she should use
anticoagulant prophylaxis at times of increased
risk. HRT has been shown to increase the risk
of venous thromboembolism, and the risk is
compounded in patients with an underlying
hereditary thrombotic defect. Data from the
Women’s Health Initiative indicate that HRT is
also not beneficial in reducing the risk of
arterial thrombotic complications.
Although warfarin therapy is likely to be highly
effective in reducing the risk of venous
thromboembolism, it is associated with a risk
of major bleeding and for that reason is
contraindicated. Aspirin provides minimal if any
protection against the development of venous
thrombosis.
Hematology and Oncology:Question 27
The correct answer is A
Educational Objectives
Recognize and treat drug-induced
thrombocytopenia.
Critique
Determining the cause of hematologic
abnormalities may be difficult in transplant
recipients. Trimethoprim-sulfamethoxazole is a
leading cause of drug-induced
thrombocytopenia, and the most likely cause of
thrombocytopenia in this patient. The
thrombocytopenia usually occurs acutely, soon
after beginning the drug, but may also develop
insidiously. Discontinuation of
trimethoprim-sulfamethoxazole should lead to
improvement in the platelet count within 7 to
14 days in most cases—if improvement does
not occur, other diagnoses should be
considered.
Because the hemoglobin level and leukocyte
count are normal, and the peripheral blood
smear shows only decreased platelets, the
likelihood that thrombocytopenia is the result
of a primary or secondary marrow process is
low and bone marrow examination is not
required. Platelet transfusion would be
indicated if the platelet count were lower than
10,000/μL or if there were active bleeding.
Epsilon-aminocaproic acid is an antifibrinolytic
drug that is sometimes of use in patients with
thrombocytopenic bleeding but is not indicated
in this patient. Immunosuppressive therapy
should not be discontinued because doing so
would risk the viability of the allograft. If the
peripheral blood smear revealed a
microangiopathic process, as demonstrated by
the presence of schistocytes, then changing
the immunosuppressive regimen or decreasing
the doses of cyclosporine and/or tacrolimus
should be considered, as these agents are
associated with the development of post-
transplant microangiopathy.
Hematology and Oncology:Question 28
The correct answer is A
Educational Objectives
Recognize the increased risk posed by a prior
episode of thrombosis and its implications for
pregnancy.
Critique
The best management would be to observe her
carefully throughout her pregnancy and
administer prophylactic low-molecular-weight
heparin for 6 weeks after she gives birth. The
use of oral contraceptives confers a fourfold
increase in the risk of venous
thromboembolism, but the risk of recurrence is
quite low in women with prior thrombosis in
association with just such a transient risk
factor.
The study from Brill-Edwards et al. reported
that the antepartum risk of venous thrombosis
was extremely low in pregnant women with a
prior thrombotic event in association with
transient risk factors. Therefore observation
rather than antepartum administration of low-
molecular-weight heparin is appropriate for
this patient. The postpartum period is
associated with a higher risk of thrombosis,
and anticoagulation is generally recommended
for a period of 6 weeks.
The hereditary thrombophilias, especially
factor V Leiden mutation, have been associated
with a threefold increase of stillbirth in the
second and third trimester, but it remains a
matter of controversy whether other adverse
obstetric outcomes such as preeclampsia and
intrauterine growth retardation are associated
findings. Termination of pregnancy would
therefore not be appropriate.
Hematology and Oncology:Question 29
The correct answer is E
Educational Objectives
Recognize when to initiate therapy for
idiopathic (immune) thrombocytopenic
purpura.
Critique
The correct approach to management of this
patient, who has asymptomatic
thrombocytopenia, is to monitor her platelet
count closely and intervene therapeutically if
the platelet count drops further. The
differential diagnosis includes hypoproliferative
disorders such as myelodysplasia, congenital
thrombocytopenias such as the May-Hegglin
anomaly, or consumption disorders such as
thrombotic microangiopathies, disseminated
intravascular coagulation, or idiopathic
thrombocytopenic purpura. The most likely
disorder in a patient of this age who is
otherwise healthy and in whom the other
hematologic parameters are normal is
idiopathic (immune) thrombocytopenic
purpura. Most experts would not recommend
instituting therapy for idiopathic
thrombocytopenic purpura in patients with a
platelet count above 30,000/μL in the absence
of bleeding.
A bone marrow aspiration with biopsy is not
required for the diagnosis of idiopathic
thrombocytopenic purpura and is unlikely to be
of value in an otherwise healthy young woman
with no hematologic abnormalities. Platelet
aggregation studies are likely to be abnormal
simply because of her thrombocytopenia; they
are used to investigate potential causes of
platelet dysfunction rather than abnormalities
in platelet number. Genetic causes of
thrombocytopenia are usually diagnosed in
childhood and would be uncommon as a first
diagnosis in a 27-year-old with no suggestive
family history. Moreover, most congenital
thrombocytopenias are associated with platelet
morphologic abnormalities that were not
observed in this case. Therefore, there is no
reason to study the patient’s relatives.
Hematology and Oncology:Question 30
The correct answer is B
Educational Objectives
Diagnose and treat polycythemia vera.
Critique
This patient has polycythemia vera; therefore,
phlebotomy is the treatment of choice.
Polycythemia vera is characterized by an
increased red cell mass; mild elevation in
leukocyte and platelet counts is also common.
Clinical symptoms are the result of
hyperviscosity, hypervolemia, and
hypermetabolism. Headache, pruritus,
dyspnea, blurred vision, and night sweats are
common. Patients usually have facial plethora
and splenomegaly.
Bleeding and thromboembolic events (arterial
and venous) represent potentially life-
threatening complications of this disease.
Elevated red cell mass (or hematocrit values
above 60% for men and 56% for women) in
the absence of secondary causes of
erythrocytosis, and the presence of
splenomegaly or an abnormal marrow
karyotype other than the Philadelphia
chromosome establish the diagnosis of
polycythemia vera. Regular phlebotomy to
maintain the hematocrit at less than 45%
promptly controls the symptoms of the
disease, but the thrombotic risk remains
elevated.
Splenectomy is reserved for the late stages of
the disease, for palliation of symptomatic
splenomegaly with hypersplenism. Induction
chemotherapy with cytarabine and idarubicin
does not play a role in this disease unless
there is progression to leukemia. Alcohol use is
not associated with polycythemia and
abstaining would not affect the course of the
disease. Imatinib mesylate is specific therapy
for chronic myeloid leukemia and is not
indicated in polycythemia vera.
Hematology and Oncology:Question 31
The correct answer is C
Educational Objectives
Recognize and manage gestational
thrombocytopenia.
Critique
The major differential diagnosis in this patient
is between idiopathic (immune)
thrombocytopenic purpura and gestational
thrombocytopenia. Although idiopathic
thrombocytopenic purpura cannot be excluded
with absolute certainty, the most common
cause of mild, asymptomatic thrombocytopenia
in pregnancy, particularly when it develops in
the third trimester, is incidental (gestational)
thrombocytopenia. The pathogenesis of this
disorder is not well understood, but it is not
associated with adverse maternal or fetal
outcomes and therefore does not require
specific intervention. If the platelet count drops
below 70,000/μL or other pregnancy-related
complications occur, other diagnoses should be
considered.
This patient presents with mild
thrombocytopenia in the third trimester of an
otherwise uncomplicated pregnancy. There is
no personal or family history of prior
hematologic disease, nor is there an elevated
blood pressure that would suggest
preeclampsia. The peripheral blood smear does
not reveal schistocytes; therefore there is no
evidence of an underlying microangiopathic
hemolytic anemia. The clinical history and
physical examination are unremarkable;
therefore there is also no evidence of an
underlying disorder such as sepsis, which
would predispose the patient to the
development of disseminated intravascular
coagulation. The patient is taking no
medications that are associated with drug-
induced thrombocytopenia.
Hematology and Oncology:Question 32
The correct answer is B
Educational Objectives
Recall the purpose of leukocyte reduction.
Critique
Donor leukocytes can act as immunizing
antigens and are the chief cause of human
leukocyte antigen (HLA) alloimmunization. This
patient’s history argues against existing HLA
sensitization through pregnancy or transfusion.
Prevention of primary alloimmunization to HLA
antigens can be achieved by leukocyte
reduction. Success in finding a compatible
cadaveric organ that will not be rejected
acutely through humoral mechanisms is
directly related to the presence of HLA class I
antibodies. Prevention of HLA alloimmunization
is critical to her chance of obtaining an organ
and stopping dialysis.
Many adverse reactions to transfusions are
related to the presence of donor leukocytes in
blood products. These cells can harbor
pathogens such as cytomegalovirus or HTLV-1,
which are not transmitted through cell-free
plasma. The patient is cytomegalovirus-
positive herself; therefore transmission of the
virus is not a concern. She is at risk for
reactivation of her own virus during immune
suppression after transplant.
Current technology for leukoreduction does not
remove enough T cells to prevent graft-versus-
host disease. Prestorage leukocyte reduction
eliminates donor-derived cytokine elaboration
that causes febrile non hemolytic reactions;
the actual incidence of these reactions is less
than 5% of transfusions. None of these effects
is as critical to her health as the prevention of
HLA alloimmunization. Leukocyte reduction has
no effect on erythrocyte alloimmunization or
hemolytic reactions.
Hematology and Oncology:Question 33
The correct answer is D
Educational Objectives
Understand the features of glucose-6-
phosphate dehydrogenase deficiency.
Critique
The patient has glucose-6-phosphate
dehydrogenase (G6PD) deficiency and acute,
drug-induced hemolytic anemia. G6PD protects
cells from membrane oxidation. In deficient
persons, membrane oxidation causes cells to
become rigid and susceptible to mechanical
hemolysis and trapping by the spleen and liver.
In this patient the occurrence of acute
intravascular and extravascular hemolysis
following administration of primaquine and the
presence of Heinz bodies and characteristic
bite cells on peripheral blood smear make this
diagnosis very likely.
The presence of a leukoerythroblastic blood
smear is consistent with the stress of acute
hemolysis and not necessarily a marrow
infiltrative process. The absence of
hemoglobinuria does not necessarily exclude
an acute intravascular hemolytic episode.
Because hemoglobin will be present only in the
immediate period of the hemolysis, the
detection of urine hemosiderin may indicate a
more remote prior episode. Hemoglobin is
absorbed into the proximal renal tubules where
it is degraded ultimately into hemosiderin,
which spills slowly into the urine.
Sensitive screening tests are available for
G6PD deficiency but may be abnormal in other
hexose monophosphate shunt deficiencies. The
detection of low G6PD enzyme activity during
an acute hemolytic crisis is often futile because
the erythrocytes most susceptible to
destruction are those with the lowest enzyme
activity (the younger cells have greater
enzyme activity and therefore survive).
Primaquine is one of a number of drugs that
produce acute hemolysis in G6PD-deficient
subjects.
Hematology and Oncology:Question 34
The correct answer is A
Educational Objectives
Avoid needless radiologic evaluation for an
occult neoplasm in an older patient with a first
venous thrombotic event.
Critique
No further evaluation is necessary. An
unprovoked episode of deep venous
thrombosis in older patients has been
associated with an increased incidence of an
occult malignancy. An extensive search for an
occult neoplasm in otherwise healthy patients
does not always detect a treatable malignancy
or is not cost-effective.
Although the factor V Leiden and prothrombin
G2021 0A mutations are frequently present in
older patients with a first deep venous
thrombosis in the absence of a family history,
deficiencies of antithrombin III, protein C, or
protein S are very rarely encountered in this
setting, and these tests are not recommended.

Hematology and Oncology:Question 35


The correct answer is C
Educational Objectives
Recognize the limitations of laboratory studies
in the diagnosis of thrombotic
microangiopathy.
Critique
This patient presents with signs and symptoms
consistent with thrombotic thrombocytopenic
purpura. The diagnosis of thrombotic
thrombocytopenic purpura is a clinical one, and
microangiopathic hemolytic anemia and an
elevated serum lactate dehydrogenase level
without another apparent cause should be
sufficient evidence of thrombotic
thrombocytopenic purpura to institute plasma
exchange therapy.
Thrombotic thrombocytopenic purpura
primarily affects the microvasculature, and
brain ischemia, although diffuse, does not
generally lead to the development of lesions of
sufficient size to be detected by CT; therefore,
CT of the head is often normal. Liver function
studies may also be normal in these patients
and are not helpful in reaching a diagnosis.
Although functional deficiency of the vWF-
cleaving protease, ADAMTS 13, is common in
patients with thrombotic thrombocytopenic
purpura, diagnosis and therapy of this disorder
should be made on clinical criteria. The utility
of functional ADAMTS 13 assays in diagnosing
and monitoring the therapy of this disorder has
not been established. Continued assay
improvement and correlation with clinical
disease in humans is required before assays of
ADAMTS 13 are accepted for use in clinical
decision making.
Hematology and Oncology:Question 36
The correct answer is B
Educational Objectives
Diagnose chronic myeloid leukemia in a patient
presenting with neutrophilia.
Critique
Typically, chronic myeloid leukemia (CML) is
diagnosed on a routine blood count that
demonstrates leukocytosis with circulating
myeloid precursors. Patients may have fatigue,
weight loss, splenomegaly, and
thrombocytosis. A balanced translocation
between chromosomes 9 and 22 (Philadelphia
chromosome) creates a unique gene, bcr-abl,
which codes a 210-kDa protein (p210) that
functions as a tyrosine kinase. This kinase was
shown to cause a CML-like disease in mice
when expressed in hematopoietic cells. The
Philadelphia chromosome can be detected in
peripheral blood mononuclear cells by
interphase fluorescence in situ hybridization
(FISH) analysis and is diagnostic of CML.
This patients abdominal symptoms can be
explained by his splenomegaly. Although
significant neutrophilia can be associated with
occult tumors, the lack of constitutional
symptoms and weight loss and absence of
anemia and occult blood in the stool make an
occult gastrointestinal malignancy very
unlikely. CT of the abdomen is not warranted
in this case.
Leukemoid reaction attributable to infection is
unlikely in this patient because he does not
have fever and lacks other manifestations of
infection. Acute bacterial endocarditis can
present with neutrophilia with a left shift but is
commonly associated with fever and
constitutional symptoms that are lacking in this
case. Therefore blood cultures and
echocardiography are not necessary.
The leukocyte alkaline phosphatase score is
often low in chronic myeloid leukemia, but a
low score is not diagnostic of CML. It may also
be low in other hematologic disorders, such as
paroxysmal nocturnal hemoglobinuria,
hypophosphatasia, myelofibrosis, and acute
myeloid leukemia.
Hematology and Oncology:Question 37
The correct answer is D
Educational Objectives
Diagnose heparin-induced thrombocytopenia.
Critique
This most likely cause of this patient’s
thrombocytopenia is heparin-induced
thrombocytopenia. Extracorporeal circulation
during coronary artery bypass grafting induces
transient thrombocytopenia, but it is
uncommon for the platelet count to drop by
more than 25% because of bypass alone.
Furthermore, platelet counts generally
normalize within 3 to 4 days after surgery.
Therefore, surgery alone cannot account for
this persistent and severe thrombocytopenia.
Although thrombotic thrombocytopenic purpura
may on occasion occur soon after bypass
surgery, the fact that the peripheral blood
smear does not reveal increased schistocytes
excludes this diagnosis.
Idiopathic thrombocytopenic purpura cannot be
entirely excluded but does not usually develop
so rapidly and would be less common than
heparin-induced thrombocytopenia in an
elderly patient in this setting. Disseminated
intravascular coagulation is relatively unlikely,
given the patient’s normal prothrombin time
and partial thromboplastin time, and no
obvious primary disorder that would stimulate
its development.
It is likely that the patient developed heparin-
induced thrombocytopenia at or immediately
after the time of her surgical procedure. The
platelet count might actually have been
improving at the time of her read mission. At
such a time she would be expected to be
particularly susceptible to recurrent exposure
to heparin, as her antibody levels to heparin
would be maximal. The role of heparin-induced
thrombocytopenia in her persistent
thrombocytopenia is supported by the
precipitous decline in the platelet count and
development of lower-extremity ischemia after
reexposure to heparin.
Hematology and Oncology:Question 38
The correct answer is B
Educational Objectives
Distinguish among different causes of
hypochromic microcytic anemia.
Critique
Anemia can result from any of the listed
conditions, although the anemias associated
with uncomplicated glucose-6-phosphate
dehydrogenase deficiency and sickle cell
disease are typically normocytic and
normochromic. By contrast, the anemias of
iron deficiency, α-thalassemia, and β-
thalassemia, are characteristically both
microcytic and hypochromic (low mean
corpuscular volume and mean corpuscular
hemoglobin values).
Iron deficiency anemia would be unusual in an
otherwise well-appearing young man.
Furthermore, it would be associated with a
hypoproliferative state (low reticulocyte count)
and is not a recognized cause of elevated
hemoglobin A2 levels (in fact, in patients with
β-thalassemia who are also iron deficient,
elevated hemoglobin A2 can be reduced into
the normal range). Iron deficiency could be
ruled out by normal plasma iron studies or the
presence of stainable iron on bone marrow
biopsy.
Patients with mild α- or β-thalassemia appear
well, and the spleen is often not palpable. They
have mild elevations in reticulocyte count
consistent with their chronic hemolytic
disorder. An elevation in the hemoglobin A2
level is consistent with β-thalassemia and is
one of the chief methods for distinguishing this
disorder from mild ct-thalassemia.
Glucose-6-phosphate dehydrogenase
deficiency would not account for the patient’s
signs and symptoms.
Hematology and Oncology:Question 39
The correct answer is C
Educational Objectives
Diagnose delayed-onset heparin-induced
thrombocytopenia.
Critique
This patient developed severe
thrombocytopenia and an arterial thrombotic
event 2 weeks after a hospitalization in which
she was given heparin. She has delayed-onset
heparin-induced thrombocytopenia. This
syndrome is associated with the development
of anti-heparin-platelet factor 4 antibodies, as
well as thrombocytopenia and thrombosis
common to standard presentations of heparin-
induced thrombocytopenia. In one study, the
mean onset of thrombocytopenia and
thrombosis was 9.2 days after withdrawal of all
heparin; in some patients, it did not occur until
19 days afterward. The presence of this
disorder could be confirmed by measuring
heparin-platelet factor 4 antibodies using the
14C-serotonin release assay. The pathogenesis
of this disorder is not well understood; heparin
is no longer present when the disease
develops. It is assumed that the pathologic
antibodies recognize conformationally altered
platelet factor 4 bound to the platelet surface
or to endogenous glycosaminoglycans
expressed on the surface of endothelial cells.
Although there are not many studies of the
therapy of this disorder, patients have
responded to alternative anticoagulation in a
manner similar to those with more standard
presentations of heparin-induced
thrombocytopenia.
Genetic assays for factor V Leiden or
prothrombin G2021 0A mutation would not
account for the patients thrombocytopenia.
Moreover, these genetic defects are associated
with venous rather than arterial thrombosis.
Platelet aggregation studies would be
unrevealing, as they would not test platelet
function in the presence of heparin.
Measurements of platelet-associated
immunoglobulin G are nonspecific and would
not help identify the patient’s disorder.
Hematology and Oncology:Question 40
The correct answer is D
Educational Objectives
Recall the risks of viral transmission through
transfusion.
Critique
Hepatitis B poses the highest risk. With the
combination of donor health screening,
confidential unit exclusion, and highly sensitive
testing, the risk of HIV transmission from
asymptomatic donors is negligible. The risk of
hepatitis C transmission is also extremely low,
despite a relatively high incidence (0.5%) in
the general first-time donor population. One of
the reasons these two viruses have such a low
risk is the use of nucleic acid testing, which
relies on polymerase chain reaction technology
to amplify viral material. Unfortunately,
technical difficulties have delayed nucleic acid
testing for hepatitis B. Although testing for
hepatitis B surface antigen was initiated in the
late 1970s and has been constantly refined and
improved, there are still donors who are
antigen-negative but capable of transmitting
the virus. Screening for hepatitis B core
antibody can identify some but not all of these
donors.
Hematology and Oncology:Question 41
The correct answer is C
Educational Objectives
Administer antithrombotic therapy to a patient
with antiphospholipid antibody syndrome and
transient cerebral ischemia.
Critique
Lupus anticoagulants are acquired defects that
are associated with arterial as well as venous
thrombotic events. The diagnosis of
antiphospholipid antibody syndrome can be
made in this patient, on the basis of clinical
findings of transient cerebral ischemic episodes
in association with the presence of a lupus
anticoagulant and a high level of cardiolipin
IgG antibodies. Retrospective studies, which
comprise for the most part patients with
antiphospholipid antibody syndrome and
underlying collagen-vascular disease, indicate
that warfarin to a target INR greater than 3 is
required to prevent recurrent thrombotic
events. Recent data however indicate that an
INR of 2 to 3 is adequate for preventing
recurrent venous thrombosis. Antiplatelet
therapy alone does not adequately protect
patients with antiphospholipid syndrome
against arterial thrombosis.
Hematology and Oncology:Question 42
The correct answer is A
Educational Objectives
Diagnose and treat warm antibody
autoimmune hemolytic anemia.
Critique
This patient has warm antibody autoimmune
hemolytic anemia. Her history is typical for the
disease, and her physical findings of jaundice
and mild splenomegaly are also consistent with
this diagnosis. Laboratory studies reveal a
decreased hemoglobin and elevated
reticulocyte count, and the peripheral blood
smear is notable for polychromatophilia and
occasional spherocytes and rare nucleated
erythrocytes. Taken together, these findings
are consistent with increased erythrocyte
destruction in the periphery, with a
compensatory bone marrow response.
The appropriate initial therapy for this patient
is prednisone, 1 mg/kg daily, to which 65% to
80% of patients respond. Intravenous
immunoglobulin and danazol may have activity
in some patients with autoimmune hemolytic
anemia but are not first-line agents. Likewise,
splenectomy should be reserved for patients
who are corticosteroid-resistant.
Erythropoietin may be of value in certain
hypoproliferative anemias or in patients with
anemias secondary to exogenous influences
such as chemotherapy. However, endogenous
erythropoietin levels are elevated in patients
with autoimmune hemolytic anemia, and the
use of exogenous erythropoietin is of uncertain
benefit.
Hematology and Oncology:Question 43
The correct answer is C
Educational Objectives
Diagnose and treat high-risk essential
thrombocythemia.
Critique
Essential thrombocythemia is the most
common myeloproliferative disorder affecting
middle-aged and older adults and is
characterized by thrombotic and hemorrhagic
complications. Diagnosis is made in patients
with platelet counts in excess of 600,000IjiL in
the absence of known causes for reactive
thrombocytosis.
Essential thrombocythemia may be difficult to
distinguish from other myeloproliferative
syndromes. A hemoglobin level of 13 g/dL or
lower excludes polycythemia vera; absence of
t(9;22) excludes chronic myeloid leukemia,
and absence of marked marrow fibrosis
excludes myelofibrosis.
Low-dose aspirin therapy is beneficial in the
treatment of ischemic neurologic and vision
symptoms and erythromelalgia in essential
thrombocythemia. Patients older than 60 years
or patients with a history of thrombotic events
are considered to be at high risk for
thromboembolic complications, and use of
hydroxyurea to reduce the platelet count below
400,000/μL significantly reduces this risk.
Therefore aspirin therapy alone would not be
sufficient for this patient.
Treatment with a nonmutagenic agent such as
anagrelide, which blocks megakaryocyte
maturation, would also reduce the platelet
count, but there are no randomized data
regarding reduction of thrombotic events.
Warfarin anticoagulation should be avoided in
this patient because her essential
thrombocythemia puts her at high risk for
bleeding complications. Busulfan therapy is
associated with significant risk of secondary
leukemias and does not have a role in
management of essential thrombocythemia.
Imatinib mesylate is specific therapy for
chronic myeloid leukemia and is not indicated
in essential thrombocythemia.
Hematology and Oncology:Question 44
The correct answer is A
Educational Objectives
Recognize the indications for transfusion of
fresh frozen plasma.
Critique
This patients liver disease gives evidence of
poor synthetic function. Although the results of
the coagulation tests are only slightly
abnormal, it can be anticipated that excessive
hemorrhage due to impaired hemostasis will be
a complication during major surgery. The
cause of his recurrent venous thrombosis is
unclear.
The risks of donor exposures preclude
prophylactic transfusion of fresh frozen plasma
merely to treat elevated coagulation findings.
In general, the hemostatic effect of fresh
frozen plasma lasts only as long as the biologic
life of the shortest factors being replaced. For
example, factor VII has a steady-state half-life
of approximately 6 hours.
Also because of the risks of transfusion,
hypovolemia and hypoproteinemia are best
treated with sterilized plasma derivatives such
as albumin or with plasma expanders. Albumin
may transiently correct hypoproteinemia
caused by malnutrition or decreased synthesis
but is usually not effective if the protein
deficiency is due to nephrosis or capillary leak.
The anticoagulant effect of warfarin can be
temporarily corrected by fresh frozen plasma
by supplying factors that are synthesized with
normal gamma carboxylation. Heparin,
however, behaves as an inhibitor and its effect
is not corrected through fresh frozen plasma
administration. This patient’s low-molecular-
weight heparin should be withheld the day
before surgery to allow for the elimination of
the anticoagulant effect; fresh frozen plasma is
not, however, required because of the
low-molecular-weight heparin, nor would fresh
frozen plasma be effective until the drug effect
was diminished.
Hematology and Oncology:Question 45
The correct answer is E
Educational Objectives
Begin appropriate initial therapy for idiopathic
thrombocytopenic purpura.
Critique
The presumptive diagnosis in this patient is
idiopathic thrombocytopenic purpura; no other
diagnostic studies are required to initiate
therapy. Prednisone, 1 mg/kg daily, would be
the most appropriate initial management.
Approximately 10% to 15% of patients may
have resolution of their disease or achieve
stable platelet counts (greater than 30,000/
μL) on a low dose of prednisone.
Splenectomy should not be routinely done if
the patient responds initially to prednisone.
Although the use of splenectomy in the
treatment of idiopathic thrombocytopenic
purpura remains in evolution, either
splenectomy or the use of other agents such as
intravenous immunoglobulin or intravenous
anti-Rh(D) could be considered if the patient
either does not respond to prednisone or
requires unacceptably high doses to maintain a
safe platelet count.
Therapy is indicated because the platelet count
is very low, and the presence of petechiae
suggests microvascular bleeding; therefore,
simple observation would not be appropriate.
Intravenous immunoglobulin and intravenous
anti-Rh(D) are both highly effective therapies
for idiopathic thrombocytopenic purpura, but
because of cost are generally reserved for
patients whose thrombocytopenia is severe or
who do not achieve a stable platelet count in
response to a low dose of prednisone (less
than 5 to 10 mg/d).
Hematology and Oncology:Question 46
The correct answer is C
Educational Objectives
Administer appropriate therapy for a
symptomatic distal deep venous thrombosis
after arthroscopic surgery.
Critique
The incidence of symptomatic venous
thrombosis after arthroscopic surgery is not
nearly as high as that after total knee
replacement. Arthroscopic surgery is, however,
a risk factor, and venographic studies have
shown an incidence of 4% to 17% after such
procedures. Symptomatic calf vein thrombosis
that has been objectively documented should
be treated with therapeutic doses of low-
molecular-weight heparin, if it is feasible to
treat the patient entirely in the outpatient
setting, followed by warfarin for a period of 3
months.
Approximately 20% of patients develop clot
extension if anticoagulation is not undertaken;
therefore it is not recommended to use anti-
inflammatory agents and to obtain a repeat
ultrasound in 1 week. There is no indication for
placement of an inferior vena cava filter, such
as inability to undergo therapeutic
anticoagulation or clot extension at a
therapeutic level of anticoagulation.
Enoxaparin, 30 mg subcutaneously every 12
hours, would be a prophylactic rather than a
therapeutic regimen for venous
thromboembolism in this setting.
Hematology and Oncology:Question 47
The correct answer is B
Educational Objectives
Select the proper plasmapheresis treatment for
thrombotic thrombocytopenic purpura
Critique
This patient has thrombotic thrombocytopenic
purpura and should be treated by
plasmapheresis with fresh frozen plasma. This
syndrome has been linked to disturbances in
the macromolecular structure of factor VIII,
caused by autoantibodies or enzyme defects.
Daily plasma exchange with fresh frozen
plasma allows an entire blood volume to be
replaced with normal plasma while avoiding
hypervolemia and is the standard of care for
this disorder. Therapy should be continued
until well after the hemolysis has subsided and
the lactate dehydrogenase level and platelet
count have returned to normal; relapses are
common if plasma exchange is discontinued
prematurely.
Plasmapheresis with the usual saline and
albumin replacement is not used for thrombotic
thrombocytopenic purpura. Thrombocytic
thrombocytopenic purpura is one of the few
disorders that requires fresh frozen plasma as
pheresis replacement, because the beneficial
effect is related to what is being given, in
addition to what is being removed. Platelet
transfusion as initial therapy in thrombotic
thrombocytopenic purpura has been reported
to worsen the microangiopathy, often with
fatal outcome.
Whether platelets can be safely used to treat
bleeding once a patient’s thrombotic
thrombocytopenic purpura has started to
respond to plasma exchange remains
controversial. Some hematologists believe that
corticosteroids used early in thrombotic
thrombocytopenic purpura can hasten
resolution, but only as an adjunct to fresh
frozen plasma infusion and plasmapheresis.

Hematology and Oncology:Question 48


The correct answer is B
Educational Objectives
Prevent venous thromboembolism after major
orthopedic surgery in a patient at high risk for
recurrent thrombosis.
Critique
Anticoagulation prophylaxis should be
continued for 6 weeks. Because this patient
had adequate therapy after a first deep venous
thrombosis, his risk of recurrence would be
quite low during the several weeks between
cessation of warfarin and the orthopedic
surgery, as long as he is reasonably active.
Therefore, “bridging” therapy with low-
molecular-weight heparin or intravenous
unfractionated heparin would be unnecessary
in the days immediately before surgery.
Administration of low-molecular-weight heparin
after knee replacement surgery for as long as
4 weeks has not been conclusively shown to
reduce the rate of deep venous thrombosis, as
compared with 7 to 10 days of therapy. The
patients risk of postoperative venous
thromboembolism is, however, greater than
average because of his previous episode.
Therefore it is recommended that
anticoagulant prophylaxis be continued for 6
weeks after surgery rather than just 1 week.
Aspirin is not a recommended agent for
prophylaxis of deep venous thrombosis after
major joint replacement.
Hematology and Oncology:Question 49
The correct answer is A
Educational Objectives
Recall the indications for red cell transfusion.
Critique
Except in the case of cardiovascular
compromise, most patients can compensate for
reductions in red cell mass through increased
stroke volume, myocardial contractility, and
other responses. Red blood cell transfusion is
indicated in order to raise the hematocrit of
this patient with cardiac disease.
Many physicians now allow patient
hemoglobins to drift to low levels, with an
absolute threshold around 7 g/dL as the point
where myocardial oxygen consumption is
impaired strictly by low red cell mass. If this
patient had not had cardiac disease, the red
cells would not have been indicated.
Increases in red cell mass will not improve
lung function, and may contribute to some
ventilation-perfusion mismatches. Most
patients are on ventilators because of airway
or alveolar dysfunction and not because of
poor oxygen transport. Hypoxia from a
pulmonary cause is not usually improved by
transfusion.
Transfusion of red cells can improve
hypotension and hypovolemia, but red cells are
not as efficient at increasing oncotic pressure
as colloids or plasma expanders. The risks of
donor exposure are not warranted simply to
increase circulating volume.
Hematology and Oncology:Question 50
The correct answer is C
Educational Objectives
Recognize the clinical manifestations of chronic
graft-versus-host disease.
Critique
Approximately 50% to 70% of the recipients of
allogeneic stem cell transplantation develop
some manifestations of chronic graft-versus-
host disease. Skin involvement is common,
with scleroderma, lichen planus, vitiligo,
scarring eczematoid rash, hyperpigmentation,
and hair loss. Anorexia, malabsorption, weight
loss, diarrhea, and abdominal pain are
predominant gastrointestinal manifestations of
the disease. Biopsy of the rectal or colonic
mucosa may be helpful in confirming the
diagnosis and eliminating other potential
causes such as cytomegalovirus colitis.
Negative biopsy results do not rule out
underlying graft-versus-host disease, and
symptomatic patients should be treated with
corticosteroids empirically.
Pseudomembranous colitis is uncommon with
trimethoprim-sulfamethoxazole therapy.
Laxative abuse is unlikely and does not explain
skin changes. Shigella, often found in stool
culture, is associated with bloody diarrhea.
Recurrence of chronic myeloid leukemia would
not cause diarrhea.
Hematology and Oncology:Question 51
The correct answer is C
Educational Objectives
Treat venous thrombosis associated with
transient risk factors.
Critique
Although this patient has protein S deficiency
and a strong positive family history of venous
thromboembolism, her first thrombotic event
occurred in association with several transient
risk factors; these include pregnancy and a
caesarean section. Therefore, despite her
protein S deficiency, it is appropriate to
discontinue oral anticoagulation therapy after 3
to 6 months at a target INR of 2 to 3 and
employ it prophylactically in high-risk
situations only. Neither low fixed doses of
warfarin nor the combination of aspirin and
clopidogrel has been shown to be efficacious
for the prevention of recurrent venous
thromboembolism.
Hematology and Oncology:Question 52
The correct answer is C
Educational Objectives
Recognize and differentiate myelodysplastic
syndrome from other causes of anemia.
Critique
Myelodysplastic syndromes are clonal
hematopoietic stem cell disorders that occur
predominantly in patients older than 50 years.
They are characterized by peripheral blood
cytopenias and dysplasia of erythroid,
granulocytic, or megakaryocytic lineages. The
mean corpuscular volume is commonly
elevated. Although a history of diverticulosis
and aspirin use suggests possible bleeding and
iron deficiency, the low leukocyte and platelet
counts, high mean corpuscular volume, and
findings on peripheral blood smear do not
support a diagnosis of iron deficiency anemia.
Angiotensin-converting enzyme (ACE)
inhibitors are rarely associated with blood
dyscrasia, and the findings described do not
occur with drug-induced cytopenias.
Vitamin Bi 2 deficiency can cause all the
laboratory abnormalities described except
reduced segmentation of neutrophils. By
contrast, there is hypersegmentation in
vitamin Bi 2 deficiency. Myelofibrosis is
commonly associated with splenomegaly.
Hematology and Oncology:Question 53
The correct answer is A
Educational Objectives
Recognize the presentation of a bacterial
contamination reaction.
Critique
Bacterial contamination of platelets can occur
when skin plugs enter the blood during needle
insertion or because of asymptomatic donor
bacteremia. It is most common in platelet
concentrates that are 4 to 5 days old—long
enough for log-phase growth to occur in the
inoculum. The symptoms are due to the
pyrogens associated with the bacteria, toxins
elaborated during their growth, and the
recipient’s inflammatory response. This
patients fever, the fact that he has no
respiratory distress or wheezing, and the time
sequence make anaphylaxis unlikely as a cause
of his symptoms. Group AB individuals have no
ABC isoagglutinins and are therefore
“universal” plasma/platelet donors. Febrile,
nonhemolytic reactions from antibodies to
donor leukocyte antigens are common but
rarely cause hypotension and vascular
collapse; leukocyte reduction prevents or
alleviates these reactions.
Passive acquisition of donor leukoagglutinins
causes transfusion-related acute lung injury,
but respiratory distress and hypoxemia are the
hallmarks of this reaction, not fever and
hypotension.
Hematology and Oncology:Question 54
The correct answer is D
Educational Objectives
Determine a schedule by which to alter a
patient’s anticoagulation therapy.
Critique
It would be best to initiate warfarin at 5 mgld,
and discontinue argatroban when the INR is
greater than 4.0. How to alter a patients
anticoagulation therapy from argatroban to
warfarin has been a confusing issue to
clinicians. Argatroban, although it is monitored
by using the partial thromboplastin time,
affects the INR even when used alone. Both
argatroban and warfarin prolong the INR; a
schedule that called for argatroban to be
discontinued when the INR reached 2.0 would
not allow enough time for the desired
anticoagulation effect of warfarin to be
achieved. Therefore, it is recommended that
argatroban be discontinued only after the INR
is greater than 4.0. The INR should then be
rechecked 4 to 6 hours after argatroban is
stopped. If it is not in the therapeutic range of
2 to 3, argatroban should be restarted and
continued until the INR attrributable to
warfarin alone is in the therapeutic range.
Some experts also recommend that argatroban
be continued for at least 3 days after starting
warfarin, to ensure that a full anticoagulant
effect of warfarin is achieved before argatroban
is discontinued. It is also recommended that
the argatroban infusion be reduced to 2 μg/kg
perminute before warfarin is initiated.
Discontinuing argatroban at the time warfarin
is initiated, or 24 hours before warfarin
initiation, would leave the patient without
anticoagulation therapy for 3 to 4 days, the
amount of time required for the anticoagulant
effect of warfarin to be achieved. Furthermore,
it would expose the patient to the transient
procoagulant effect of warfarin that can occur
soon after therapy is initiated (owing to a rapid
fall in the levels of protein C, the vitamin K-
dependent coagulation inhibitor).
A loading dose of warfarin is unnecessary; it
does not hasten the achievement of a stable
therapeutic INR any more than does therapy
initiated at a dose of 5 mg/d. Furthermore,
initiation of therapy at high doses is associated
with an increased incidence of bleeding
complications.
Hematology and Oncology:Question 55
The correct answer is B
Educational Objectives
Determine the appropriate duration of
anticoagulant therapy for a patient with a
postoperative deep venous thrombosis.
Critique
The appropriate period of anticoagulation for a
symptomatic post-operative deep venous
thrombosis is 3 months. Although low-
molecular-weight heparin is effective in
reducing the rate of deep venous thrombosis
after total hip replacement, the incidence
remains at 10% to 20% as assessed by
bilateral venography after 7 to 10 days of
therapy. The rate increases to 25% to 30% at
1 month if extended prophylaxis is not
employed. Therapy for 6 weeks will result in a
higher recurrence rate than therapy that lasts
3 to 6 months. Clearly, this patient’s
thrombosis was provoked by the joint
replacement surgery. Therefore the recurrence
risk will be quite low, and anticoagulation for 1
year or for the rest of her life is not warranted.

Hematology and Oncology:Question 56


The correct answer is E
Educational Objectives
Diagnose platelet refractoriness.
Critique
The refractoriness observed in this patient is
most probably caused by alloimmunization.
Approximately 30% of pregnancies are
associated with low levels of HLA
alloimmunization, presumably through
stimulation from placental tissue as well as
unapparent fetal-maternal hemorrhage. These
HLA antibodies often decay to subdetectable
levels quickly, but the womans immune system
retains the ability to form an anamnestic
response when rechallenged.
Just like patients who have received many
transfusions, multiparous women are at risk of
having a high level of circulating HLA antibody.
Although these alloantibodies do not
themselves cause thrombocytopenia (because
they do not destroy autologous platelets), they
are a major cause of platelet refractoriness.
When immune refractoriness exists,
unmatched platelet transfusions are usually
ineffective, and the patient requires specially
ordered matched platelets, usually obtained
from a single donor through apheresis.
This patients catastrophic hemorrhage is a
nonimmunologic complication of pregnancy.
The post-transfusion platelet increment was
less than 25% of the expected rise for
someone of average body size and blood
volume. The best discriminator of immune and
nonimmune platelet destruction is the
measurement of the immediate post-
transfusion platelet count. Immune destruction
occurs quickly, often within minutes of a
transfusion and many times without evidence
of a transfusion reaction. Nonimmune causes
do not usually lower the platelet increment
until well after 1 hour post-transfusion. The
observed poor platelet count increment in this
patient suggests platelet transfusion
refractoriness, which can have multiple causes.

Although moderate thrombocytopenia can


develop through loss of platelets during
vigorous bleeding and through the dilutional
effect of massive transfusion, these
mechanisms usually do not cause transfusion
refractoriness. Mild thrombocytopenia of
nonimmune origin can occur in many
pregnancies, but the patients normal platelet
count prior to delivery rules out this
complication.
Disseminated intravascular coagulation can
lower platelet counts, but almost always
causes abnormalities in the tests of clotting
function. In the HELLP (hemolysis, elevated
liver enzymes, and low platelet count)
syndrome, the low platelet count is evident
before delivery.
Hematology and Oncology:Question 57
The correct answer is B
Educational Objectives
Diagnose and treat iron overload
Critique
The most effective procedure to assess the
possibility of iron overload in this patient is a
diagnostic liver biopsy. The high serum indirect
bilirubin and low serum haptoglobin levels in
association with the normal reticulocyte count
are evidence of ineffective erythropoiesis. The
elevated serum ferritin concentration and high
transferrin saturation levels are consistent with
iron overload. Given the patients ancestry and
clinical picture, he could have iron overload
related to 3-thalassemia intermedia or
sideroblastic anemia.
Homozygosity for the C282Y mutation in the
HFE gene occurs in approximately 1 in 200
persons of northern European ancestry and is
the most common cause of iron overload in
whites. Mutations in other genes (TfR2, FPN 1)
that lead to hereditary iron overload have
recently been reported. Dietary iron overload is
common in blacks from rural southern Africa
and may have a genetic component.
Nontransfusional iron overload also occurs in
certain anemias characterized by high degrees
of ineffective erythropoiesis. Such anemias
include β-thalassemia major and intermedia,
hemoglobin H disease, β-
thalassemia/hemoglobin E, and hereditary or
acquired sideroblastic anemias.
A quantitative phlebotomy program is an
alternative way to document iron overload but
would be difficult to perform in an anemic
patient and does not inform about the
presence of iron-related hepatic damage.
Staining a bone marrow aspirate for iron is the
gold standard for diagnosing iron deficiency
but is not reliable for diagnosing iron overload.
Treatment with desferrioxamine should be
instituted only in an anemic patient with
confirmed iron overload, because the toxicity
of the agent is markedly increased if iron
stores are not increased.
Hematology and Oncology:Question 58
The correct answer is E
Educational Objectives
Treat chronic myeloid leukemia in an elderly
patient.
Critique
This patients presentation, examination
findings, and laboratory test results are
consistent with chronic-phase chronic myeloid
leukemia (CML). Demonstration of a balanced
translocation between chromosomes 9 and 22
(Philadelphia chromosome) confirms the
diagnosis.
Despite her advanced age and multiple medical
problems, this patient can be successfully
treated with imatinib mesylate (Gleevec). In
90% of patients treated with a daily oral dose
of imatinib mesylate, blood counts become
normal, and 30% to 40% of the patients no
longer have detectable CML cells in their bone
marrow.
Transient leukopenia, mild nausea, and muscle
cramps are common side effects of therapy
with imatinib mesylate. Rash, hepatotoxicity,
and fluid retention may also occur. The optimal
duration of therapy, long-term benefits, and
toxicity of imatinib mesylate are under
investigation. Without therapy, this patient is
at risk of developing hyperleukocytosis and
circulatory compromise, progressive
splenomegaly, metabolic complications, and
infections.
Because there is no evidence of progression to
acute leukemia, there is no need for systemic
induction chemotherapy.
Allogeneic bone marrow transplantation would
not be an appropriate treatment option,
especially given the patient’s age and history
of cardiac problems. However, interferon-alfa
therapy would be poorly tolerated and would
not be a good choice for this patient. Daily
subcutaneous interferon-alfa injections are
more effective in decreasing CML cells in the
bone marrow, and if not discontinued because
of toxicity can result in 5-year survival rates of
50% to 70%. Hydroxyurea is commonly used
to normalize blood counts quickly and results
in a 5-year survival of 35% to 50%.
Hydroxyurea is well tolerated and could be an
alternative therapy for this patient, particularly
if the cost of imatinib mesylate is prohibitive.

Hematology and Oncology:Question 59


The correct answer is B
Educational Objectives
Recognize and initiate therapy for the HELLP
syndrome
Critique
This patient has the HELLP (hemolysis,
elevated liver function tests, low platelets)
syndrome. Some authorities consider it a
variant of preeclampsia, because
approximately 10% of patients with HELLP
syndrome ultimately meet criteria for
preeclampsia. As this case illustrates, however,
a diagnosis of HELLP syndrome does not
require hypertension and proteinuria. As with
preeclampsia, definitive therapy for HELLP
syndrome should be focused toward urgent
delivery of the fetus, after which the disorder
usually resolves.
There is no role for intravenous
immunoglobulin in management of HELLP
syndrome or preeclampsia. Plasma exchange is
indicated for thrombotic thrombocytopenic
purpura, which must be considered in the
differential diagnosis of this patient’s disorder.
Her presentation and laboratory studies,
however, are most consistent with the HELLP
syndrome.
There is no role for aggressive diuresis or
antiplatelet agents in the management of
established preeclampsia or HELLP syndrome.
Randomized trials of prophylactic aspirin to
decrease the incidence of preeclampsia have
not shown a benefit to treatment, and
antiplatelet agents may also increase the
incidence of thrombocytopenic bleeding
complications.
Hematology and Oncology:Question 60
The correct answer is E
Educational Objectives
Recognize myelofibrosis.
Critique
Myelofibrosis is characterized by splenomegaly,
hepatomegaly, normocytic anemia, circulating
erythroblasts and myeloid precursors, teardrop
cells, and marked bone marrow fibrosis. The
splenomegaly and hepatomegaly result from
extramedullary hematopoiesis. Other
myeloproliferative syndromes have to be
excluded in order to ascertain the diagnosis.
Although this patient has circulating myeloid
precursors, absence of an elevated leukocyte
count and FISH analysis that is negative for
t(9;22) exclude chronic myeloid leukemia.
Hairy cell leukemia can present with
pancytopenia, splenomegaly, and dry bone
marrow aspiration. Peripheral blood smear in
this disorder shows characteristic hairy
mononuclear cells, and leukoerythroblastosis is
uncommon.
Patients with myelodysplastic syndromes can
have varying degrees of marrow fibrosis, but
massive splenomegaly and hepatomegaly are
uncommon. There are typically severe
dysplastic abnormalities in one or more
lineages in blood and marrow of patients with
myelodysplastic syndrome. Disseminated
tuberculosis is also associated with marrow
fibrosis. However, caseating or noncaseating
granulomas are typically observed on bone
marrow biopsy.
Hematology and Oncology:Question 61
The correct answer is B
Educational Objectives
Diagnose an allergic transfusion reaction.
Critique
This patient is having an acute anaphylactic
transfusion reaction. The presence of mucosal
edema, bronchospasm, and hypotension make
it more than a severe systemic allergic
reaction.
Bacterial contamination can lead to respiratory
distress and shock, but fever is often present.
Furthermore, because fresh frozen plasma is
stored frozen and used within hours of
thawing, there is insufficient time for any
contaminating bacteria to grow to a sufficient
inoculum size.
ABC-incompatible acute intravascular
hemolytic reactions can cause symptoms like
those of this patient, even before overt
hemolysis is evident. The units given were
group AB; therefore ABC plasma
incompatibility cannot be the cause. Fresh
frozen plasma contains virtually no intact red
cells; therefore the recipient’s naturally
occurring anti-B has no cellular target in the
group AB plasma.
The hallmark of transfusion-related acute lung
injury is hypoxemia from pulmonary
leukoagglutination and alveolitis rather than
bronchospasm and edema, and allergic
reactions like this patients urticaria and rash
are uncommon. Patients taking angiotensin-
converting enzyme inhibitors may have sudden
acute hypotension due to disordered
bradykinin metabolism, but these reactions are
quick to resolve and cause few other
symptoms.
Hematology and Oncology:Question 62
The correct answer is B
Educational Objectives
Recognize the risk posed by the prothrombin
G20210A mutation.
Critique
The prothrombin G2021 0A gene mutation is a
definite risk factor for an initial episode of deep
venous thrombosis, and she should encourage
her first-degree relatives to be tested for the
mutation.
The incidence of recurrence in patients with a
first unprovoked episode of deep venous
thrombosis after adequate treatment (3 to 6
months of anticoagulation at therapeutic
doses) is 5% to 15% per year. Heterozygosity
for the prothrombin gene mutation is a definite
risk factor for an initial episode of deep venous
thrombosis in whites; most studies, however,
have not shown the recurrence risk to be any
greater in patients with an identifiable
thrombophilic defect than in those without one.
Lifelong anticoagulation is generally not
undertaken for an uncomplicated first episode
of deep venous thrombosis, although recent
data from the PREVENT trial indicate that
anticoagulation at a target INR of 1.5 to 2 will
provide significant protection against recurrent
venous thromboembolism without a significant
risk of major bleeding.
Hematology and Oncology:Question 63
The correct answer is D
Educational Objectives
Identify patients with idiopathic
thrombocytopenic purpura in whom anti-Rh(D)
therapy is not appropriate.
Critique
This patient has idiopathic thrombocytopenic
purpura that has proved refractory to
corticosteroid therapy and splenectomy. The
use of intravenous anti-Rh(D) would not be
indicated in this patient. Although the
mechanisms of this agent in idiopathic
thrombocytopenic purpura are complex, it is
believed to act largely by blocking uptake of
antibody-coated platelets by Fc receptors in
the spleen. It is not efficacious in
splenectomized patients.
Initial therapy of idiopathic thrombocytopenic
purpura with corticosteroids leads to a
remission rate of 70% to 80%, although
ultimately only 15% to 25% of these
remissions can be sustained on an acceptable
dosage of corticosteroids. Second-line therapy
of idiopathic thrombocytopenic purpura may
include intravenous immunoglobulin, anti-
Rh(D), splenectomy, or a number of other
approaches. Splenectomy induces remission in
approximately 80% of such patients, although
relapses, most of which occur in the first year
after splenectomy, reduce the number of long-
term responders to approximately 65%.
Intravenous immunoglobulin is an acceptable
agent to use in refractory idiopathic
thrombocytopenic purpura after splenectomy
and probably the most likely of the agents
mentioned to induce a response. Responses to
intravenous immunoglobulin, however, are
most often transient. Rituximab, an anti-CD20
monoclonal antibody, is associated with a
response rate of 25% to 50% in patients with
refractory idiopathic thrombocytopenic
purpura. Danazol and cyclophosphamide are
also of benefit in some patients, but their
effect may require several weeks to become
apparent.
Hematology and Oncology:Question 64
The correct answer is B
Educational Objectives
Recognize the late complications of breast
cancer treatment.
Critique
This patients previous radiation therapy is the
likely cause of her fibrosarcoma. Therapeutic
radiation is associated with a 1% per year risk
of a second solid tumor within the radiated
field for at least 25 years after treatment. It is
not clear whether the risk diminishes with
longer time periods after therapy. Patients who
receive therapeutic radiation, even in very
small total doses, are at increased risk, and
the radiation field should be carefully examined
regularly each year after treatment.
People cured of cancer remain at risk for other
cancers. Second (and third) metachronous
(that is, occurring at another time) cancers can
develop from a variety of causes.
Approximately 15% of patients who are cured
of a first cancer by a surgical procedure (no
chemotherapy or radiation therapy) develop a
second neoplasm in another organ. Typically
the majority of these cases involve two
common forms of cancer, for example, colon
and prostate cancer in men or colon and breast
cancer in women.
Fibrosarcoma is a rare tumor; therefore, it is
unlikely to be completely unrelated to the
breast cancer and its treatment. Some tumors
occur in distinct organs but have very similar
risk factors; for example, head and neck
cancer patients are at increased risk of lung
cancer and both kinds of tumor occur with
increased frequency in tobacco and alcohol
abusers. Fibrosarcoma does not share risk
factors with breast cancer; therefore, similar
risk factors do not account for these
metachronous cancers.
Tamoxifen is associated with an increased risk
of endometrial cancer and a decreased risk of a
second breast cancer and colon cancer. It is
not known to cause fibrosarcoma. Some
organs are susceptible to multiple
metachronous tumors, such as bladder cancer
and breast cancer. Fibrosarcoma is not a tumor
that routinely occurs as multiple metachronous
tumors.
Hematology and Oncology:Question 65
The correct answer is C
Educational Objectives
Confirm staging test results before precluding
surgery for non-small-cell lung cancer.
Critique
It is first necessary to determine whether the
nodules in the liver are benign or malignant by
percutaneous needle biopsy. Surgery with
curative intent should be offered if the non-
small-cell lung cancer appears localized to the
area near the left hilum; it would be obviated if
the nodules in the liver were found to be
malignant.
Radiation therapy alone would be inappropriate
if liver metastases were confirmed because
palliative chemotherapy would be the preferred
approach. Chemotherapy would be
inappropriate if the liver nodules are benign
(outside of a clinical trial), because it would
deny or delay potentially curative surgery.
Measuring serum carcinoembryonic antigen
(CEA) would not be helpful in any case; CEA
may be elevated in localized or metastatic lung
cancer.
The positron emission tomography scan was
unhelpful because it did not indicate abnormal
uptake in the liver nodules. The false-negative
rate of positron emission tomography scans in
this setting is not yet known with certainty,
especially because faint activity can be
interpreted as positive or negative, depending
on the radiologist.
Hematology and OnCology:Question 66
The correct answer is A
Educational Objectives
Recognize modifiable risk factors related to
cancer of the head and neck.
Critique
The patient should avoid excessive use of
alcohol, a factor that has been linked to
squamous cell carcinoma of the head and neck.
Tobacco and alcohol abuse appear to be
synergistic as causative agents for head and
neck cancer.
Benzene exposure is known to be a causative
factor for leukemias, but is not known to cause
lung or head and neck cancers. Daily vitamin
supplementation may improve other health
outcomes, but there is no evidence that this
strategy will reduce cancer risk. There is no
evidence that supplementation with β-carotene
has any effect on the risk of head and neck
cancer.
Hematology and Oncology:Question 67
The correct answer is D
Educational Objectives
Recognize the effects of radiation therapy in
patients treated for clinically localized prostate
cancer.
Critique
Radiation therapy to the prostate gland has
significant morbidity; for example, it is
associated with acute proctitis in 18.7% of
patients.
Incontinence is more common with surgically
treated patients (severe urinary incontinence
in 9.6 % vs. 3.5 %), and impotence is common
in both groups. Thrombosis occasionally occurs
in the postoperative period, and it is not
commonly related to radiation therapy.
Hematology and Oncology:Question 68
The correct answer is D
Educational Objectives
Make an appropriate referral for a patient with
potentially resectable pancreatic cancer.
Critique
This case is a classic presentation of pancreatic
cancer. CT results indicate that the patients
tumor may be resectable. The best option is
referral of the patient to a center with surgical
expertise in the management of pancreatic
cancer, because surgical resection offers the
only potential for long-term survival.
Futhermore, operative mortality is significantly
lower at a center where many such procedures
are performed than at centers with limited
experience.
Because the patient does not have
symptomatic jaundice, emergency biliary
drainage is not indicated. Establishing a
definitive histologic diagnosis is not necessary
before the referral, in a patient with potentially
resectable disease.
Hematology and OnCology:Question 69
The correct answer is A
Educational Objectives
Recognize which screening tests are associated
with the strongest evidence of reducing the
mortality rate in cancer
Critique
All three randomized trials of biennial fecal
occult blood testing (FOBT) reported to date
have shown a decrease in colorectal cancer
mortality with screening compared with
controls. One of the trials had an annual
screening arm, which showed an even greater
reduction in the number of deaths from
colorectal cancer (Mandel et al). The reduction
appears to be similar whether or not FOBT
slides are rehydrated, although the false-
positive rate of the tests is increased by
rehydration.
In one of the three trials of biennial screening,
it was estimated that approximately five to ten
deaths from colorectal cancer would be averted
for every serious complication or postoperative
death from the screening-treatment process
(Robinson et al). It is uncertain whether FOBT
decreases the incidence as well as the
mortality from colorectal cancer; results have
been inconsistent.
Most North American professional
organizations recommend the use of
sigmoidoscopy in screening for colorectal
cancer every 5 years. The recommendation is
supported by well-performed case-control
studies that show an association between
sigmoidoscopy within the previous 10 years
and lower risk of death from colorectal cancer.
Because this study design relies on prior choice
to undergo the procedure and people who
choose to undergo screening tests may have
better underlying health than those who do
not, its findings represent an intermediate
strength of evidence.
Furthermore, case-control study designs do not
allow for a direct comparison of benefits and
harms such as perforation or bleeding. A
federally sponsored randomized trial of
sigmoidoscopy versus a control arm in more
than 150,000 volunteer participants is
therefore being conducted, but results are
unlikely to be known for several years.
To date, the effects of human papillomavirus
(HPV) DNA testing and Pap smears have never
been tested in a randomized trial. In the case
of Pap smears, however, virtually every other
study design has shown a substantial decrease
in the incidence of cervical cancer as well as
mortality associated with institution of Pap
smears. Because Pap smears target
preneoplastic lesions of the cervix, the test is
in large measure a form of primary prevention,
aimed at lowering the incidence of invasive
cancer. Pap smears do detect a large number
of noninvasive lesions such as low-grade
squamous cell intraepithelial lesions and
atypical squamous cells of unknown
significance, whose natural history is ill
defined. A randomized study is ongoing to
determine if some of these low-grade lesions
can be monitored without definitive therapy,
and if HPV testing can aid in the decision.
In a randomized trial of chest radiography plus
sputum cytology every 4 months in male
cigarette smokers, there was no decrease in
the rate of death from lung cancer, even
though more operable lung cancers were
detected and survival time from diagnosis of
lung cancer was increased in the screened
group. These observations are most likely due
to overdiagnosis—detection of non-life-
threatening cancers—by the screening tests.
Therefore, no major organizations recommend
routine screening for lung cancer, even of
current or former cigarette smokers.

Hematology and OnCology:Question 70


The correct answer is A
Educational Objectives
Evaluate adenocarcinoma presenting in an
isolated axillary lymph node.
Critique
In a young, nonsmoking woman with
adenocarcinoma of the axilla, breast cancer
remains the most likely diagnosis. Optimal
therapy for breast cancer should be initiated,
especially because breast cancer is the most
treatable type of adenocarcinoma (even with
curative intent in this case). Options include
modified radical mastectomy, which identifies
an occult primary tumor 30% to 50% of the
time, or radiation therapy for breast
conservation. In either case, adjuvant
chemotherapy would be recommended. The
tissue from biopsy should be stained for
HER2/neu; positive results would confirm the
diagnosis of breast cancer.
Although use of radiation therapy is certainly
one option, it is not the only option, and some
patients and clinicians might prefer modified
radical mastectomy. Proceeding with resection
alone would not be advised in such a young
patient to whom potentially curative options
are available.
Hematology and Oncology:Question 71
The correct answer is A
Educational Objectives
Evaluate a breast mass.
Critique
One of the most common causes of
malpractice in the United States is failure to
diagnose breast cancer at an early stage.
Several studies have demonstrated that
prognosis and the degree of therapy required
are both directly related to the size and stage
at diagnosis, which are in turn directly related
to early or delayed detection. Ultrasonography
is the procedure of choice for this patient. It
can be very helpful in evaluating a palpable
abnormality of the breast, frequently
increasing suspicion or providing evidence of
benign conditions.
The sensitivity of mammography is 75% to
90%; therefore, a negative mammogram is
insufficient reason to discontinue thorough
evaluation of a breast abnormality. No
identifiable risk factor (other than being female
and over 40 years of age) can be found in 75%
of all patients with newly diagnosed breast
cancer. MRI of the breast is probably more
sensitive than mammography, but the
specificity of MRI is quite low and this
technique is reserved for special
circumstances, such as evaluation of a patient
who has no evidence of breast cancer but has
axillary lymph nodes containing
adenocarcinoma. It is also used in the
screening of women with BRCA-1 or BRCA-2
germ-line abnormalities. Measurement of
circulating tumor markers, although helpful in
selected patients with metastatic disease, is
insufficiently sensitive or specific to be of use
in evaluating a suspicious breast mass.
Hematology and Oncology:Question 72
The correct answer is A
Educational Objectives
Treat febrile neutropenia in a patient with
cancer.
Critique
A thorough re-assessment should be
performed to determine the causative agent.
Because this patient’s condition is stable, and
the neutrophil count is increasing, it is
reasonable to continue the same therapy. For
this reason, adding vancomycin or removing
the central venous port is not indicated.
Although often used in clinical practice, colony-
stimulating factors are not recommended
because results of randomized trials have
failed to demonstrate consistent improvements
in the duration of hospitalization, duration of
antibiotic therapy, fever, total cost for
managing the febrile neutropenic episode, or
infection-related mortality. The presence of a
central venous port removes this patient from
the low-risk category; otherwise, his disease
might be treated with oral antibiotics.
Intravenous ceftazidime remains the best
choice.
Hematology and Oncology:Question 73
The correct answer is B
Educational Objectives
Recognize the long-term complications of
mediastinal radiation therapy.
Critique
The most likely diagnosis is exercise-related
myocardial ischemia in this young man. The
delivery of therapeutic doses of radiation
therapy to the anterior mediastinum includes
large segments of the coronary arteries, and
the radiation exposure promotes premature
atherosclerosis. The Stanford group
documented a threefold increase in the rate of
fatal myocardial infarction in their patients
receiving mantle-field radiation therapy. Thus,
it is important that patients who receive
mantle-field radiation therapy as a component
of their treatment for Hodgkin’s disease be
informed of the increased risk of myocardial
infarction and counseled about the importance
of maintaining a healthy weight and lowering
cholesterol with a statin as prophylaxis.
Recurrent Hodgkins disease is exceedingly
unlikely because the disease rarely recurs this
late after diagnosis. Half of all cases of relapse
of Hodgkin’s disease occur within the first year
of completing treatment, and nearly all the rest
occur 1 to 5 years after treatment.
Furthermore, Hodgkins disease usually recurs
in previously involved sites, particularly sites of
bulky disease. Therefore, recurrent disease is
most likely to be detected by the development
of B symptoms according to the Ann Arbor
criteria (unexplained fever, drenching night
sweats in the previous month, unexplained
weight loss of more than 10%) or dyspnea on
exertion from a mediastinal recurrence that
limits lung function, not by myocardial
ischemia.
Pulmonary fibrosis can occur in patients
treated with bleomycin plus radiation therapy.
The presenting signs of pulmonary fibrosis,
however, are dyspnea on fibrosis and
shortness of breath.
Secondary marrow failure is so rare after
doxorubicin, blemoycin, vinblastine, and
dacarbazine plus radiation therapy that any
case would be reportable. Furthermore,
presenting symptoms would include pallor,
fatigue, and exercise intolerance.
Constrictive pericarditis is also unlikely given
the time period (15 years ago) of this mans
treatment. Twenty to 30 years ago, some
centers were administering mantle-field
radiation therapy with anteriorly weighted
ports such that two thirds of the radiation was
administered to the front and one third to the
back. This method produced a high rate of
constrictive pericarditis that has since been
dramatically reduced by administering the
radiation therapy in equal doses front and
back. The onset of constrictive pericarditis
from radiation therapy in this setting was
usually insidious and associated with
progressive exercise intolerance.
Hematology and Oncology:Question 74
The correct answer is C
Educational Objectives
Recognize androgen ablation as the
cornerstone of treatment in metastatic disease
of the prostate.
Critique
Since the 1940s, androgen ablation has been
the cornerstone of treatment in metastatic
prostate cancer. Although the timing of the
commencement of therapy is somewhat a
matter of controversy, most authorities agree
that symptomatic patients should be treated
with androgen ablation. Chemotherapy may
palliate this patient’s bone pain and fatigue.
Chemotherapy with docetaxel or mitoxantrone,
corticosteroids, and radiation therapy may be
used in the palliation of advanced disease.
Outside of a clinical trial, however, such
therapy should be used only after disease
progression when the patient is receiving
androgen blockade.
Hematology and Oncology:Question 75
The correct answer is B
Educational Objectives
Treat rectal cancer with adjuvant
chemoradiation therapy.
Critique
This patient has stage II rectal cancer. Because
the tumor has penetrated the bowel wall, she
is at risk of developing a local recurrence, even
though no lymph nodes are involved. Adjuvant
therapy involving systemic chemotherapy and
concurrent pelvic radiation therapy have been
shown to reduce the risk of local recurrence
and increase survival.
Postoperative radiation therapy alone is inferior
to the combination of chemotherapy and
radiation therapy. Although regular
colonoscopy is a reasonable surveillance
strategy, adjuvant combined-modality therapy
should be offered to patients with stage II and
III (regional lymph node involvement) disease
who otherwise have good performance status.
Hematology and Oncology:Question 76
The correct answer is C
Educational Objectives
Recognize the relative risks and benefits of
hormone replacement therapy (HRT) and
tamoxifen to
women at increased risk for breast cancer.
Critique
This patient should stop taking the HRT and
start taking tamoxifen. Her risk of developing
breast cancer is substantially elevated. It
would be reasonable to encourage her to
participate in a clinical trial for breast cancer
prevention, such as the STAR (Study of
Tamoxifen and Raloxifene) trial.
The breast cancer risk criterion that qualified
women for the Breast Cancer Prevention Trial
(BCPT), a placebo-controlled randomized trial
of tamoxifen, was an absolute risk of at least 1
.66% over the next 5-year period. The BCPT
showed that administration of tamoxifen for as
long as 5 years decreased the relative risk of
breast cancer by approximately 50%,
representing an absolute risk of approximately
5.2% over the next 5 years for this patient.
In order to enter the BCPT study, women had
to have stopped taking HRT at least 3 months
before randomization (and remain off it for the
duration of the trial). Therefore, the efficacy of
tamoxifen for the prevention of breast cancer
is not known for women who are concurrently
being treated with HRT. The risk of ischemic
heart disease in the BCPT was similar in the
tamoxifen and placebo arms, although the
total number of ischemic heart disease events
was small. (In therapeutic studies of tamoxifen
for established breast cancer, cardiac disease
has also been similar in the treatment and
control groups.)
Use of estrogen plus progestin HRT does,
however, appear to increase the risk of
ischemic heart disease in postmenopausal
women. In the Women’s Health Initiative
(WHI) trial, which studied the use of estrogen
plus progestin by healthy postmenopausal
women, the relative risk of coronary heart
disease was elevated by 29% in the HRT arm.
In that trial, HRT also increased the relative
risk of breast cancer by 26%. The study was
therefore halted because the harms of
progestin plus estrogen were believed to
outweigh the benefits in the preventive setting.

The effect of estrogen alone on heart disease is


still under study in the WHI trial, but estrogen
alone is not generally given to women with an
intact uterus, because it causes endometrial
cancer.
Hematology and Oncology:Question 77
The correct answer is D
Educational Objectives
Recognize the potential emergency of spinal
cord compression in a cancer patient with new-
onset back pain.
Critique
Spinal cord compression must be ruled out in a
patient with cancer who presents with back
pain of new onset, and MRI is currently the
diagnostic test of choice. Nondiagnostic plain
radiographs do not rule out the diagnosis, nor
do bone scans, in which increased uptake is
dependent on either an osteoblastic reaction or
new bone deposition. Because the rate of
progression cannot be accurately predicted,
the MRI should be obtained promptly.
CT scanning might be helpful in determining
the degree of bone destruction and whether
bone or tumor is impinging on the cord, but it
is not the procedure of choice in this situtation.

Most patients with spinal cord compression


have pain that is localized to the spine or
radicular in nature. Left untreated, spinal cord
compression associated with malignancy can
progress to myelopathy over a period of
weeks. Therefore, it would not be prudent to
watch and wait under these circumstances.
Because the patient does not have any
neurologic symptoms, there is no immediate
need to administer dexamethasone before
establishing the diagnosis.
Hematology and Oncology:Question 78
The correct answer is E
Educational Objectives
Diagnose intestinal obstruction in a patient
treated for rectal cancer.
Critique
The patient presents with signs of intestinal
obstruction, and the most urgent need is to
define the cause. Because the distal colon and
rectum are not filled with stool, rectal stricture
is unlikely. Possibilities include either
malignant obstruction (peritoneal
carcinomatosis, for example, or metachronous
primary tumor) or nonmalignant obstruction
(caused by radiation enteritis or adhesions). CT
scanning is the most useful diagnostic study to
distinguish among the possibilities. It is more
sensitive and specific than a gastrointestinal
series and shows more of the intra-abdominal
anatomy.
Because colonoscopy done 1 year earlier was
unremarkable, and the current radiographic
studies suggest that the obstruction is at the
level of the small bowel, another colonoscopy
is unlikely to be helpful. The patient does not
have signs of an acute abdomen; therefore a
surgical consultation can be deferred until
additional diagnostic studies have been done to
identify the cause of the obstruction. The
results of a carcinoembryonic antigen test
would not be available for a few days, and
would have little influence on management of
this case.
Hematology and Oncology:Question 79
The correct answer is E
Educational Objectives
Appropriately monitor a patient taking
tamoxifen.
Critique
Women taking tamoxifen should be carefully
questioned about abnormal gynecologic
symptoms such as bleeding and discharge and
given a careful pelvic examination, reserving
diagnostic work-up for those with symptoms or
physical abnormalities.
Tamoxifen taken to prevent breast cancer
increases the risk of endometrial cancer
approximately two- to threefold in
postmenopausal women with an intact uterus;
the absolute risk is approximately 2 to 3 per
1000 women per year. Most of the endometrial
cancers diagnosed in these women involve
such symptoms as dysfunctional uterine
bleeding or discharge, and most are localized
(International Federation of Gynecology and
Obstetrics stage I) and surgically curable.
Approximately 40% of women taking
tamoxifen will develop endometrial thickening
as assessed by ultrasound examination. The
false-positive rate of ultrasononography of the
uterus is high in asymptomatic women,
however, leading to a high rate of invasive
testing and attendant risk of uterine
perforation. The diagnostic yield of routine
endometrial cytologic sampling or biopsy for
the detection of early endometrial cancer in
asymptomatic women taking tamoxifen is too
low to warrant the pain and risk of the
procedure. Therefore, despite the increased
risk of endometrial cancer in women receiving
tamoxifen, routine screening tests such as
ultrasound and biopsy or endometrial
aspiration are not indicated.
Hematology and Oncology:Question 80
The correct answer is A
Educational Objectives
Recognize the clinical presentation of
bronchoalveolar cell carcinoma.
Critique
This is a classic presentation of
bronchoalveolar cell carcinoma. The fact that
the patient is an older woman who is a
nonsmoker suggests she has a type of
adenocarcinoma, of which bronchoalveolar cell
carcinoma is considered a subtype.
The tumor cells of bronchoalveolar cell
carcinoma are better differentiated than those
of the usual adenocarcinoma, and the disease
follows a more indolent course. Many patients
live for months without requiring palliative
therapy. The disease is usually bilateral when
multifocal or metastatic. Infiltrates and nodules
are often seen. Palliative chemotherapy is
usually offered when patients become
symptomatic. Overt metastases to other
organs are much less common for this type of
cancer at presentation.
Hematology and OnCology:Question 81
The correct answer is B
Educational Objectives
Monitor a patient after primary and adjuvant
systemic therapy for breast cancer.
Critique
Breast cancer survivors have special concerns.
One of these is whether they should undergo
special studies routinely to detect
asymptomatic, occult metastases. No trials
have demonstrated that doing so provides any
benefit to the patient, and two prospective
randomized trials demonstrated that frequent,
routine restaging with chest radiographs and
hepatic imaging failed to result in improved
survival or quality of life. Therefore, the value
of such testing is questionable at best, and
harmful at worst. Women who are rendered
postmenopausal by treatment require special
counseling regarding sexual dysfunction and
prevention of osteoporosis. Although estrogen
is the most effective treatment for these
symptoms, it is now established that estrogen
replacement therapy with progesterone
modestly increases the risk of a new primary
breast cancer.
Hematology and Oncology:Question 82
The correct answer is C
Educational Objectives
Recognize the role of second-line systemic
chemotherapy in patients with metastatic
colorectal cancer.
Critique
Data from randomized clinical trials have
shown that second-line chemotherapy
regimens using drugs with established activity
in colorectal cancer are superior to best
supportive care in patients with good
performance status whose cancer has
progressed on chemotherapy. Because this
patient’s performance status is good, and she
wants to continue receiving therapy, switching
to a second-line chemotherapy regimen is
reasonable, preferably in the context of a
clinical trial.
Oxaliplatin has recently been approved for use
in combination with infusional 5-fluorouracil
and leucovorin in patients whose colorectal
cancer has progressed while they were taking
irinotecan given with 5-fluorouracil and
leucovorin. Prospective studies have not shown
metastatectomy to be beneficial for patients
with more than three liver lesions, nor in
patients with metastases to liver and lungs.
There is no established role for marrow-
ablative high-dose chemotherapy regimens in
the treatment of patients with metastatic
colorectal cancer.
Hematology and Oncology:Question 83
The correct answer is D
Educational Objectives
Diagnose and treat gastric mucosa-associated
lymphoid tissue (MALT) lymphoma.
Critique
The first course of therapy should be
eradication of the Helicobacter pylon
organisms. In about half the cases of gastric
MALT lymphoma associated with H. pylon
infection, eradication of the infection induces a
permanent complete remission.
In half of these patients, the proliferation of
the tumor has become dissociated from the
presence of the H. pylon antigens. If the tumor
does not regress within a year after eradication
of the organism and the absence of the
organism is confirmed and persistent,
chemotherapy can be used effectively to treat
the disease. Tumor progression after H. pylon
eradication could indicate acceleration in its
natural history and transformation to diffuse
large B-cell lymphoma.
Total gastrectomy is not indicated in the
management of gastric lymphoma. The main
role of surgery in this patient would be
diagnostic. The fluorouracil, doxorubicin,
mitomycin (FAM) regimen has been used in
patients with gastric carcinomas but is not
used in lymphoma. Cyclophosphamide,
hydroxydaunomycin, vincristine, and
prednisone (CHOP) is effective therapy for
some lymphomas, and the treatment of choice
for localized nodal or extranodal diffuse large
B-cell lymphomas is four cycles of CHOP plus
involved-field radiation therapy. CHOP is
generally not considered the first choice of
therapy in the setting of gastric MALT
lymphoma, however, because this form of
lymphoma appears to be strongly influenced
by antigens that are components of H. pylon.
Hematology and Oncology:Question 84
The correct answer is C
Educational Objectives
Treat breast cancer with adjuvant radiation
and systemic therapy.
Critique
Overall mortality from breast cancer has been
declining over the past 10 years in the Western
world, due in part to widespread application of
adjuvant systemic therapy. However, these
therapies have toxicities and therefore must be
applied judiciously.
Chest wall radiation has been shown to
decrease both local recurrences and systemic
recurrences by 65% and 25%, respectively, of
whatever they were destined to be. If this
woman had a larger tumor or many positive
lymph nodes, chest wall radiation would be
appropriate. However, given her excellent
prognosis, the benefits of chest wall radiation,
like those of chemotherapy, are outweighed by
the risks.
Tamoxifen is the drug of choice in this
situation. Aromatase inhibitors, like
anastrozole, may be slightly more effective,
but they appear to increase the risk of
osteoporosis and bone fractures, and long-
term side effects are not known. In a patient
with such a favorable prognosis, these
unknown risks are not worth what must be a
very small incremental benefit for anastrozole.
A recently reported prospective randomized
trial has suggested that the combination of
tamoxifen and anastrozole is no better than
tamoxifen alone, and it may be worse than
anastrozole alone.
This patient has a very good prognosis. The
chances that her disease will recur and that
she will die over the next 10 years are no more
than 10%. Although chemotherapy reduces
these odds, the proportional reduction in
recurrence for a woman in her 60s is no more
than 15%. Most guidelines panels, patients,
and their physicians do not believe this small
benefit outweighs the risks.
Hematology and Oncology:Question 85
The correct answer is E
Educational Objectives
Recognize the limitations of palliative
chemotherapy in metastatic non-small-cell
lung cancer
Critique
The patient’s performance status would be the
limiting factor. Palliative chemotherapy offers
modest prolongation of overall survival (8 to
16 weeks), but only in patients with adequate
performance status. Patients in bed more than
50% of the day derive no benefit from
palliative chemotherapy, and most large trials
even suggest decreased survival for these
patients with poor performance status who are
exposed to cytotoxic agents.
Patients whose status is adequate can be
offered chemotherapy, and the choice of drugs
can be modulated by factors such as jaundice.
Hypercalcemia can be treated with
bisphosphonates (intravenous pamidronate or
zoledronate) concomitant with chemotherapy.
Hematology and Oncology:Question 86
The correct answer is C
Educational Objectives
Recommend interventions of proven benefit in
decreasing the risk of lung cancer death in
smokers.
Critique
The antidepressant bupropion has been shown
to double the smoking cessation rate at 1 year
in a placebo-controlled trial of cigarette
smokers. Because smoking cessation is the
only proven way to decrease smokers’ risk of
dying of lung cancer, prescribing bupropion is
the best option for this patient.
Potential strategies to decrease the risk of lung
cancer death in a patient who is having
difficulty stopping smoking include more
intensified smoking cessation efforts,
medication to limit the cellular or DNA damage
of the tobacco carcinogens, or screening to
detect and cure small tumors. A number of
studies have been undertaken.
In placebo-controlled randomized clinical trials
of 3-carotene to prevent lung cancer in
smokers, the incidence and mortality rate of
lung cancer were increased in the 13-carotene
study groups. Therefore, on the basis of
current evidence, 13-carotene supplementation
is probably contraindicated for the prevention
of lung cancer in smokers.
Although isotretinoin was shown to decrease
the incidence of second primary tumors in
patients treated for a primary squamous cell
cancer of the head and neck, it did not improve
the overall survival rate. In a study of patients
who had been treated for early-stage non-
small-cell lung cancer, those randomized to
receive isotretinoin did no better than those
randomized to receive placebo. Furthermore,
there was a trend toward increased lung
cancer recurrence and lung cancer death in
subjects who were smokers at study baseline.
In randomized trials, sputum cytology has not
been shown to decrease the rate of death from
lung cancer in smokers. In the Mayo Lung
Project, monitoring of cigarette smokers by
sputum cytology plus chest radiography every
4 months for 6 years was compared with usual
care as a control arm. Although more surgically
resectable cancers were detected in the
screening arm, there was no decrease in the
number of late-stage lung cancers.
Furthermore, there was no decrease in lung
cancer mortality in the screened arm.
Spiral CT has been shown to detect early-stage
lung cancers, but the false-positive rate is very
high, and the effect on lung cancer mortality is
unknown. Several randomized trials are in
progress to test the net benefits and harms.
Hematology and Oncology:Question 87
The correct answer is D
Educational Objectives
Recognize the presenting features of
hyponatremia of malignancy and the
association with small-cell lung cancer.
Critique
This patient with mild hyponatremia is newly
diagnosed with small-cell lung cancer. He has
no symptoms that are clearly attributed to the
hyponatremia (the fatigue may also be due to
the underlying cancer), and therefore initiating
nonspecific therapy such as fluid restriction
with or without demeclocycline therapy is not
warranted. The most important therapeutic
intervention is to treat the underlying cancer.
Combination chemotherapy would be the
appropriate treatment.
Because he has an established diagnosis of
small-cell lung cancer, confirming that the
laboratory studies meet the criteria for
syndrome of inappropriate diuretic hormone
secretion, ruling out other causes such as
adrenal insufficiency and discontinuing drugs
that might possibly lead to free water retention
(such as thiazide diuretics) is for academic
rather than practical reasons.

Hematology and Oncology:Question 88


The correct answer is D
Educational Objectives
Initiate treatment of testicular carcinoma.
Critique
This patient has testicular carcinoma as
manifested by a mass and abnormal markers.
He cannot have pure seminoma, because both
his serum ct-fetoprotein and β -human
chorionic gonadotropin levels are elevated. The
next appropriate step would be to perform an
inguinal orchiectomy.
Retroperitoneal lymph node dissection or
combination chemotherapy might be
appropriate after a diagnosis is made and
clinical staging is complete. Pelvic irradiation is
not indicated in the management of patients
with newly diagnosed nonseminomatous germ
cell tumors.
Hematology and Oncology:Question 89
The correct answer is B
Educational Objectives
Recognize the most important risk factor for
ovarian cancer.
Critique
This patient should be given standard medical
care, including an annual rectovaginal pelvic
examination. Because this patients family
history gives no evidence of ovarian cancer
and because she is young, she should be
reassured. Familial cancer syndromes account
for only 5% to 10% of all cases. Apart from
age, ovarian cancer in a first-degree relative is
the most important risk factor for ovarian
cancer. There is no established role for ovarian
cancer screening. There is no need to screen
for BRCA-1 and BRCA-2 because the family
history shows no evidence of this genetic
mutation.
Hematology and Oncology:Question 90
The correct answer is A
Educational Objectives
Improve the quality of life for breast cancer
survivors.
Critique
Recently reported pilot and prospective
randomized clinical trials have demonstrated
that selective serotonin reuptake inhibitors, in
particular fluoxetine, venlafaxine, and
paroxetine, reduce the incidence and severity
of hot flushes by more than half in 50% to
75% of patients, compared with similar
reductions in only 20% to 30% of patients
taking placebo. Over-the-counter,
nonhormonal, nongreasy vaginal preparations
(such as Replens® and Astroglide®) are
frequently, although not always, helpful in
facilitating intercourse.
Postmenopausal symptoms are quite common
in women taking tamoxifen after
chemotherapy. Atrophic vaginitis and
dyspareunia as a consequence of menopause
can be so severe that women sometimes
discontinue taking tamoxifen, even though it
provides clear-cut benefits in regard to
preventing recurrence of incurable cancer.
Sympathetic counseling is appropriate but not
sufficient. Symptoms may lessen or resolve
with time but sometimes they persist
throughout treatment with tamoxifen. Although
estrogen is the most effective treatment for
these symptoms, it is now established that
estrogen replacement therapy with
progesterone modestly increases the risk of a
new primary breast cancer.
To date, no prospective randomized trials have
demonstrated that estrogen replacement
therapy increases the risk of breast cancer
recurrence or death, but the most successful
therapies in estrogen receptor-positive breast
cancers are anti-estrogenic (selective estrogen
receptor modulaters and aromatase inhibitors).
Therefore, most oncologists are unwilling to
prescribe estrogens, and patients with breast
cancer are reluctant to take them unless
nonhormonal therapies are ineffective.
Tamoxifen has estrogenic agonist properties in
the uterus, and, like unopposed estrogen,
increases the risk of uterine cancer two- to
threefold. This modest relative risk translates
to only a small absolute risk, and routine
uterine screening is therefore not
recommended. Although symptoms such as
pelvic discomfort, pain, or vaginal bleeding
should be thoroughly evaluated, this patients
dyspareunia is a symptom of vaginal estrogen
depletion and does not suggest endometrial
cancer.
Hematology and Oncology:Question 91
The correct answer is B
Educational Objectives
Understand the role of screening for prostate
cancer
Critique
The incidence of prostate cancer increased
dramatically when PSA screening began to be
used. At this time, it is unclear whether
screening will lead to a change in prostate
cancer mortality or overall mortality rates. It is
hoped that the Prostate, Colorectal, Lung and
Ovary randomized cancer screening trial will
answer that question. This patient should be
counseled that prostate cancer can be detected
earlier with screening, but it is not known
whether earlier treatment would improve his
longevity. In addition, he should be advised of
the significant morbidity that could accompany
the treatment of early-stage prostate cancer.
Screening does lead to the earlier detection of
prostate cancer. Most patients are
asymptomatic; therefore a lack of symptoms
does not exclude the possibility of an elevated
PSA. The digital rectal examination is not as
sensitive as a PSA screen. There are, however,
tumors that will be detected on digital rectal
examination in patients with a PSA lower than
4.0 ng/mL.
Hematology and Oncology:Question 92
The correct answer is A
Educational Objectives
Order appropriate further treatment for
patients with small-cell lung cancer who
achieve clinical complete remission.
Critique
Patients with small-cell lung cancer of limited
or extensive stage should be offered
prophylactic cranial irradiation. A meta-
analysis of 7 randomized trials evaluating this
therapy in patients who achieve a complete
response demonstrated 15% to 21%
improvement in disease-free and overall
survival at 3 years. Additional chemotherapy,
even with other agents, has not been shown to
benefit complete responders with limited-stage
disease. Surgery is rarely considered for small-
cell lung cancer because micrometastatic
disease is almost always present, and response
to chemotherapy is the key to survival.
Hematology and Oncology:Question 93
The correct answer is D
Educational Objectives
Recognize when palliative resection of the
primary tumor should be considered in a
patient who presents with metastatic colon
cancer.
Critique
Systemic chemotherapy should be offered to
this patient because he has symptoms related
to his cancer and his performance status is still
reasonably good. Clinical trials have shown
that immediate institution of systemic 5-
fluorouracil-based therapy is superior to best
supportive care in the treatment of metastatic
colorectal cancer, in terms of time to disease
progression, time to symptom progression, and
overall survival.
The indications for palliative resection of the
primary tumor in a patient with metastatic
disease are obstruction (existing or imminent)
and bleeding. The patient has no symptoms of
obstruction and has only mild anemia.
Therefore, resection of the primary tumor
would not affect his overall survival, would
subject him to the risks of surgery, and would
delay the initiation of systemic therapy.
Regional chemotherapy to the liver is not
indicated because the patient’s disease is not
confined to the liver. There is no evidence to
suggest that combined regional and systemic
chemotherapy is beneficial in patients with
metastatic disease involving the liver and
extrahepatic sites.
Because systemic chemotherapy is not
curative, best supportive care can be
considered if the patient chooses not to receive
chemotherapy.
Hematology and Oncology:Question 94
The correct answer is C
Educational Objectives
Recognize the risk factors for metastasis in
patients with a biochemical failure after
surgery for prostate cancer.
Critique
The natural history of a rise in PSA levels after
radical prostatectomy is poorly understood.
The best series (Pound et al.) found that
patients with a recurrent rise in PSA level
within 2 years of surgery, with a high Gleason
score and a rapid PSA doubling time, were at
the greatest risk for the development of
metastatic disease. This patient, with a late,
slow rise in his PSA level, is at low risk for the
rapid development of metastatic disease.
There is no evidence that patients with
recurrent disease treated with a prostatectomy
have an outcome different from patients
treated with other primary therapies. Neither
the age of the patient nor the PSA nadir has
been shown to affect the length of time to
metastatic disease in this population.
Hematology and Oncology:Question 95
The correct answer is E
Educational Objectives
Understand the role of adjuvant therapy in
completely resected non-small-cell lung
cancer.
Critique
A 50% to 70% cure rate with surgery alone
would be predicted for early-stage disease.
Randomized trials have failed to show any
survival advantage as yet for any adjuvant
chemotherapy. Meta-analyses confirm that
additional radiation therapy confers no benefit
in this setting and in fact may be deleterious to
survival in early-stage disease.
The fact that 30% to 50% of patients like this
one will develop metastatic disease within a
few years makes them good candidates for
clinical trials involving novel therapies (such as
immunomodulators or vaccines), with or
without standard chemotherapy.
Hematology and Oncology:Question 96
The correct answer is A
Educational Objectives
Treat advanced ovarian cancer.
Critique
Randomized controlled trials have established
that a combination of either cisplatin or
carboplatin with paclitaxel is the standard of
care for the treatment of advanced ovarian
cancer (stage III/IV). Docetaxel is another
taxane, but to date has not been shown to
offer an advantage over paclitaxel in first-line
therapy of advanced ovarian cancer.
The addition of intraperitoneal chemotherapy
may be beneficial in patients with optimally
debulked stage III disease (no tumor larger
than 1 cm) after completion of systemic
chemotherapy, but this approach requires
second-look laparotomy to confirm miminal
residual disease and is not recommended if the
patient has adhesions or the treatment center
is not experienced with regional therapy.
More aggressive approaches should be
recommended only in the context of a clinical
trial.
Hematology and Oncology:Question 97
The correct answer is B
Educational Objectives
Know the effects of tamoxifen, other than
reduction of breast cancer risk, when used in
the cancer prevention setting.
Critique
The Breast Cancer Prevention Trial (BCPT)
compared 5 years of tamoxifen use versus
placebo in women at elevated risk for breast
cancer. An important life-threatening adverse
outcome associated with tamoxifen was an
increased risk of deep venous thrombosis or
pulmonary embolism. The rate of pulmonary
embolism was 0.69 per 1000 women in the
tamoxifen arm versus 0.23 per 1000 in the
placebo arm (risk ratio of 3.01, 95% CI 1.15 to
9.27). The rates of deep venous thrombosis
were 1.34 versus 0.84 versus 0.84 per 1000,
respectively (RR 1.6, 95% CI 0.91 to 2.86).
Although tamoxifen decreases plasma
cholesterol levels, it has not been shown to
have a statistically significant effect on
ischemic heart disease rates in women with or
without a history of heart disease. Changes in
serum lipid levels appear to be a poor
surrogate for the impact of hormonal
manipulations on ischemic heart disease. For
example, hormone replacement therapy (HRT)
with estrogen plus progestin has recently been
shown to increase the risk of coronary artery
disease in healthy postmenopausal women
despite the fact that HRT lowers serum lipid
levels.
The BCPT showed no increase in the incidence
of any cancer other than uterine endometrial
cancer, with a risk ratio of about 2.5. This
patient is, however, presumably not at risk for
endometrial cancer, given her prior
hysterectomy.
There have been anecdotal reports of
depression and mood disorders in association
with tamoxifen. The BCPT found no difference
in the incidence of depression between patients
taking tamoxifen or placebo, irrespective of
underlying risk of depression (on the basis of
medical history obtained at study baseline).

Hematology and Oncology:Question 98


The correct answer is B
Educational Objectives
Treat adenocarcinoma presenting with diffuse
peritoneal involvement in a woman.
Critique
When a woman presents with adenocarcinoma
and diffuse peritoneal involvement, therapy
should be directed toward ovarian cancer, even
if no ovarian mass is evident. Patients with
ovarian cancer can have long-term disease-
free survival at 10 years, even with this
presentation. Management involves debulking
surgery, followed by combination
chemotherapy.
Ovarian cancer rarely spreads to bone, and a
bone scan would not be considered unless
symptoms of bone involvement were evident.
There is no established role for positron
emission tomography scan in this setting, and
a false-positive scan should not preclude a
curative approach with surgery and
chemotherapy.
Radiation therapy plays no role in this setting.
Although combination chemotherapy with a
taxane and a platinum agent is an important
treatment for this woman, the standard of care
mandates debulking surgery first.
Hematology and Oncology:Question 99
The correct answer is D
Educational Objectives
Understand which patients with newly
diagnosed breast cancer should have
mastectomy vs. breast-conserving therapy and
the therapy involved in the latter.
Critique
Breast-conserving (or preserving’) therapy is
as effective as and cosmetically more
acceptable than mastectomy. There are,
however, some relative contraindications,
mostly related to an unacceptably high risk of
subsequent recurrence of breast cancer (>15%
over 5 to 10 years). They include the presence
of grossly unresected breast cancer, relatively
large lesions, and relatively small breasts. The
presence of widespread ductal carcinoma in
situ is also a high risk factor for recurrence in
the breast.
Several randomized trials have demonstrated
that even with systemic therapy, recurrence of
breast cancer is quite high in women who do
not receive radiation therapy. In most studies,
the risk of in-breast recurrence for patients
who receive radiation therapy is less than
10%, whereas that for lumpectomy alone
exceeds 40%. Therefore, because most
recurrences in the breast are treated by
mastectomy, an initial course of radiation
therapy will result in fewer mastectomies in
the long run. Patients who have previously had
mediastinal or ipsilateral chest radiation (for
example, women with a history of Hodgkin’s
disease) and patients at high risk of toxicity
from radiation (for example, women with lupus
dermatitis) are also poor candidates for breast-
conserving therapy.
This patient appears to be an ideal candidate
for breast-conserving therapy, which would be
completed with radiation therapy.
Because this patient has positive axillary lymph
nodes, there is no role for tamoxifen; she is
unlikely to benefit, given her negative hormone
receptors.

Hematology and Oncology:Question 100


The correct answer is E
Educational Objectives
Treat lymphadenopathy in a young smoker.
Critique
A chest radiograph is indicated because of the
dyspnea on exertion and the night sweats. The
age of the patient tends to favor observation
as the standard approach, because
lymphadenopathy in young people generally
has a benign cause. The supraclavicular
location, the size of the node (greater than 2
cm in diameter), and the presence of systemic
symptoms, however, heighten the suspicion of
more serious illness. Watching and waiting
would not be appropriate under these
circumstances.
Referral to an otolaryngologist might be
indicated because smoking-related head and
neck cancers can present with
lymphadenopathy in the absence of an obvious
mucosal lesion. The patient’s age, however,
and the fact that she abstains from alcohol
make head and neck cancer unlikely.
There is no proven role for empiric use of
antibiotics in the management of a patient with
unexplained lymphadenopathy. Needle
aspiration is not the diagnostic procedure of
choice in a patient with undiagnosed
lymphadenopathy because it fails to obtain
adequate tissue for histologic, immunologic,
and genetic analysis.
Hematology and Oncology:Question 101
The correct answer is A
Educational Objectives
Recognize the various nonspecific
manifestations of multiple myeloma
Critique
The patient is older than 40 years, has
unexplained anemia, renal dysfunction, and
recurrent infections:
This scenario is common in patients with
multiple myeloma. In addition, his total protein
level is increased in the face of a normal
albumin level, meaning that the gamma
globulin fraction is increased. Serum protein
electrophoresis would document the presence
of a monoclonal gammopathy and he would
need to be staged with a bone marrow
evaluation (to quantitate marrow plasma cells)
and skeletal survey (to search for lytic bone
lesions). It would also be necessary to
measure his serum 32
-microglobulin level.
Anemia is not a common feature of chronic
pyelonephritis until renal failure develops. Iron
deficiency anemia is most often noted in the
setting of chronic bleeding, of which this
patient has no history or symptoms.
When cancer of the colon is associated with
anemia, the cause is chronic blood loss and the
mechanism is iron deficiency, which would
produce microcytic, hypochromic anemia.
Small-cell lung cancer could produce similar
generalized symptoms but is rarely associated
with hypercalcemia and is not associated with
recurrent urinary tract infection.
The diagnosis of myeloma should be
considered in any patient older than 40 years
with unexplained anemia, renal failure, bone
lesions, or recurrent infections.

Hematology and Oncology:Question 102


The correct answer is C
Educational Objectives
Recognize the indications for surgical therapy
after diagnostic testing for non-small-cell lung
cancer.
Critique
Surgical therapy is offered in two situations for
patients with non-small-cell lung cancer and
obvious metastasis. First, surgery is considered
if there is reasonable curative intent. In the
four options presented, curative intent is most
unlikely, especially because microscopic
metastases are almost always present when an
overt metastasis is demonstrated on chest
radiograph.
Second, surgical therapy is considered in
selected situations of isolated single
metastasis. A randomized study showed a 1-
year survival advantage for patients with a
single metastasis to the brain treated with
surgery and radiation therapy compared with
radiation therapy alone.
A finding of bone metastasis would be best
treated by palliative radiation therapy or
chemotherapy or both. A finding of liver
metastasis or cytologically positive pleural
effusion would be best treated by palliative
chemotherapy.
Surgical therapy might also be called for to
relieve an epidural compression if radiation
therapy is contraindicated or if a second lung
nodule is found that is suspected of being
cancerous. In the latter case, resection of both
lesions might even offer the possibility of cure,
especially if the second lesion is not malignant.

Hematology and Oncology:Question 103


The correct answer is D
Educational Objectives
Treat metastatic breast cancer.
Critique
A recently reported prospective randomized
clinical trial in patients with metastatic breast
cancer has demonstrated that the combination
of trastuzumab and paclitaxel are at least
additive, if not synergistic, in women with
HER2-positive disease. Patients treated with
this combination had significantly longer
survival than those treated initially with
chemotherapy alone.
Few if any patients are cured of metastatic
breast cancer. For many patients, however,
systemic therapy probably does result in
modest survival prolongation of several
months to a few years compared with best
supportive care, and the palliative benefits are
well recognized. Therefore, this patient should
receive treatment. The key to effective
palliation is balancing the potential benefits
with the expected side effects of specific
therapies.
In this regard, endocrine therapy is almost
always less toxic than chemotherapy. The
steroid hormone receptors in her tumor,
however, have been consistently negative.
Therefore, she is very unlikely (less than 10%)
to respond to endocrine therapy of any sort.
Given that she has symptomatic visceral
disease, she should be treated with therapy
that has a higher chance of working, even
though she will suffer more side effects from it.

Preclinical studies and some preliminary clinical


data suggest that aromatase inhibitors are not
likely to be effective and may be dangerous in
premenopausal women.

Hematology and Oncology:Question 104


The correct answer is C
Educational Objectives
Recognize the clinical significance of the
Gleason score in patients with newly diagnosed
prostate cancer.
Critique
This patients Gleason score indicates that he
would have a high risk of recurrence after
prostatectomy. The Gleason score is obtained
by grading from 1 to 5 the two predominant
histologic types of adenocarcinoma of the
prostate and adding the grades. This score is
an extremely important prognostic factor in
predicting the biology of clinically localized
prostate cancer. The scores range from 2 to
10, with the highest scores predictive of
patients who will not have organ-confined
disease. This patient with a high Gleason score
(despite the fact that his examination reveals
no abnormalities and his PSA is low) would
have only about a 37% probability of having
organ-confined disease. There is some
variability among pathologists when scoring
prostate cancer using this system. The
concordance rate is high enough, however,
that the Gleason score can be used as a
prognostic variable. The Gleason score is not
influenced by the absence of symptoms in a
patient.
Hematology and OnCology:Question 105
The correct answer is A
Educational Objectives
Provide supportive care in the management of
pancreatic cancer.
Critique
This patient almost certainly has metastatic
pancreatic cancer. The median time to disease
progression of patients with advanced
pancreatic cancer-related symptoms is only a
few months, even with the best available
chemotherapy. This patient is not a good
candidate for chemotherapy; therefore, best
supportive care and referral to a hospice is the
best recommendation. Since the patient’s
upper abdominal discomfort is vague and mild,
a trial of narcotic analgesics is warranted.
Even if the patients clinical status improves
slightly as he recovers from the pulmonary
embolism, his baseline performance status is
poor because of his other medical conditions
and cachexia caused by the cancer. As such,
he is unlikely to benefit from palliative
chemotherapy. Palliative radiation therapy is
indicated only for the relief of local symptoms.
Diagnostic biopsy is not needed because it
would not influence the current management.
Hematology and Oncology:Question 106
The correct answer is B
Educational Objectives
Recognize the potential toxicities of androgen
suppression.
Critique
It is important for this patient to realize that
with long-term androgen ablation he will be at
risk for osteopenia and potential nonpathologic
fractures. The use of PSA screening has led to
the earlier detection of prostate cancer
recurrence after definitive local therapy. Many
patients are asymptomatic and have no clinical
manifestations of cancer except for an
abnormal PSA level. There is no standard
therapy for these patients. Although there is
no evidence that early hormone therapy can
affect survival in this group, many patients will
commence long-term androgen ablation.
Leuprolide treatment should not impair urinary
flow in this asymptomatic patient. Weight gain
and loss of muscle mass can occur with
testosterone suppression. In addition, many
patients note loss of hair on their arms and
legs with leuprolide therapy.
Hematology and Oncology:Question 107
The correct answer is D
Educational Objectives
Treat limited-stage small-cell lung cancer.
Critique
Clinical staging for small-cell lung cancer
classifies disease as limited-stage (unilateral
lung and mediastinal involvement) or
extensive-stage (bone, liver, brain, or bilateral
lung metastases). Approximately 25% of
patients with limited-stage small-cell lung
cancer will be free of disease 5 years after
combined-modality therapy. A meta-analysis of
randomized trials documents that combined-
modality therapy is the preferred option, with
chemotherapy and radiation therapy
administered concomitantly at the start of
treatment. Surgery is not indicated because of
the systemic nature of small-cell lung cancer.
Hematology and Oncology:Question 108
The correct answer is C
Educational Objectives
Treat symptomatic hypercalcemia of
malignancy.
Critique
The patient has hypercalcemia, the most
common metabolic complication of malignancy.
She is profoundly dehydrated. Despite her
history of congestive heart failure, the most
appropriate initial therapy is to rehydrate as
vigorously as possible with normal saline.
Furosemide must not be given until the patient
has first been adequately rehydrated.
Administration of a bisphosphonate is not an
appropriate initial therapeutic intervention in
this patient because of her volume depletion;
in addition, it would slow the necessary
decrease in her serum calcium level. Although
corticosteroids can be considered as an
adjunctive therapy in a patient with a
potentially hormone-responsive tumor such as
breast cancer, the top priority in this seriously
dehydrated patient is volume repletion.
Hematology and Oncology:Question 109
The correct answer is C
Educational Objectives
Select the best test to evaluate the histology of
carcinoma of unknown primary site.
Critique
When a poorly differentiated cancer is negative
for leukocyte common antigen and cytokeratin,
the most likely diagnosis is melanoma (which
can be amelanotic) or sarcoma. An inguinal
lymph node would be an expected site for
melanoma but very unusual for sarcoma. The
best test for establishing melanoma is a stain
for S-100, although other immunoperoxidase
stains can be employed (melan A, vimentin). If
positive, an exhaustive search of the skin
should be undertaken to locate the primary
tumor, focusing on sites that drain to the right
inguinal region.
Melanoma can be further confirmed with
electron microscopy. Sometimes the primary
site may not be evident, even when the tumor
is known to be a melanoma. In general,
primary sites might include the eye or even
mucosal areas that can receive sun exposure.

Hematology and Oncology:Question 110


The correct answer is B
Educational Objectives
Recognize the presentation of superior vena
cava syndrome and identify the most useful
initial diagnostic test.
Critique
The presentation is consistent with obstruction
of the superior vena cava (SVC), not
pulmonary embolism. Because the patient does
not have any life-threatening symptoms such
as upper airway obstruction, he should
undergo diagnostic procedures to confirm the
cause and guide the most appropriate therapy.

A contrast-enhanced CT is the most useful test


to diagnose SVC syndrome and to detect the
presence of a mediastinal mass; it may also
provide an opportunity for CT-guided needle
biopsy. CT findings indicating SVC obstruction
may include intraluminal filling defects of the
SVC, decreased opacification of the innominate
vein or SVC below the level of obstruction,
opacification of collateral vessels, and external
compression of mediastinal venous channels.
Initiating both pulmonary and thoracic surgery
consultations early on is prudent to facilitate
the work-up and obtain a tissue diagnosis.
Administration of radiation therapy while the
diagnostic work-up is occurring is not
necessary because the patient’s symptoms do
not require emergency intervention. A venous
dye study would not add to the findings of the
CT. Mediastinoscopy with biopsy is more
invasive than necessary for this diagnosis. The
clinical picture does not suggest pulmonary
embolism; ventilation/perfusion scan is not
useful to establish a diagnosis in SVC
syndrome
Hematology and Oncology:Question 111
The correct answer is B
Educational Objectives
Treat metastatic testicular carcinoma with
chemotherapy.
Critique
Testicular carcinoma is the rare metastatic
solid tumor that can be routinely cured with
combination chemotherapy such as bleomycin,
etoposide, and cisplatin. The introduction of
cisplatin approximately 25 years ago has
resulted in effective therapy, even for patients
with advanced disease. Therefore, this patient,
whose tumor has recurred in the lungs (a
common location after retroperitoneal lymph
node dissection), should be treated with
curative intent.
Surgical resection of lung lesions is sometimes
appropriate in testicular cancer, generally after
treatment with chemotherapy if there are
remaining nodules. Teratoma is not
chemoresponsive, and, after chemotherapy,
residual tumor of this histologic type may need
to be resected. Radiation is sometimes used
for palliation, but chemotherapy would be the
appropriate choice for multiple asymptomatic
lesions. There is no advantage to watchful
waiting in a potentially curable patient.
Hematology and Oncology:Question 112
The correct answer is D
Educational Objectives
Recommend appropriate cancer risk
assessment and testing.
Critique
Her family should be reassured that they are
not at particularly high risk for cancer,
especially the younger ones, and both she and
her family should be encouraged to follow
routine screening and risk reduction guidelines.

Although several members of this patients


family have had cancer, in each case the
cancer was one that occurs commonly in the
age groups in which these individuals were
diagnosed. Hereditary genetic cancer
syndromes are usually associated with cancers
in patients younger than 60 years, and they
often result in multiple cancers in the same
person. Furthermore, there is no specific
pattern of organ-oriented cancers, as one
might see in families that harbor BRCA-1 or
BRCA-2 (breast and ovarian cancers). Even
though there are two instances of breast
cancer in the family, both occurred in elderly
women. Furthermore, the patients ancestry is
not one in which hereditary genetic cancer
syndromes are more prevalent.
Taken together, these data suggest that the
cancers observed in this family are sporadic
and it is very unlikely that her family harbors a
germ-line abnormality in any of the currently
identified tumor suppressor genes. Testing is
very expensive and can be associated with
occasional false-positive results, which could
have devastating consequences for her or
members of her family.
Hematology and Oncology:Question 113
The correct answer is C
Educational Objectives
Apply the principles of supportive care in
multiple myeloma.
Critique
The only one of the options that is
contraindicated is intravenous gamma globulin.
Patients with multiple myeloma have
hypogammaglobulinemia when the paraprotein
is not considered and are susceptible to
infections. Intravenous gamma globulin,
however, has not been shown to reduce the
incidence of serious infections when used
prophylactically. Monthly intravenous gamma
globulin is not routinely administered to
patients with myeloma. However, it can be
used in patients who have recurrent episodes
of bacterial infections or sepsis that are life-
threatening.
The supportive care of the patient with
myeloma is as important as the anticancer
treatment to the patients quality of life.
Erythropoietin has been shown to alleviate the
anemia and improve quality of life. Radiation
therapy may be necessary to control localized,
symptomatic bony destructive lesions.
Pain management is an important part of the
care of many cancer patients. The goal of total
relief of pain should not be sacrificed to fears
of causing narcotic addiction in this population.
Bisphosphonates such as pamidronate or
zoledronate are effective agents at minimizing
bony destruction and bone fractures. They also
control hypercalcemia. Evidence is also
emerging for a role for bisphosphonates in
killing tumor cells, and randomized studies
have found that bisphosphonate therapy
improves patients’ length and quality of life.

Вам также может понравиться